You are on page 1of 179

के न्द्रीय विद्यालय संगठन, जयपुर सम्भाग SET-1

Kendriya Vidyalaya Sangthan, Jaipur Region


विषय: अर्थशास्त्र
Subject: Economics
अभ्यास पत्र
Practice Paper
कक्षा: XII समय: 3:00 घंटे अविकतम अंक: 80 अंक
Class: XII Time: 3:00 Hours Maximum Marks: 80 Marks
सामान्द्य वनर्देश:
(1) इस प्रश्न पत्र के र्दो खण्ड है:-
खण्ड-अ (पररचयात्मक समवि अर्थशास्त्र) खण्ड-ब (भारतीय अर्थव्यिस्र्ा का विकास)
(2) र्दोनों खण्डों के सभी प्रश्न करना अवनिायथ है | यद्यवप कु छ प्रश्नों में आन्द्तररक चयन प्रर्दान ककया गया है | प्रत्येक
प्रश्न के अंक उस प्रश्न के सामने अंककत है |
(3) प्रश्न संख्या 1 से 10 एिं 18 से 27 तक के सभी एक-एक अंक के अवत लघूत्तरात्मक / बहुविकल्पीय प्रश्न है | इन
प्रश्नों के उत्तर एक शब्र्द या एक िाक्य में र्दीवजए |
(4) प्रश्न संख्या 11 से 12 एिं 28 से 29 तक के सभी प्रश्न लघूत्तरात्मक प्रश्न है, जो कक तीन-तीन अंक के है | इन प्रश्नों
के उत्तर 60-80 शब्र्दों से अविक नहीं होने चावहए |
(5) प्रश्न संख्या 13 से 15 एिं 30 से 32 तक के सभी प्रश्न भी लघूत्तरात्मक प्रश्न है, जो कक चार-चार अंक के है | इन
प्रश्नों के उत्तर 80-100 शब्र्दों से अविक नहीं होने चावहए |
(6) प्रश्न संख्या 16 से 17 एिं 33 से 34 तक के सभी प्रश्न र्दीघथ-उत्तरात्मक प्रश्न है, जो कक छह-छह अंक के है | इन
प्रश्नों के उत्तर 100-150 शब्र्दों से अविक नहीं होने चावहए |
(7) सभी प्रश्नों के उत्तर यर्ा सम्भि प्रर्दत्त शब्र्द सीमा को ध्यान में रखते हुए संवक्षप्त, तथ्यात्मक एिं सटीक होने चावहए |
(8) इस प्रश्न पत्र को पढ़ने हेतु पंरह (15) वमनट का अवतररक्त समय आिंरटत ककया गया है |

General Instructions:
(1) This question paper contains two sections:
Section–A (Introductory Macro Economics) Section–B (Indian Economic Development)
(2) All questions in both the sections are compulsory. However, there is internal choice in some
questions. Marks for questions are indicated against each question.
(3) Question number 1 –10 and 18 – 27 are very short-answer / multiple choice questions
carrying 1 mark each. They are required to be answered in one word or one sentence each.
(4) Question number 11 – 12 and 28 – 29 are short-answer questions carrying 3 marks each.
Answers to them should normally not exceed 60-80 words each.
(5) Question number 13 – 15 and 30 – 32 are also short-answer questions carrying 4 marks
each. Answers to them should normally not exceed 80-100 words each.
(6) Question number 16 – 17 and 33 – 34 are long-answer questions carrying 6 marks each.
Answers to them should normally not exceed 100-150 words each.
(7) Question number 16 – 17 and 33 – 34 are long-answer questions carrying 6 marks each.
Answers to them should normally not exceed 100-150 words each.
(8) An additional 15 minutes has been allotted to read the question paper.
प्र. सं. खण्ड-अ (पररचयात्मक समवि अर्थशास्त्र) अंक
Q. NO. Section-A (Introductory Macro Economics) Marks
1. राष्ट्रीय आय के संबंि में वनम्नवलवखत कर्नों पर विचार कीवजए: 1
1. कारक लागत िह इनपुट लागत है, जो वनमाथता को िस्तुओं और सेिाओं के उत्पार्दन की प्रकिया में
खचथ करनी पड़ती है।
2. उत्पार्द की कारक लागत पर अप्रत्यक्ष करों को घटाने के बार्द बाजार लागत वनकाली जाती है।
ऊपर कर्दए गए कर्नों में से कौन-सा / से सही है / हैं ?
(a) के िल 1 (c) 1 और 2 र्दोनों
(b) के िल 2 (d) न तो 1 और न ही 2
Regarding the National Income, consider the following statements:
1. Factor cost is the input cost that the producer has to incur in the process of
producing goods and services.
2. The market cost is derived after deducting the indirect Taxes to the factor cost of
the product.
Which of the statement (s) given above is/are correct?
(a) 1 only (c) Both 1 and 2
(b) 2 Only (d) Neither 1 nor 2

2. यकर्द सीमांत उपभोग प्रिृवत्त (MPC) आय (Y) के सभी स्तरों के वलए औसत उपभोग प्रिृवत्त (APC) के 1
बराबर है, तो सही संगत उपभोग (C) फलन होगा: -
If Marginal Propensity to Consume (MPC) is equal to Average Propensity to
Consume (APC) for all levels of Income (Y), the correct corresponding
Consumption (C) function will be:-
(a) C = a + bY (c) C = bY
(b) C = a +
𝒃 (d) C = b𝒀𝟐
𝒀

3. वनम्नवलवखत अवभकर्न (A) और कारण (R) को ध्यानपूिथक पकढ़ए | 1


वनम्नवलवखत विकल्पों में से एक सही विकल्प का चयन कीवजए |
अवभकर्न (A): APC कभी भी शून्द्य नहीं हो सकता है जबकक यह 1 से अविक हो सकता है।
कारण (R): यकर्द आय शून्द्य है तो उपभोग व्यय हमेशा सकारात्मक होता है।
विकल्प:
(a) A सत्य है लेककन R गलत है।
(b) A गलत है लेककन R सत्य है।
(c) A और R र्दोनों सत्य हैं और R, A की सही व्याख्या है।
(d) A और R र्दोनों सत्य हैं लेककन R, A की सही व्याख्या नहीं है।
Read the following Assertion (A) and Reason (R).
Choose the correct alternative.
Assertion (A): APC can never be Zero while it can be greater than 1.
Reason (R): Consumption expenditure is always positive if income is Zero.
Alternatives:
(a) A is true but R is false.
(b) A is false but R is true.
(c) Both A and R are true and R is the correct explanation of A.
(d) Both A and R are true but R is not the correct explanation of A.
4. वित्त िषथ 2022-23 में भारत का चालू खाता घाटा बढ़कर 67 वबवलयन अमेररकी डॉलर हो गया जो 1
कक सकल घरे लू उत्पार्द का 2.0 प्रवतशत है। चालू खाता घाटा में िृवि –
(I) भारतीय रुपए को कमजोर कर सकता है |
(II) आयात को महँगा बना सकता है |
(III) अर्थव्यिस्र्ा पर महंगाई का असर पड़ सकता है |
उपरोक्त मे से ककतने पररणाम संभि हैं?
(a) के िल 1 (c) उपरोक्त सभी
(b) के िल II (d) उपरोक्त में से कोई नहीं
India’s current account deficit jumped to US $ 67 billion in the FY 2022-23 which
is 2.0 per cent of the GDP. Rise in the current account deficit –
(I) Can weaken Indian Rupee
(II) Make Import Costly
(III) Can have inflationary effect on the economy.
How many Consequences given above is/are possible?
(a) 1 only (c) All the three
(b) 2 Only (d) None of the above

5. संकीणथ मुरा के संर्दभथ में वनम्नवलवखत कर्नों पर विचार कीवजए - 1


1. इसमें के िल ककसी भी कर्दन जनता द्वारा रखे गए करें सी नोट शावमल हैं |
2. संकीणथ मुरा आपूर्तथ में सबसे अविक तरल वित्तीय संपवत्तयां होती हैं |
3. 'M1' को संकीणथ मुरा के रूप में जाना जाता है, क्योंकक इसमें के िल 100% तरल जमा शावमल है |
उपरोक्त कर्नों में से कौन-सा / से सही है / हैं?
(a) के िल 1 और 2 (c) के िल 1 और 3
(b) के िल 2 और 3 (d) 1, 2 और 3
With reference to narrow money, consider the following statements –
1. It includes, only the currency notes being held by the public on any given day
2. Narrow money supply contains the most liquid financial assets
3. ‘M1’ is known as narrow money, as it includes only 100% liquid deposit
Which of the statement (s) given above is/are correct?
(a) 1 and 2 only (c) 1 and 3 only
(b) 2 and 3 Only (d) 1, 2 and 3

6. बैंककं ग प्रणाली में तरलता के संर्दभथ में, वनम्नवलवखत कर्नों पर विचार कीवजए : 1
1. इसका तात्पयथ आसानी से उपलब्ि नकर्दी से है वजसकी बैंकों को अल्पकावलक व्यािसावयक और
वित्तीय जरूरतों को पूरा करने के वलए आिश्यकता होती है।
2. यकर्द बैंककं ग प्रणाली आरबीआई का शुि उिारकताथ है, तो वसस्टम तरलता को अविशेष में कहा जा
सकता है।
उपरोक्त कर्नों में से कौन-सा / से सही है / हैं?
(a) के िल 1 (c) 1 और 2 र्दोनों
(b) के िल 2 (d) न तो 1 और न ही 2
With reference to the liquidity in the banking system, consider the following
statements:
1. It refers to readily available cash that banks need to meet short-term business
and financial needs.
2. If the banking system is a net borrower to the RBI, then the system liquidity can
be said to be in surplus.
Which of the statement(s) given above is/are correct?
(a) 1 only (c) Both 1 and 2
(b) 2 only (d) Neither 1 nor 2

7. कर्दए गए वचत्र में प्रभािी मांग को _____________ बबंर्दु द्वारा र्दशाथया गया है |
In the given diagram, a situation of Excess Demand has been represented by ______. 1

(a) EY (b) KT ̅
(c) O𝑨 (d) K 𝒀𝒇

8. वनम्नवलवखत में से कौन-सा / से रुपये के मूल्यह्रास का प्रभाि है / हैं? 1


1. िस्तुओं एिं सेिाओं का आयात सस्ता हो जाता है।
2. िस्तुओं और सेिाओं का वनयाथत प्रवतस्पिी हो जाता है।
वनम्नवलवखत कू ट का उपयोग करके सही उत्तर का चयन कीवजए :
(a) के िल 1 (c) 1 और 2 र्दोनों
(b) के िल 2 (d) न तो 1 और न ही 2
Which of the following is/are the effect of the depreciation of the rupee?
1. Import of goods and services becomes cheaper.
2. Export of goods and services becomes competitive.
Select the correct answer using the codes given below:
(a) 1 only (c) Both 1 and 2
(b) 2 only (d) Neither 1 nor 2

9. वनम्नवलवखत जानकारी प्रर्दान की गई है :- 1


𝟑
C = 20 + Y एिं Y = 80
𝟒
इसमें APC का मूल्य क्या है?
Following information are given: -
𝟑
C = 20 + Y and Y = 80
𝟒
What is the value of APC?
(a) 0 (b) 1 (c) 0.5 (d) 0.075

10. वनवित विवनमय र्दर के संबंि में वनम्नवलवखत कर्नों पर विचार कीवजए: 1
1. वस्र्र विवनमय र्दर प्रणाली बाजार संचावलत है |
2. यह विवनमय र्दर की वस्र्रता का आश्वासन र्देता है।
3. FERs विवनमय र्दर को बनाए रखने के वलए सरकार पर भारी बोझ डालते हैं |
उपरोक्त कर्नों में से कौन-सा / से सही है / हैं?
(a) के िल 1 और 2 (c) के िल 1 और 3
(b) के िल 2 और 3 (d) 1,2 और 3
Regarding the Fixed Exchange Rate consider the following statements:
(1) The fixed exchange rate system is market driven
(2) It assures the stability of the exchange Rate.
(3) FERs put a heavy burden on the Government to maintain the Exchange rate
Which of the statement (s) given above is/are correct?
(a) 1 and 2 only (c) 1 and 3 only
(b) 2 and 3 Only (d) 1,2 and 3

11. यूिेन युि वछड़ने के बार्द से एक कर्दन की सबसे बड़ी वगरािट झेलने के बार्द रुपया वपछले बंर्द भाि के 3
79.98 के मुकाबले 80.87 पर बंर्द हुआ । 2022 में घरे लू मुरा में अब तक 8 प्रवतशत की वगरािट आई
है । डॉलर इं डक्
े स, जो गुरुिार को 20 साल के उच्च स्तर पर पहुंच गया, कै लेंडर िषथ में अब तक 16
प्रवतशत बढ़ा है । गुरुिार को पहली बार रुपया 80 से नीचे बंर्द हुआ है । जबकक स्र्ानीय मुरा ने
अतीत में तीन मौकों पर उस स्तर का उल्लंघन ककया र्ा - वपछले कर्दन के वनचले स्तर 80.13 पर -
यह मजबूत होकर बंर्द हुआ।
सौजन्द्य से: र्द वबजनेस स्टैंडडथ; वसतम्बर 23, 2022
उपरोक्त समाचार ररपोटथ के आलोक में, भारतीय आयात, वनयाथत और भुगतान संतुलन पर वस्र्वत के
प्रभाि पर चचाथ कीवजए ।
After suffering its steepest single-day fall since the Ukraine war broke out, the
rupee settled at 80.87 as against 79.98 at the previous close. The domestic
currency has depreciated 8 per cent so far in 2022. The dollar index, which rose to
fresh 20-year highs on Thursday, has gained 16 per cent so far in the calendar year.
Thursday is the first time that the rupee has ended lower than 80. While the local
currency had breached that level on three occasions in the past -- with the
previous intraday low at 80.13 -- it recovered to close stronger.
Source: The Business Standard; Sept. 23, 2022
In the light of the above news report, discuss the impact of the situation on Indian
Imports, Exports and Balance of Payments.

12. वनम्नवलवखत आँकड़ों से "स्टॉक में पररितथन" के मूल्य की गणना कीवजए : 3


Calculate the value of “Change in Stock” from the following data:
ि. सं. मर्दे रकम (₹ करोड़ों में)
S.no. Items Amount (₹ in Crores)
(i) वबिी 400
Sales
(ii) कारक मूल्य पर शुि मूल्य िृवि (NVAFC) 200
Net value added at factor cost (NVAFC)
(iii) अनुर्दान 10
Subsidies
(iv) स्टॉक में पररितथन ?
Change in stock
(v) मूल्यह्रास 40
Deprecation
(vi) मध्यिती उपभोग 100
Intermediate Consumption
अर्िा / OR
"र्दो-क्षेत्रीय अर्थव्यिस्र्ा में राष्ट्रीय उत्पार्द राष्ट्रीय आय के बराबर होता है"। रटप्पणी कीवजए ।
“In a two-sector economy national product is equal to national income”.
Comment.

13. "एक अर्थव्यिस्र्ा में, स्िायत्त खपत 100 है और सीमांत उपभोग प्रिृवत्त (MPC) 0.6 है। यकर्द आय 4
का संतल
ु न स्तर ₹ 2,000 है, तो स्िायत्त वनिेश ₹ 300 है।" िैि गणना द्वारा कर्न की पुवि कीवजए।
"In an economy, the autonomous consumption is 100 and Marginal
Propensity to Consume (MPC) is 0.6. If the equilibrium level of Income is
₹ 2,000, then the autonomous investment is ₹ 300." Justify the statement with
valid calculation.

14. "ककसी अर्थव्यिस्र्ा में, पूि-थ पूिथ समग्र आपूर्तथ, पूि-थ पूिथ समग्र मांग से अविक होती है"| उत्पार्दन, आय 4
और रोजगार के स्तर पर इसके संभावित प्रभाि का विस्तार से िणथन कीवजए ।
“In an economy, ex-ante Aggregate Supply is more than ex-ante Aggregate
Demand”. Elaborate the possible impact of the same on the level of output,
income and employment.
अर्िा / OR
"एक काल्पवनक अर्थव्यिस्र्ा में, वनयोवजत बचत योजनाबि वनिेश से कम हो जाती है, वजससे
रोजगार और आय में वगरािट आती है" | क्या आप कर्दए गए कर्न से सहमत हैं? िैि स्पिीकरण के
सार् अपने उत्तर का समर्थन कीवजए ।
“In a hypothetical economy, planned savings fall short of planned
investments, leading to fall in employment and income” Do you agree with the
given statement? Support your answer with a valid explanation.

15. 'ररिसथ रे पो रे ट और खुले बाज़ार की कियाएँ' ककसी अर्थव्यिस्र्ा में अवतररक्त िन आपूर्तथ को कै से 4
वनयंवत्रत करें ग?े
How will ‘Reverse Repo Rate and Open Market Operations’ control excess
money supply in an economy.

16. "ककसी र्देश के सरकारी बजट में राजस्ि घाटे के अवस्तत्ि के वबना राजकोषीय घाटा नहीं हो सकता |" 6
कर्दए गए कर्न का बचाि या खंडन कीवजए ।
“The Government Budget of a country cannot have fiscal deficit without the
existence of revenue deficit.” Defend or refute the given statement.
अर्िा / OR
"आय की असमानताओं को कम करने के वलए करािान एक प्रभािी उपकरण है |" कर्दए गए कर्न को
िैि कारणों के सार् उवचत ठहराइए ।
“Taxation is an effective tool to reduce the inequalities of income” Justify the
given statement with valid reasons.
17.(a) उत्पार्दन, आय और व्यय के प्रिाह को पररपत्र क्यों कहा जाता है? 3
Why the flows of production, income and expenditure is called Circular?

(b) "राष्ट्रीय आय में के िल घरे लू क्षेत्र के भीतर उत्पार्दन के कारकों द्वारा अर्जथत आय शावमल है |” िैि 3
कारणों के सार् कर्दए गए कर्न का बचाि या खंडन कीवजए ।
"National income includes income earned by factors of production, within
the domestic territory only.” Defend or refute the given statement
with valid reasons.
खण्ड-ब (भारतीय अर्थव्यिस्र्ा का विकास)
SECTION–B (Indian Economic Development)
18. वनम्नवलवखत कर्नों पर विचार कीवजए: 1
1. GST पररषर्द ने ऑनलाइन गेबमंग और कै सीनो के वलए पूणथ अंककत मूल्य पर एक समान 18
प्रवतशत कर लगाने का वनणथय वलया।
2. लॉटरी, सट्टेबाजी और जुए को कारथ िाई योग्य र्दािों के रूप में िगीकृ त ककया गया र्ा और उन्द्हें
कें रीय माल और सेिा कर अविवनयम, 2017 के तहत माल के रूप में पररभावषत ककया गया है।
उपरोक्त कर्नों में से कौन-सा / से सही है / हैं?
(a) के िल 1 (c) 1 और 2 र्दोनों
(b) के िल 2 (d) न तो 1 और न ही 2
Consider the following statements:
1. GST Council decided to levy a uniform 18 per cent tax on full face value for
online gaming and casinos.
2. Lottery, betting, and gambling were classified as actionable claims and they are
defined as goods under the Central Goods and Services Tax Act, 2017.
Which of the above statements given above is/are correct?
(a) 1 only (c) Both 1 and 2
(b) 2 only (d) Neither 1 nor 2

19. मुरास्फीवत के संर्दभथ में, वनम्नवलवखत कर्नों पर विचार कीवजए : 1


1. इसकी गणना र्ोक मूल्य सूचकांक का उपयोग करके की जाती है।
2. ईंिन और अन्द्य िस्तुओं की तुलना में खाद्य िस्तुएं उच्च मुरास्फीवत के वलए अविक वजम्मेर्दार हैं।
उपरोक्त कर्नों में से कौन-सा / से सही है / हैं?
(a) के िल 1 (c) 1 और 2 र्दोनों
(b) के िल 2 (d) न तो 1 और न ही 2
With reference to the headline inflation, consider the following statements:
1. It is calculated using the Wholesale Price Index.
2. Food items are more responsible in higher headline inflation than fuel and other
items.
Which of the statements given above is/are correct?
(a) 1 only (c) Both 1 and 2
(b) 2 only (d) Neither 1 nor 2

20. राष्ट्रीय कृ वष बाज़ार (e-NAM) के संर्दभथ में वनम्नवलवखत कर्नों पर विचार कीवजए : 1
1. यह एक अवखल भारतीय इलेक्रॉवनक रेबडंग पोटथल है जो कृ वष िस्तुओं के वलए एकीकृ त राष्ट्रीय
बाजार बनाने के वलए मौजूर्दा कृ वष उपज बाजार सवमवत (APMC) मंवडयों को नेटिकथ बनाता है।
2. यह आंवशक रूप से कें र सरकार द्वारा और आंवशक रूप से संबंवित राज्य सरकारों द्वारा वित्त
पोवषत है।
3. इसे लघु ककसान कृ वष व्यिसाय कं सोर्टथयम द्वारा कायाथवन्द्ित ककया जाता है।
उपरोक्त कर्नों में से ककतने कर्न सही हैं?
(a) के िल एक (c) सभी तीन
(b) के िल र्दो (d) कोई नहीं
With reference to the National Agricultural Market (e-NAM), consider the
following statements:
1. It is a pan-India electronic trading portal which networks the existing
Agricultural Produce Market Committee (APMC) mandis to create a unified
national market for agricultural commodities
2. It is funded partially by the Central Government and partially by the respective
State Governments.
3. It is implemented by Small Farmers Agribusiness Consortium.
How many of the statements given above are correct?
(a) Only one (c) All three
(b) Only two (d) None of these

21. चीन ने एक बच्चे की नीवत बंर्द कर र्दी क्योंकक: 1


(a) इससे आवित आबार्दी की संख्या में िृवि हुई।
(b) युिा लोगों के अनुपात में बुजुगथ लोग अविक होंगे।
(c) लोग नीवत से असंतुि हो गए।
(d) र्देश की जनसंख्या कम हो गई।
China discontinued the one child policy because:
(a) It increased the number of dependent populations.
(b) There will be more elderly people in proportion to young people.
(c) People became dissatisfied with the policy.
(d) Population of the country decreased.

22. वनम्नवलवखत अवभकर्न (A) और कारण (R) को ध्यानपूिथक पकढ़ए | 1


वनम्नवलवखत विकल्पों में से एक सही विकल्प का चयन कीवजए |
अवभकर्न (A): वपछले र्दो र्दशकों में, राष्ट्र साकथ , यूरोपीय संघ, आवसयान, जी -7, विक्स इत्याकर्द जैसे
क्षेत्रीय और िैवश्वक आर्र्थक समूह बन रहे हैं।
कारण (R): इससे उनकी अपनी घरे लू अर्थव्यिस्र्ाएं मजबूत होंगी।
विकल्प:
(a) अवभकर्न (A) और कारण (R) र्दोनों सत्य हैं और कारण (R) अवभकर्न (A) का सही स्पिीकरण
है।
(b) अवभकर्न (A) और कारण (R) र्दोनों सत्य हैं लेककन कारण (R) अवभकर्न (A) का सही
स्पिीकरण नहीं है।
(c) अवभकर्न (A) सत्य है लेककन कारण (R) गलत है।
(d) अवभकर्न (A) गलत है लेककन कारण (R) सच है।
Read the following Assertion (A) and Reason (R).
Choose the correct alternative.
Assertion (A): Over the last two decades or so, nations are forming regional and
global economic groupings such as the SAARC, European Union, ASEAN, G-7,
BRICS etc.
Reason (R): This will strengthen their own domestic economies.
Alternatives:
(a) Both Assertion (A) and Reason (R) are true and Reason (R) is the correct
explanation of Assertion (A).
(b) Both Assertion (A) and Reason (R) are true but Reason (R) is not the correct
explanation of Assertion (A).
(c) Assertion (A) is true but Reason (R) is false.
(d) Assertion (A) is false but Reason (R) is true.

23. वनम्नवलवखत कर्नों को ध्यानपूिथक पकढ़ए । 1


कर्न 1: भारत और पाककस्तान र्दोनों ने वबना ककसी बाहरी र्दबाि के अपने आर्र्थक सुिार शुरू
ककए।
कर्न 2: पाककस्तान ने SEZ नीवत को सफलतापूिक थ लागू ककया है और वनयाथत संििथन नीवत का
उपयोग करके इसके लाभ प्राप्त ककए हैं।
कर्दए गए कर्नों के आलोक में सही विकल्प का चयन कीवजए :
(a) कर्न 1 सत्य है और कर्न 2 गलत है |
(b) कर्न 1 गलत है और कर्न 2 सत्य है |
(c) कर्न 1 और 2 र्दोनों सत्य हैं |
(d) कर्न 1 और 2 र्दोनों गलत हैं |
Read the following statements carefully.
Statement 1: Both India and Pakistan initiated their economic reforms without
any external pressures.
Statement 2: Pakistan has successfully implemented the SEZ policy and reaped its
benefits using the Export Promotion policy.
In the light of the given statements, choose the correct alternative:
(a) Statement 1 is true and statement 2 is false
(b) Statement 1 is false and statement 2 is true
(c) Both statements 1 and 2 are true
(d) Both statements 1 and 2 are false

24. वनम्नवलवखत कर्नों को ध्यानपूिथक पकढ़ए | 1


कर्न I: 1960 के र्दशक में, तत्कालीन खाद्य और कृ वष मंत्री वचर्दम्बरम सुिमण्यम के नेतृत्ि में भारत
सरकार ने एक आनुिंवशकीविर्द्, एम.एस. स्िामीनार्न की मर्दर्द से हररत िांवत शुरू की।
कर्न II: हररत िांवत के र्दौरान भारत में बौने चािल के बीजों की उच्च उपज िाली ककस्में (HYVS)
अर्ाथत् लेमाथ रोजो-64-ए और सोनारा 64 पेश की गईं, वजन्द्हें जापान से आयात ककया गया र्ा।
उपरोक्त कर्नों के आलोक में नीचे कर्दए गए विकल्पों में से सबसे उपयुक्त उत्तर का चयन कीवजए -
(a) र्दोनों कर्न I और II सही हैं।
(b) र्दोनों कर्न I और II गलत हैं।
(c) कर्न I सही है, लेककन कर्न II गलत है |
(d) कर्न I गलत है, लेककन कर्न II सही है |
Read the following statements carefully.
Statements I: In 1960's, the Government of India under the then Food and
Agriculture Minister Chidambaram Subramaniam, launched the Green Revolution
with the help of a geneticist, M.S. Swaminathan.
Statement II: During Green revolution high-yielding varieties (HYVS) of dwarf rice
seeds namely Lerma Rojo-64-A and Sonara 64 was introduced in India, which
were imported from Japan.
In the light of the above statements, choose the most appropriate answer from the
options given below-
(a) Both statements I and II are correct.
(b) Both statements I and II are incorrect.
(c) Statement I is correct, but Statement II is incorrect
(d) Statement I is incorrect, but Statement II is correct

25. वनम्नवलवखत अवभकर्न (A) और कारण (R) को ध्यानपूिथक पकढ़ए | 1


वनम्नवलवखत विकल्पों में से एक सही विकल्प का चयन कीवजए |
अवभकर्न (A): गैर-आर्र्थक कारक आर्र्थक विकास की प्रकिया में महत्िपूणथ भूवमका वनभाते हैं।
कारण (R): पूंजी का संचय मानि पूंजी वनमाथण का भी रूप लेता है ।
विकल्प:
(a) A सत्य है लेककन R गलत है।
(b) A गलत है लेककन R सत्य है।
(c) A और R र्दोनों सत्य हैं और R, A की सही व्याख्या है।
(d) A और R र्दोनों सत्य हैं लेककन R, A की सही व्याख्या नहीं है।
Read the following Assertion (A) and Reason (R).
Choose the correct alternative.
Assertion (A): Non-economic factors play an important role in the process of
economic development.
Reason (R): Accumulation of capital takes the form of human capital formation
also.
Alternatives:
(a) A is true but R is false.
(b) A is false but R is true.
(c) Both A and R are true and R is the correct explanation of A.
(d) Both A and R are true but R is not the correct explanation of A.

26. वनम्नवलवखत अवभकर्न (A) और कारण (R) को ध्यानपूिथक पकढ़ए | 1


वनम्नवलवखत विकल्पों में से एक सही विकल्प का चयन कीवजए |
अवभकर्न (A): माइिो-िे वडट मवहलाओं को सशक्त बनाने और उन्द्हें वित्तीय रूप से स्ितंत्र बनाने में
मर्दर्द कर सकता है।
कारण (R): माइिो-िे वडट में उवचत ब्याज र्दरों पर प्रर्दान ककए गए छोटे ऋण शावमल हैं जो लोगों
को अपना उद्यम शुरू करने में मर्दर्द कर सकते हैं।
विकल्प:
(a) A सत्य है लेककन R गलत है।
(b) A गलत है लेककन R सत्य है।
(c) A और R र्दोनों सत्य हैं और R, A की सही व्याख्या है।
(d) A और R र्दोनों सत्य हैं लेककन R, A की सही व्याख्या नहीं है।
Read the following Assertion (A) and Reason (R).
Choose the correct alternative.
Assertion (A): Micro-credit can help empower women and make them financially
independent.
Reason (R): Micro-credit involves small loans provided at reasonable interest
rates that can help people start their own ventures.
Alternatives:
(a) A is true but R is false.
(b) A is false but R is true.
(c) Both A and R are true and R is the correct explanation of A.
(d) Both A and R are true but R is not the correct explanation of A.

27. वनम्नवलवखत कर्नों को ध्यानपूिथक पकढ़ए और सही विकल्प का चयन कीवजए : 1


कर्न-I: एक-र्दलीय शासन के तहत पीपुल्स ररपवब्लक ऑफ चाइना की स्र्ापना के बार्द, अर्थव्यिस्र्ा
के सभी महत्िपूणथ क्षेत्रों, उद्यमों और व्यवक्तयों के स्िावमत्ि और संचावलत भूवम को सरकारी वनयंत्रण
में लाया गया।
कर्न-II: ग्रेट लीप फॉरिडथ (GLF) अवभयान 1958 में पाककस्तान में शुरू ककया गया र्ा वजसका
उद्देश्य र्देश को बड़े पैमाने पर औद्योवगकीकरण करना र्ा ।
(a) र्दोनों कर्न गलत हैं।
(b) र्दोनों कर्न सत्य हैं।
(c) कर्न-I सत्य है, कर्न-II गलत है।
(d) कर्न-II सत्य है, कर्न-I गलत है।
Read the following statements carefully and choose the correct alternative:
Statement-I: After the establishment of People's Republic of China under one-
party rule, all critical sectors of the economy, enterprises and lands owned and
operated by individuals were brought under government control.
Statement-II: The Great Leap Forward (GLF) campaign initiated in 1958 in
Pakistan aimed at industrialising the country on a massive scale.
(a) Both the statements are false.
(b) Both the statements are true.
(c) Statement-I is true, Statement-II is false.
(d) Statement-II is true, Statement-I is false

28. ‘'आत्मवनभथर भारत' योजना प्रकिया के उद्देश्य के रूप में 'आत्मवनभथरता' के रूप में भारतीय वनयोजन 3
प्रकिया की जड़ों में रहा है। क्या आप कर्दए गए कर्न से सहमत हैं? कर्दए गए कर्न के औवचत्य की
पुवि कीवजए ।
‘Atamnirbhar Bharat' had been at the roots of the Indian planning process in the
form of 'self-reliance' as an objective of the planning process. Do you agree with
the given statement? Justify the rationale of the given statement.
अर्िा / OR
विरटश काल में स्िर्देशी भारतीय उद्योगों को व्यिवस्र्त रूप से अवस्र्र करने के र्दो उद्देश्य बताइए।
State the two-fold motive for the systematic destabilisation of indigenous Indian
industries in the British era.

29. "मानि पूज


ं ी वनमाथण निाचार, आविष्कार और तकनीकी सुिार को जन्द्म र्देता है।" क्या आप कर्दए 3
गए कर्न से सहमत हैं? िैि तकों के सार् अपने उत्तर का समर्थन कीवजए ।
"Human Capital Formation Gives birth to innovation, invention and
technological improvements." Do you agree with the given statement? Support
your answer with valid arguments.

30. 1958 में शुरू ककए गए चीन के ग्रेट लीप फॉरिडथ अवभयान की व्याख्या कीवजए । सार् ही, GLF 4
अवभयान के सामने आने िाली समस्याओं का भी उल्लेख कीवजए ।
Explain the Great Leap Forward campaign of China as initiated in 1958. Also, state
the problems which GLF campaign met with.

31. ग्रामीण विकास क्या है? भारतीय आर्र्थक विकास के वलए ग्रामीण विकास क्यों आिश्यक है? ग्रामीण 4
विकास के प्रमुख मुद्दों पर प्रकाश डावलए ।
What is Rural Development? Why is rural development essential for Indian
economic development? Bring out the key issues in rural development.

32. कोविड के बार्द की र्दुवनया में हम एक नया विश्व लोकाचार र्देखने जा रहे हैं। व्यिस्र्ा और भारत 4
अंतरराष्ट्रीय अर्थव्यिस्र्ाओं के पुनरुिार और बहाली में बढ़ती भूवमका वनभाएगा; सरकार
रणनीवतक रूप से िम-गहन विवनमाथण को बढ़ािा र्दे रही है। क्या िम प्रिान विवनमाथण भारत में
रोजगार को बढ़ािा र्देने में मर्दर्द कर सकता है?
In the post-Covid world we are going to see a new world ethos. Order and India
will play an increasing role in the revival and restoration of international
economies; Government is strategically promoting labour-intensive
manufacturing. Can the labour-intensive manufacturing help in promoting
employment in India?

33. वनम्नवलवखत पररच्छेर्द को साििानीपूिथक पकढ़ए :


नॉमथन बोरलॉग भले ही हररत िांवत के जनक रहे हों, लेककन भारत में इसके िास्तुकार वनस्संर्दह े
मोनकोम्ब संबावशिन स्िामीनार्न र्े। महान कृ वष िैज्ञावनक, वजनका 7 अगस्त कर्दन गुरुिार को 98
िषथ की आयु में वनिन हो गया, 1955 में जब उन्द्होंने वहतोशी से सुना र्ा तब िे मुवश्कल से 30 िषथ
के र्े। जापान के जाने-माने गेहं आनुिंवशकीविर्द् ककहारा, नोररन-10 के बारे में बताते हैं, जो उस र्देश
के इिाते प्रीफे क्चर में एक प्रायोवगक स्टेशन पर पैर्दा की गई एक अिथ-बौनी ककस्म है।
1960 के र्दशक की शुरुआत में, भारत का गेहं और चािल का उत्पार्दन िमशः
10-12 वमवलयन टन (एमटी) और 35-36 वमवलयन टन पर र्ा, वजससे बड़े पैमाने पर अनाज
आयात हुआ जो 1966-67 में 10 वमवलयन टन को पार कर गया। 2013-14 में, घरे लू गेहं उत्पार्दन
95.85 वमवलयन टन, जबकक चािल के वलए 106.65 वमवलयन टन होने का अनुमान लगाया गया
र्ा। यह सच है कक वजन लोगों ने गेहं में ब्लॉकबस्टर ककस्मों (कल्याण सोना, सोनावलका, अजुथन,
जनक) का िास्तविक प्रजनन या चयन ककया र्ा। एचडी-2285 और एचडी-2329) और चािल
(आईआर-8, जया और पद्मा) जो ककसानों ने बड़े पैमाने पर बोए र्े, िे लोकवप्रय कल्पना में उतने
प्रवसि नहीं हैं - जैसे िी.एस. मार्ुर, एस.पी. कोहली, डी.एस. अठिाल और, बेशक महान जी.एस.
ख़ुश। लेककन इसमें कोई संर्दह े नहीं है कक भारत में हररत िांवत को रे खांककत करने िाली बुवनयार्दी
रणनीवतक र्दृवि - उिथरक और पानी के अनुप्रयोग में िृवि के वलए उत्तरर्दायी एक नए आनुिंवशक
तनाि या 'पौिे के प्रकार' की शुरुआत - स्िामीनार्न से आई र्ी।
पारं पररक गेहं और चािल की ककस्में लंबी और पतली र्ीं। जब िे बड़े हो गए और उच्च उिथरक खुराक
के जिाब में पैर्दा हुए अच्छी तरह से भरे अनाज से उनके बाल भारी हो गए, तो िे जमीन पर वगर
गए।
1954 में, कै वम्िज विश्वविद्यालय से पीएचडी करने और विस्कॉवन्द्सन विश्वविद्यालय में पोस्ट-
डॉक्टोरल ररसचथ एसोवसएटवशप करने के बार्द कटक में कें रीय चािल अनुसंिान संस्र्ान में रहते हुए,
स्िामीनार्न ने अपेक्षाकृ त गैर-आिास और उिथरक-उत्तरर्दायी से जीन स्र्ानांतररत करने के वलए एक
कायथिम पर काम ककया। 'जैपोवनका' चािल की ककस्मों से लेकर स्िर्देशी 'इं वडका' प्रजावत तक। उस
िषथ के अंत में नई कर्दल्ली में भारतीय कृ वष अनुसि ं ान संस्र्ान (आई.ए.आर.आई.) में शावमल होने के
बार्द उन्द्होंने उन्नत उिथरक प्रवतकिया के वलए प्रजनन के इस र्दृविकोण को गेहं तक बढ़ाया।
स्िामीनार्न ने अवनिायथ रूप से पौिों की ऊंचाई में कमी की मांग की, वजससे इसे आिास-प्रिण कम
ककया जा सके । उत्पररितथन का उपयोग करके अिथ-बौनी गेहं की ककस्मों को विकवसत करने की
उनकी रणनीवत - पौिों को उनके डीएनए में िांछनीय संशोिन लाने के वलए रसायनों या विककरण
के संपकथ में लाना - हालांकक, काम नहीं आई: पौिों की ऊंचाई कम होने से अनाज के आकार में एक
सार् कमी आई- पुष्पगुच्छ या कणथफूल िारण करना!
सौजन्द्य से: र्द इं वडयन एक्सप्रेस; वसतम्बर 29, 2023
वनम्नवलवखत प्रश्नों के उत्तर उपरोक्त पररच्छेर्द एिं सामान्द्य समझ के आिार पर र्दीवजए :
(i) मोनकोम्ब साम्बवशिन स्िामीनार्न कौन र्े? 2
(ii) हररत िांवत के उद्देश्य क्या र्े? 2
(iii) हररत िांवत का भारत की अर्थव्यिस्र्ा पर क्या प्रभाि पड़ा? 2

Read the following passage carefully:


Norman Borlaug may have been the Father of the Green Revolution, but its
architect in India was undoubtedly Monkomb Sambasivan Swaminathan. The
legendary agricultural scientist, who passed away on Thursday after turning 98 on
August 7, was hardly 30 in 1955 when he heard from Hitoshi Kihara, the well-
known wheat geneticist from Japan, about Norin-10, a semi-dwarf variety bred at
an experimental station in that country’s Iwate Prefecture.
In the early 1960s, India’s wheat and rice production were
languishing at 10-12 million tonnes (mt.) and 35-36 mt. respectively, forcing
massive grain imports that crossed 10 mt. in 1966-67. In 2013-14, domestic wheat
output was estimated at 95.85 mt. while at 106.65 mt. for rice. It is true that the
people who did the actual breeding or selection of the blockbuster varieties in
wheat (Kalyan Sona, Sonalika, Arjun, Janak, HD-2285 and HD-2329) and rice (IR-8,
Jaya and Padma) that farmers planted in a big way aren’t as well known in popular
imagination — the likes of VS Mathur, SP Kohli, DS Athwal and, of course, the
legendary G.S. Khush. But there isn’t any doubt that the basic strategic vision
underpinning the Green Revolution in India — introducing a new genetic strain or
‘plant type’ responsive to increased fertiliser and water application — came from
Swaminathan.
The traditional wheat and rice cultivars were tall and slender. These ‘lodged’ – fell
flat on the ground — when they grew and their ear heads were heavy with well-
filled grains produced in response to high fertiliser doses.
In 1954, while at the Central Rice Research Institute at Cuttack after doing a PhD
from Cambridge University and a post-doctoral research associateship at the
University of Wisconsin, Swaminathan worked on a programme for transferring
genes from the relatively non-lodging and fertiliser-responsive ‘Japonica’ rice
varieties to indigenous ‘Indica’ races. This approach of breeding for enhanced
fertiliser response he extended to wheat after joining the Indian Agriculture
Research Institute (IARI) at New Delhi later that year. Swaminathan essentially
sought a reduction in plant height making it less lodging-prone. His strategy of
developing semi-dwarf wheat varieties using mutagenesis — exposing plants to
chemicals or radiation to introduce desirable modifications in their DNA — did
not, however, work: The lowering of plant heights led to a simultaneous reduction
in the size of the grain-bearing panicles or ear heads!
Source: The Indian Express; Sept. 29, 2023
On the basis of the given text and common understanding, answer the following
question:
(i) Write the answers of following question on the basis of above paragraph: -
(ii) Who was the Monkomb Sambasivan Swaminathan?
(iii) What were the objectives of Green Revolution?

34. सतत िारणीय विकास से आप क्या समझते हैं? ितथमान समय में भारत सतत िारणीय विकास के 6
लक्ष्यों को प्राप्त करने के वलए पयथटन का उपयोग ककस प्रकार से कर रहा है?
What do you understand by sustainable development? Recently how India is using
tourism to achieve sustainable development goals?
के न्द्रीय विद्यालय संगठन, जयपुर सम्भाग SET-2
Kendriya Vidyalaya Sangthan, Jaipur Region
विषय: अर्थशास्त्र
Subject: Economics
अभ्यास पत्र
Practice Paper
कक्षा: XII समय: 3: 00 घंटे अविकतम अंक: 80 अंक
Class: XII Time: 3:00 Hours Maximum Marks: 80 Marks
सामान्द्य वनर्देश:
(1) इस प्रश्न पत्र के र्दो खण्ड है:-
खण्ड-अ (पररचयात्मक समवि अर्थशास्त्र) खण्ड-ब (भारतीय अर्थव्यिस्र्ा का विकास)
(2) र्दोनों खण्डों के सभी प्रश्न करना अवनिायथ है | यद्यवप कु छ प्रश्नों में आन्द्तररक चयन प्रर्दान ककया गया है | प्रत्येक प्रश्न
के अंक उस प्रश्न के सामने अंककत है|
(3) प्रश्न संख्या1 से 10 एिं 18 से 27 तक के सभी एक-एक अंक के अवत लघूत्तरात्मक बहुविकल्पीय / प्रश्न है | इन प्रश्नों
के उत्तर एक शब्र्द या एक िाक्य में र्दीवजए |
(4) प्रश्न संख्या 11 से 12 एिं 28 से 29 तक के सभी प्रश्न लघूत्तरात्मक प्रश्न है, जो कक तीन-तीन अंक के है | इन प्रश्नों के
उत्तर 60- 80शब्र्दों से अविक नहीं होने चावहए|
(5) प्रश्न संख्या 13 से 15 एिं 30 से 32 तक के सभी प्रश्न भी लघूत्तरात्मक प्रश्न है, जो कक चार चार-अंक के है | इन प्रश्नों
के उत्तर80 - 100शब्र्दों से अविक नहीं होने चावहए|
(6) प्रश्न संख्या 16 से 17 एिं 33 से 34 तक के सभी प्रश्न र्दीघथ-उत्तरात्मक प्रश्न है, जो कक छह छह-अंक के है | इन प्रश्नों
के उत्तर100 - 150शब्र्दों से अविक नहीं होने चावहए |
(7) सभी प्रश्नों के उत्तर यर्ा सम्भि प्रर्दत्त शब्र्द सीमा को ध्यान में रखते हुए संवक्षप्त तथ्यात्मक ,एिं सटीक होने चावहए |
(8) इस प्रश्न पत्र को पढ़ने हेतु पंरह (15) वमनट का अवतररक्त समय आिंरटत ककया गया है |

General Instructions:
(1) This question paper contains two sections:
Section–A (Introductory Macro Economics) Section–B (Indian Economic Development)
(2) All questions in both the sections are compulsory. However, there is internal choice in some
questions. Marks for questions are indicated against each question.
(3) Question number 1 –10 and 18 – 27 are very short-answer / multiple choice questions carrying
1 mark each. They are required to be answered in one word or one sentence each.
(4) Question number 11 – 12 and 28 – 29 are short-answer questions carrying 3 marks each.
Answers to them should normally not exceed 60-80 words each.
(5) Question number 13 – 15 and 30 – 32 are also short-answer questions carrying 4 marks each.
Answers to them should normally not exceed 80-100 words each.
(6) Question number 16 – 17 and 33 – 34 are long-answer questions carrying 6 marks each.
Answers to them should normally not exceed 100-150 words each.
(7) Question number 16 – 17 and 33 – 34 are long-answer questions carrying 6 marks each.
Answers to them should normally not exceed 100-150 words each.
(8) An additional 15 minutes has been allotted to read the question paper.
प्र. सं. खण्ड-अ (पररचयात्मक समवि अर्थशास्त्र) अंक
Q. NO. Section-A (Introductory Macro Economics) Marks
1 वनम्नवलवखत कर्नों को ध्यानपूिथक पकढ़ए : 1
कर्न 1: लाभ- अजथन की वस्र्वत तब उत्पन्न होती है जब औसत उपभोग प्रिृवत्त का मूल्य एक के
बराबर हो।
कर्न 2: स्िायत्त उपभोग के कारण बचत िि मूल बबंर्दु के नीचे ऋणात्मक Y- अक्षांश से शुरू होता
है।
कर्दए गए कर्नों के आलोक मे, वनम्नवलवखत में से सही विकल्प का चयन कीवजए :
(a) कर्न 1 सत्य है और कर्न 2 गलत है |
(b) कर्न 1 गलत है और कर्न 2 सत्य है |
(c) कर्न 1 और 2 र्दोनों सत्य है |
(d) र्दोनों कर्न 1 और 2 गलत है |
Read the following statements carefully:
Statement 1: Break-even point occurs when the value of APC=1.
Statement 2: The saving curve starts below the origin point on the negative Y-
axis because of autonomous consumption.
In the light of the given statements, choose the correct alternative from the
following:
(a) Statement 1 is true and statement 2 is false.
(b) Statement 1 is false and statement 2 is true.
(c) Both statements 1 and 2 are true.
(d) Both statements 1 and 2 are false.
2 " मैं इसॆ रुपए में वगरािट के रूप में नहीं बवल्क डॉलर की मजबूती के रूप में र्देखती हं"। IMF और 1
विश्व बैंक के िार्षथक सम्मेलन के पिात वनमथला सीतारमण ने कहा। विर्देशी मुरा डॉलर के मजबूत
होने के कारण भारत के आयात और वनयाथत पर सबसे संभावित प्रभाि की पहचान कीवजए:
(a) आयात महंगा होगा (c) वनयाथत सस्ता होगा
(b) आयात सस्ता होगा (d) र्दोनों (a) और (c)
“First of all I would look at it as not rupee sliding, I would look at it as dollar
strengthening, dollar strengthening incessantly” said Nirmala Sitaraman after
attending the Annual meetings of the IMF and the world Bank.
Identify the most likely impact on India’s imports and exports due to
strengthening of foreign currency dollar:
(a) Imports become costlier (c) Exports become cheaper
(b) Imports become cheaper (d) Both (a) and (c )
3 मुरा आपूर्तथ से तात्पयथ ककसी अर्थव्यिस्र्ा में समय के _________ विवभन्न रूपों में जनता द्वारा 1
िाररत िन की कु ल मात्रा से हैं।
(a) एक अिवि में (c) अल्प अिवि
(b) एक बबंर्द ु पर (d) इनमें से कोई नहीं
Money supply refers to the total quantity of money held by the public in various
forms ____________________ of time in an economy.
(a) Over a period (c) Short period
(b) At point of time (d) None of these
4 वनम्नवलवखत कर्नों को ध्यानपूिथक पकढ़ए : 1
कर्न 1: बीओपी खाते में संतुलन बनाए रखने के वलए व्यापार की िस्तुओं को समायोवजत ककया
जाता है।
कर्न 2: संतुवलत बीओपी शेष विश्व के सार् आर्र्थक संबंिों को इं वगत करता है।
कर्दए गए कर्नों के आलोक मे, वनम्नवलवखत में से सही विकल्प का चयन कीवजए :
(a) कर्न 1 सत्य है और कर्न 2 गलत है |
(b) कर्न 1 गलत है और कर्न 2 सत्य है |
(c) कर्न 1 और 2 र्दोनों सत्य है |
(d) र्दोनों कर्न 1 और 2 गलत है |
Read the following statements carefully:
Statement 1: Accommodating items of trade are undertaken in order to maintain
the balance in the BOP account.
Statement 2: Balance BOP indicates stable economic relation with the rest of the
world.
In the light of the given statements, choose the correct alternative:
(a) Statement 1 is true and statement 2 is false.
(b) Statement 1 is false and statement 2 is true.
(c) Both statements 1 and 2 are true.
(d) Both statements 1 and 2 are false.
5 वनम्नवलवखत कर्नों को ध्यानपूिथक पकढ़ए : 1
कर्न 1: आय का प्रिाह वद्व-क्षेत्रीय अर्थव्यिस्र्ा में िृत्ताकार होता है।
कर्न 2: बचत स्टॉक अििारणा है।
कर्दए गए कर्नों के आलोक मे, वनम्नवलवखत में से सही विकल्प का चयन कीवजए :
(a) कर्न 1 सत्य है और कर्न 2 गलत है |
(b) कर्न 1 गलत है और कर्न 2 सत्य है |
(c) कर्न 1 और 2 र्दोनों सत्य है |
(d) र्दोनों कर्न 1 और 2 गलत है |
Read the following statements carefully:
Statement 1: Flow of income is circular in a two-sector economy.
Statement 2: Savings are stock concepts.
In the light of the given statements, choose the correct alternative:
(a) Statement 1 is true and statement 2 is false.
(b) Statement 1 is false and statement 2 is true.
(c) Both statements 1 and 2 are true.
(d) Both statements 1 and 2 are false.
6 एक अर्थव्यिस्र्ा में, वनिेश व्यय में 1,000 की िृवि के पररणाम स्िरुप आय में 10,000 की िृवि 1
होती है। उपभोग की सीमांत प्रिृवत्त की गणना करें ।
In an economy, income increases by 10,000 as a result of a rise in investment
expenditure by 1,000. Marginal Propensity to consume will be:
(a) 0.2 (b) 0.5 (c) 0.9 (d) 0.8
7 अर्थव्यिस्र्ा में वनिेश को प्रोत्सावहत करने के वलए, कें रीय बैंक ______सकता है। 1
(सही विकल्प का चयन कीवजए)
(a) नकर्द आरवक्षत अनुपात बढ़ा
(b) बैंक र्दर बढ़ा
(c) नकर्द आरवक्षत अनुपात कम करके
(d) सरकारी प्रवतभूवतयों को खुले बाजार में बेचना
In order to encourage investment in the economy, the central bank may________.
( Choose the correct alternative)
(a) Increase cash reserve ratio
(b) Increase bank rate
(c) Reduce cash reserve ratio
(d) Sell government securities in open market
अर्िा / OR
कानूनी आरवक्षत अनुपात की गणना कीवजए यकर्द ₹ 200 करोड़ की प्रारं वभक जमा रावश ₹ 1600
करोड़ की कु ल जमा रावश का वनमाथण करती है।
The legal reserve requirement if initial deposit of ₹ 200 crores lead to creation of
total deposits of ₹ 1600 crores will be:
(a) 10% (b) 12% (c) 12.5% (d) 10.5%
8 कॉलम A कॉलम B के सही सुमेवलत युगम को पहचावनए: 1
कॉलम A कॉलम B

1. उत्पार्द शुल्क (A) पूंजीगत प्रावप्तयां

2. आयकर (B) प्रत्यक्ष कर

3. पीएसयू से कमाई (C) अप्रत्यक्ष कर

4. िृिािस्र्ा पेंशन (D) गैर-कर राजस्ि प्रावप्तयां


(a) 1-(A) (b) 2-(B) (c) 3-(C) (d) 4-(D)
Identify the correctly matched pair I column A to that of column B:
Column A Column B
1. Excise duty (A) Capital receipts

2. Income tax (B) Direct tax


3. Earning from PSUs (C) Indirect tax

4. Old age pensions (D) Non-tax revenue receipts


(a) 1-(A) (b) 2-(B) (c) 3-(C) (d) 4-(D)
9 वनम्नवलवखत अवभकर्न (A) एिं कारण (R) को पकढ़ए । 1
वनम्नवलवखत विकल्पों में से एक सही विकल्प का चयन कीवजए :
अवभकर्न (A): मंर्दी के र्दौरान राज्य सरकारों द्वारा कर िृवि अक्सर कें र सरकार द्वारा
राजकोषीय नीवत के विस्तार िार्दी प्रभाि को कम करती है।
कारण (R): कर िृवि से प्रयोज्य आय में कमी आती है वजसके कारण अर्थव्यिस्र्ा में उपभोग से वगर
जाता है।
विकल्प:
(a) अवभकर्न (A) और कारण (R) र्दोनों सत्य है और कारण (R) अवभकर्न (A) की सही व्याख्या है।
(b) कर्न (A) और कारण (R) र्दोनों सत्य है और कारण (R) अवभकर्न (A) की सही व्याख्या नहीं है
(c) अवभकर्न (A) सही है लेककन कारण (R) गलत है
(d) अवभकर्न (A) गलत है लेककन कारण(R) सत्य है
Read the following statement- Assertion (A) and Reason (R).
Choose one of the correct alternatives given below:
Assertion (A): Tax increases by state governments during recessions often reduce
the expansionary impact of fiscal policy by the central government.
Reason (R): Tax increases lead to decrease in disposable income due to which
consumption expenditure in the economy falls.
Alternatives:
(a) Both Assertion (A) and Reason (R) are true and Reason (R) is the correct
explanation of Assertion (A).
(b) Both Assertion (A) and Reason (R) are true and Reason (R) is not the correct
explanation of Assertion (A).
(c) Assertion (A) is true but Reason (R) is false.
(d) Assertion (A) is false but Reason (R) is true.
10 वनम्नवलवखत अवभकर्न (A) एिं कारण (R) को पकढ़ए । 1
वनम्नवलवखत विकल्पों में से एक सही विकल्प का चयन कीवजए :
अवभकर्न (A) : उिारों की अर्दायगी एक पूंजीगत व्यय है।
कारण (R) : पूंजीगत व्यय सरकार का र्दावयत्ि पर जाता है।
विकल्प:
(a) अवभकर्न (A) और कारण (R) र्दोनों सत्य है और कारण (R) अवभकर्न (A) की सही व्याख्या है।
(b) कर्न (A) और कारण (R) र्दोनों सत्य है और कारण (R) अवभकर्न (A) की सही व्याख्या नहीं है
(c) अवभकर्न (A) सही है लेककन कारण (R) गलत है
(d) अवभकर्न (A) गलत है लेककन कारण (R) सत्य है
Read the following statement- Assertion (A) and Reason (R).
Choose one of the correct alternatives given below:
Assertion (A): Repayment of borrowings is a capital expenditure.
Reason (R): Capital expenditure increases the liability of the government.
Alternatives:
(a) Both Assertion (A) and Reason (R) are true and Reason (R) is the correct
explanation of Assertion (A).
(b) Both Assertion (A) and Reason (R) are true and Reason (R) is not the correct
explanation of Assertion (A).
(c) Assertion (A) is true but Reason (R) is false.
(d) Assertion (A) is false but Reason (R) is true.
11 (a) ककसी र्देश के कल्याण के सूचकांक के रूप में सकल घरे लू उत्पार्द का उपयोग करने की सीमा के रूप 3
में उर्दाहरण सवहत “बाहरी कारकों” की व्याख्या कीवजए ।
Explain ‘Externalities’ With example As a limitation of using gross domestic
product as an index of welfare of a country.
अर्िा / OR
(b) 'उत्पार्दकों को प्रर्दान ककए जाने िाला अनुर्दान हस्तांतरण भुगतान के रूप में माना जाना चावहए।'
कर्दए गए कर्न का उवचत कारण र्देकर बचाि या खंडन कीवजए ।
‘Subsidies to the producers should be treated as transfer payments’. Defend or
refute the given statement with valid reason.
12 एक अर्थव्यिस्र्ा का बजट फलन इस प्रकार कर्दया गया है: S= -25+0.25Y. 3
यकर्द वनयोवजत वनिेश ₹ 200 करोड़ है तो वनम्नवलवखत की गणना कीवजए:
The saving function of an economy is given as: S= -25+0.25Y.
If the planned investment is ₹ 200 crores, calculate the following:
(a) अर्थव्यिस्र्ा में आय का संतुलन स्तर
Equilibrium level of income in the economy
(b) ₹ 500 करोड़ की आय पर कु ल मांग
Aggregate demand at income of ₹ 500 crores.
13 सरकारी बजट के उद्देश्य को स्पि कीवजए: 4
Elaborate the objective of government budget:
(a) संसािनों का आिंटन
Allocation of resources
(b) आर्र्थक वस्र्रता
Economic stability
14 िावणवज्यक बैंक साख का वनमाथण कै से करते हैं? संख्यात्मक उर्दाहरण की सहायता से स्पि कीवजए 4

How do commercial banks create deposits? Explain with the help of a numerical
example.
15 (a) बताइए कक वनम्नवलवखत कर्न सत्य है या असत्य। अपने उत्तर का कारण स्पि कीवजए । 4
State whether the following statements are true or false. Give reasons for your
answer.
(i) मुरा का अवभमूल्यन और अिमूल्यन एक ही बात है |
Devaluation and depreciation of currency are one and the same thing.
(ii) िस्तुओं एिं सेिाओं के वनयाथत और आयात मूल्य के मध्य के अंतर को ही व्यापार में संतुलन कहा
जाता है |
Differences between value of exports and imports of goods and services are called
balance of trade.
अर्िा / OR
(b) स्िायत्त मर्दों और समायोवजत मर्दों के मध्य अंतर स्पि कीवजए ।
Distinguish between Autonomous items and Accommodating items.
16 (a) राजकोषीय नीवत क्या है? अर्थव्यिस्र्ा में अपस्फीवतक अंतराल को वनयंवत्रत करने के वलए कें र 3+3
सरकार अपनी राजकोषीय नीवत के उपाय का उपयोग कै से करती है।
What is fiscal policy? How does the central government use his fiscal policy
measure to control the deflationary gap in an economy?
(b) 'भारत में मुरास्फीवत र्दर को वनयंवत्रत करने के वलए 30 वसतंबर 2022 को आरबीआई की मौकरक
नीवत सवमवत ने नीवतगत रे पो र्दर में 50 आिार अंकों की िृवि की'। बताइए कक रे पो रे ट बढ़ने से
भारत में महंगाई कम करने में कै से मर्दर्द वमलेगी।
‘To control the inflation rate in India the monetary policy committee (MPC) of
the RBI on 30 September, 2022 increased policy repo rate by 50 basis points’.
Explain how increased repo rate will help to decrease the inflation in India.
17 राष्ट्रीय आय की गणना कीवजए : 3+3
Calculate National Income:
ि.सं. वििरण ₹ करोड़ में
Sr.no Particulars ₹ in crores

(1) विर्देशों से कारक आय 15


Factor income from abroad

(2) वनजी अंवतम उपभोग व्यय 600


Private final consumption expenditure

(3) अचल पूंजी की खपत 50


Consumption of fixed capital

(4) सरकार का अंवतम उपभोग व्यय 200


Government final consumption expenditure

(5) स्टॉक में पररितथन (-) 10


Change in stock
(6) शुि घरे लू अचल पूंजी वनमाथण 110
Net domestic fixed capital formation
(7) विर्देशों में शुि कारक आय 10
Net factor income to abroad
(8) शुि आयात (-) 20
Net imports
(9) शुि अप्रत्यक्ष कर 70
Net indirect tax
(b) मध्यिती िस्तुओं और अंवतम िस्तुओं के मध्य क्या अंतर है?
What is the difference between intermediate good and final good?
खण्ड-ब (भारतीय अर्थव्यिस्र्ा का विकास)
SECTION–B (Indian Economic Development)
18 स्ितंत्रता की पूिथ संध्या पर र्देश के सकल घरे लू उत्पार्द में प्रमुख योगर्दानकताथ के सार्-सार् सबसे 1
बड़ा वनयोक्ता कौन सा क्षेत्र र्ा?
(a) कृ वष (c) सेिाएं
(b) उद्योग (d) उत्पार्दन
Which sector was the major contributor to GDP of the country as well as the
biggest employer on the eve of independence?
(a) Agricultural (c) Services
(b) Industry (d) Manufacturing
19 िषथ 1921 को 'महान विभाजन का िषथ' माना जाता है,_____ को इं वगत करता है, जो र्दूसरे चरण 1
की शुरुआत को इं वगत करता है।
(a) उच्च मृत्यु र्दर एिं उच्च जन्द्म र्दर
(b) उच्च मृत्यु र्दर एिं वनम्न जन्द्म र्दर
(c) वनम्न मृत्यु र्दर एिं उच्च जन्द्म र्दर
(d) वनम्न मृत्यु र्दर एिं वनम्न जन्द्म र्दर
The year 1921 is regarded as the ‘year of great divide’ indicates ________, which
indicates the onset of the second stage.
(a) High mortality rate and high birth rate.
(b) High mortality rate and low birth rate
(c) Low mortality rate and high birth rate
(d) Low mortality rate and low birth rate
अर्िा / OR
वनम्नवलवखत घटनाओं के कालानुिवमक िम को र्दशाथने िाले सही विकल्प का चयन कीवजए :
(I) महान विभाजन का िषथ
(II) बंगाल के महान अकाल वजसने एक बड़े स्तर पर जीिन को खत्म कर कर्दया
(III) मुंबई से ठाणे तक चलने िाली प्रर्म रेन
(IV) स्िेज नहर का उद्घाटन
विकल्प:
(a) (IV),(II),(I),(III) (c) (II),(III),(IV),(I)
(b) (I),(IV), (III),(II) (d) (III),(I),(IV),(II)
Choose the correct alternative showing chronological order of the following
events:
(I) The year of great divide
(II) Great famines of Bengal which claimed a large toll of lives
(III) First train run from Bombay to Thane
(IV) opening of Suez Canal
Alternatives:
(a) (IV),(II),(I),(III) (c) (II),(III),(IV),(I)
(b) (I),(IV), (III),(II) (d) (III),(I),(IV),(II)
20 वनम्नवलवखत में से कौन-सा मानि पूंजी वनमाथण का एक महत्िपूणथ वहस्सा नहीं है? 1
(a) जीिन प्रत्याशा में िृवि (c) उत्पार्दन की लागत बढ़ाएं
(b) भौवतक पूंजी का प्रभािी उपयोग (d) र्दृविकोण का आिुवनकीकरण
Which of the following is not an important part of human capital formation?
(a) Increase in life expectancy (c) Increase cost of production
(b) Effective use of physical capital (d) Modernisation of attitude
21 ककसानो द्वारा संकट वबिी से तात्पयथ है: 1
(a) सरकार द्वारा तय न्द्यूनतम समर्थन मूल्य पर फसल की वबिी
(b) बहुत कम कीमत पर फसल की वबिी
(c) मध्यस्र्ों के माध्यम से फसल की वबिी
(d) सड़ी फसल की वबिी
Distress sale by the farmers refers to:
(a) Sale of crop at the MSP fixed by the government
(b) Sale of crop at a very poor price
(c) Sale of crop through commissions agents
(d) Sale of rotten crop
अर्िा / OR
प्रच्छन्न बेरोजगारी का अर्थ है:
(a) वबना नौकरी िाले लोग
(b) नौकरी के वलए वनयोवजत अविक व्यवक्त जो कम पूरा कर सकते र्े
(c) जो लोग काम करने को तैयार नहीं है
(d) मवहलाओं में बेरोजगारी
Disguised unemployment refers to:
(a) Persons with no jobs
(b) More persons employed for a job which less can accomplish
(c) People who are not willing to work
(d) Unemployment among women
22 वनम्नवलवखत कर्नों को पकढ़ए - 1
कर्न 1: प्रार्वमक क्षेत्र भारत में अविकांश िवमकों के वलए रोजगार का मुख्य स्त्रोत है।
कर्न 2: प्रार्वमक क्षेत्र में मवहला िवमकों की सघनता बहुत कम है।
कर्दए गए कर्नों के आलोक मे, वनम्नवलवखत में से सही विकल्प का चयन कीवजए :
(a) कर्न 1 सत्य है और कर्न 2 गलत है |
(b) कर्न 1 गलत है और कर्न 2 सत्य है |
(c) कर्न 1 और 2 र्दोनों सत्य है |
(d) र्दोनों कर्न 1 और 2 गलत है |
Read the following statements carefully:
Statement 1: A Primary sector is the main source of employment for the majority
of workers in India.
Statement 2: Women workers concentration is very low in the primary sector.
In the light of the given statements, choose the correct alternative:
(a) Statement 1 is true and statement 2 is false.
(b) Statement 1 is false and statement 2 is true.
(c) Both statements 1 and 2 are true.
(d) Both statements 1 and 2 are false.
23 सौर ऊजाथ और पिन ऊजाथ,उजाथ के ______ स्त्रोत है। (निीकरणीय / गैर-निीकरणीय) 1
Solar energy and wind energy are ____________________ sources of energy.
(Renewable / Non-renewable)
24 सतत विकास प्राप्त ककया जा सकता है: 1
(a) जनसंख्या िृवि वनयंत्रण द्वारा
(b) प्रर्दूषण को वनयंवत्रत करके
(c) निीकरणीय संसािनों के उपयोग को प्रवतबंवित करके
(d) उपरोक्त सभी
Sustainable development can be achieved by:
(a) controlling the growth of population
(b) Controlling pollution
(c) Restricting use of renewable resources
(d) All of these
25 कर्न के सही युग्म का चयन कीवजए: 1
कॉलम I कॉलम II
A. ग्रेट लीप फॉरिाडथ I. सामूवहक रूप से कृ वष करना

B. कम्यून वसस्टम II. उद्योगों को अपने घरों में खोलना

C. सिथहारा संस्कृ वतक िांवत III. छात्रों को काम करने और ग्रामीण इलाकों से
सीखने के वलए भेजा गया र्ा

D. चीन में आर्र्थक सुिार IV. 1988


विकल्प:
(a) A-I (b) B-II (c) C-III (d) D-IV
Choose the correct pair of the statement:
Column I Column II

A. Great leap forward I. Cultivating land collectively

B. Commune system II. Opening of the industries in their homes


C. Proletarian cultural revolution III. Students were sent to work and learn
from the countryside
D. Economic reforms in china IV. 1988
Alternatives:
(a) A-I (b) B-II (c) C-III (d) D-IV
26 वनम्नवलवखत अवभकर्न (A) एिं कारण (R) को पकढ़ए । 1
वनम्नवलवखत विकल्पों में से एक सही विकल्प का चयन कीवजए :
अवभकर्न (A): भारत में छोटे और सीमांत िारकों के वलए ग्रामीण ऋण के संस्र्ागत स्त्रोत कम
सुलभ है।
कारण (R): बैंकों को संपार्श्वथक प्रर्दान ककया जाना चावहए जो छोटे और सीमांत ककसान िाहन करने
में सक्षम नहीं है।
विकल्प:
(a) अवभकर्न (A) और कारण (R) र्दोनों सत्य है और कारण (R) अवभकर्न (A) की सही व्याख्या है।
(b) कर्न (A) और कारण (R) र्दोनों सत्य है और कारण (R) अवभकर्न (A) की सही व्याख्या नहीं है
(c) अवभकर्न (A) सही है लेककन कारण (R) गलत है
(d) अवभकर्न (A) गलत है लेककन कारण (R) सत्य है
Read the following statement- Assertion (A) and Reason (R).
Choose one of the correct alternatives given below:
Assertion (A): Institutional sources of rural credit are less accessible to small and
marginal holders in India.
Reason (R): Collaterals have to be provided to the banks which the small and
marginal farmers are not able to afford.
Alternatives:
(a) Both Assertion (A) and Reason (R) are true and Reason (R) is the correct
explanation of Assertion (A).
(b) Both Assertion (A) and Reason (R) are true and Reason (R) is not the correct
explanation of Assertion (A).
(c) Assertion (A) is true but Reason (R) is false.
(d) Assertion (A) is false but Reason (R) is true.
27 वनम्नवलवखत अवभकर्न (A) एिं कारण (R) को पकढ़ए । 1
वनम्नवलवखत विकल्पों में से एक सही विकल्प का चयन कीवजए :
अवभकर्न (A): इनकी सुिार प्रकिया में र्दोहरी मूल्य वनिाथरण नीवत शुरू की गई र्ी।
कारण (R): विर्देशी वनिेशकों को आकर्षथत करने के वलए विशेष आर्र्थक क्षेत्रों की स्र्ापना की गई।
विकल्प:
(a) अवभकर्न (A) और कारण (R) र्दोनों सत्य है और कारण (R) अवभकर्न (A) की सही व्याख्या है।
(b) कर्न (A) और कारण (R) र्दोनों सत्य है और कारण (R) अवभकर्न (A) की सही व्याख्या नहीं है
(c) अवभकर्न (A) सही है लेककन कारण (R) गलत है
(d) अवभकर्न (A) गलत है लेककन कारण (R) सत्य है
Read the following statement- Assertion (A) and Reason (R).
Choose one of the correct alternatives given below:
Assertion (A): Dual pricing policy was initiated in the reforms process of China.
Reason (R): In order to attract foreign investors, special economic zones were set
up.
Alternatives:
(a) Both Assertion (A) and Reason (R) are true and Reason (R) is the correct
explanation of Assertion (A).
(b) Both Assertion (A) and Reason (R) are true and Reason (R) is not the correct
explanation of Assertion (A).
(c) Assertion (A) is true but Reason (R) is false.
(d) Assertion (A) is false but Reason (R) is true.
28 िणथन कीवजए कक नकारात्मक पयाथिरणीय प्रभाि की अिसर लागत ककस प्रकार से अविक है। 3
Explain how the opportunity costs of negative environmental impact are high.
29 "सकल घरे लू उत्पार्द में िृवि और रोजगार के बीच की अंतराल चौड़ी होती जा रही है।" उस 3
प्रिृवत्त का उल्लेख कीवजए जो इस पररघटना को उजागर करती है।
”The gap between the growth of GDP and employment is widening”. State the
trend which highlights this phenomenon.
अर्िा / OR
वशक्षा को ककसी राष्ट्र के विकास में एक महत्िपूणथ आगत माना जाता है। क्यों?
Education is considered to be an important input for the development of a nation.
How?
30 "आर्र्थक सुिार प्रकिया से कृ वष क्षेत्र र्दुष्प्रभावित हुआ प्रतीत होता है।" कर्दए गए कर्न की व्याख्या 4
कीवजए।
“Agriculture sector appears to be adversely affected by the economic reform
process”. Explain the given statement.
31 विरटश औपवनिेवशक प्रशासन द्वारा अपनाई गई औद्योवगक नीवत की कु छ कवमयों का 4
समालोचनात्मक मूल्यांकन कीवजए ।
Critically appraise some of the shortfalls of the industrial policy pursued by the
British colonial administration.
अर्िा / OR
जबकक अनुर्दान ककसानों को नई तकनीक का उपयोग करने के वलए प्रोत्सावहत करती है, िे
सरकारी वित्त पर भारी बोझ है इस तथ्य के आलोक में सवब्सडी की उपयोवगता की वििेचना
कीवजए।
While subsidies encourage farmers to use new technology, they are a huge
burden on government finances. Discuss the usefulness of subsidies in the light of
this fact.
32 "प्रिानमंत्री ने गैर-कृ वष गवतविवियों को बढ़ाकर ग्रामीण आय बढ़ाने का आग्रह ककया है"। चचाथ 4
करें कक गैर-कृ वष गवतविवियों से ग्रामीण क्षेत्रों में लोगों की आय में िृवि कै से हो सकती है?
”The prime minister urged to increase the rural income by increasing non-farm
activities”. Discuss how non-farm activities can lead to rise in income of people in
the rural sector.
33 (A) 1958 में प्रारं भ की गई चीन के ग्रेट लीप फॉरिडथ अवभयान की व्याख्या कीवजए। 3+3
Explain the Great Leap campaign of China as initiated in 1958.
(B) स्ितंत्रता संकेतक की पररभाषा र्दीवजए। स्ितंत्रता संकेतकों के र्दो उर्दाहरण र्दीवजए |
Define the liberty indicator. Give two examples of Liberty indicators.
34 वनम्नवलवखत पररच्छेर्द को ध्यानपूिथक पकढ़ए :
जुलाई वसतंबर 2020 वतमाही के र्दौरान भारत में मवहला िम भागीर्दारी र्दर वगरकर 16.1% हो
गई, जो प्रमुख अर्थव्यिस्र्ाओं में सबसे कम है। एक सरकारी ररपोटथ में कहा गया है, जो महामारी
के प्रभाि और व्यापक नौकरी संकट को र्दशाथता है। सोमिार र्देर रात सांवख्यकी मंत्रालय द्वारा
जारी ररपोटथ में कहा गया कक जब भारत में कोविड-19 िायरस के प्रसार को रोकने के वलए कड़ा
लोकडाउन लागू ककया र्ा तब िम बल में मवहलाओं का प्रवतशत अप्रैल-जून 2020 की वतमाही के
र्दौरान 15.5% के ररकॉडथ वनचले स्तर पर आ गया र्ा | विश्व बैंक के अनुमानों के अनुसार भारत में
र्दुवनया की सबसे कम मवहला िम शवक्त भागीर्दारी है। एक वतहाई से भी कम मवहलाएं, वजन्द्हें
ररपोटथ में 15 या उससे अविक उम्र के रूप में पररभावषत ककया गया है- काम कर रही है या सकिय
रूप से नौकरी की तलाश कर रही है।
भारत में अविकांश वनयोवजत मवहलाएं कम कौशल कायों में कायथरत है जैसे कक खेत और कारखाने
के िम और घरे लू मर्दर्द, ऐसे क्षेत्र महामारी से बुरी तरह प्रभावित हुए हैं। वसतंबर 2020 में समाप्त
हुए 3 महीनों के र्दौरान पुरुष िवमकों के बीच 12.6% की तुलना में मवहलाओं में बेरोजगारी की र्दर
15.8% तक पहुंच गई, वजसके वलए निीनतम वतमाही वनर्र्दथि जारी ककया गया र्ा।
पररच्छेर्द को पढ़ने के उपरान्द्त वनम्न प्रश्नों के उत्तर पररच्छेर्द के अनुसार अपनी सामान्द्य समझ के
आिार पर र्दीवजए :
(i) कायथ में मवहलाओं की कम भागीर्दारी के विवभन्न कारणों पर चचाथ कीवजए । 3
(ii) र्देश में रोजगार की वस्र्वत का विश्लेषण करने के वलए िवमक जनसंख्या अनुपात एक संकेतक कै से हैं? 3
Read the following text carefully:
Female labour participation rate in India fell to 16.1% during the July- September
2020 quarter, the lowest among the major economies, a government report said,
reflecting the impact of pandemic and a widening job crisis. The percentage of
women in the labour force had fallen to a record of 15.5% during the April June
2020 quarter, when India imposed strict lockdown to curb the spread of covid-19
virus, according to the report released late Monday by the Ministry of statistics.
According to the World Bank estimates, India has one of the lowest female labour
force participation rates in the world. Less than one third of women- defined in
report as 15 or older- are working or actively looking for a job.
Most employed women in India are in low skilled work, such as farm and factory
labour and domestic help, sectors that have been hit hard by the pandemic. The
unemployment rate among women touched 15.8%, compared with 12.6% among
male workers during three months that ended in September 2020, the latest
quarter for which data was released.
After reading the text answer the given questions on the basis of your common
understanding:
(i) Discuss the various reasons for lower women participation in work.
(ii) How is the worker-population ratio an indicator for analyzing the employment
situation in the country?
के न्द्रीय विद्यालय संगठन, जयपुर सम्भाग SET-3
Kendriya Vidyalaya Sangthan, Jaipur Region
विषय: अर्थशास्त्र
Subject: Economics
अभ्यास पत्र
Practice Paper
कक्षा: XII समय: 300: घंटे अविकतम अंक: 80 अंक
Class: XII Time: 3:00 Hours Maximum Marks: 80
Marks सामान्द्य वनर्देश:
(1) इस प्रश्न पत्र के र्दो खण्ड है:-
खण्ड-अ (पररचयात्मक समवि अर्थशास्त्र) खण्ड-ब (भारतीय अर्थव्यिस्र्ा का विकास)
(2) र्दोनों खण्डों के सभी प्रश्न करना अवनिायथ है | यद्यवप कु छ प्रश्नों में आन्द्तररक चयन प्रर्दान ककया गया है | प्रत्येक प्रश्न
के अंक उस प्रश्न के सामने अंककत है|
(3) प्रश्न संख्या1 से 10 एिं 18 से 27 तक के सभी एक-एक अंक के अवत लघूत्तरात्मक बहुविकल्पीय / प्रश्न है | इन प्रश्नों
के उत्तर एक शब्र्द या एक िाक्य में र्दीवजए |
(4) प्रश्न संख्या 11 से 12 एिं 28 से 29 तक के सभी प्रश्न लघूत्तरात्मक प्रश्न है, जो कक तीन-तीन अंक के है | इन प्रश्नों के
उत्तर 60- 80शब्र्दों से अविक नहीं होने चावहए|
(5) प्रश्न संख्या 13 से 15 एिं 30 से 32 तक के सभी प्रश्न भी लघूत्तरात्मक प्रश्न है, जो कक चार चार-अंक के है | इन प्रश्नों
के उत्तर80 - 100शब्र्दों से अविक नहीं होने चावहए|
(6) प्रश्न संख्या 16 से 17 एिं 33 से 34 तक के सभी प्रश्न र्दीघथ-उत्तरात्मक प्रश्न है, जो कक छह छह-अंक के है | इन प्रश्नों
के उत्तर100 - 150शब्र्दों से अविक नहीं होने चावहए |
(7) सभी प्रश्नों के उत्तर यर्ा सम्भि प्रर्दत्त शब्र्द सीमा को ध्यान में रखते हुए संवक्षप्त तथ्यात्मक ,एिं सटीक होने चावहए |
(8) इस प्रश्न पत्र को पढ़ने हेतु पंरह (15) वमनट का अवतररक्त समय आिंरटत ककया गया है |

General Instructions:
(1) This question paper contains two sections:
Section–A (Introductory Macro Economics) Section–B (Indian Economic Development)
(2) All questions in both the sections are compulsory. However, there is internal choice in some
questions. Marks for questions are indicated against each question.
(3) Question number 1 –10 and 18 – 27 are very short-answer / multiple choice questions carrying
1 mark each. They are required to be answered in one word or one sentence each.
(4) Question number 11 – 12 and 28 – 29 are short-answer questions carrying 3 marks each.
Answers to them should normally not exceed 60-80 words each.
(5) Question number 13 – 15 and 30 – 32 are also short-answer questions carrying 4 marks each.
Answers to them should normally not exceed 80-100 words each.
(6) Question number 16 – 17 and 33 – 34 are long-answer questions carrying 6 marks each.
Answers to them should normally not exceed 100-150 words each.
(7) Question number 16 – 17 and 33 – 34 are long-answer questions carrying 6 marks each.
Answers to them should normally not exceed 100-150 words each.
(8) An additional 15 minutes has been allotted to read the question paper.
प्र. सं. खण्ड-अ (पररचयात्मक समवि अर्थशास्त्र) अंक
Q. NO. Section-A (Introductory Macro Economics) Marks
Q.1 वनम्नवलवखत कर्न-I एिं कर्न-II को साििानीपूिथक पकढ़ए और वनम्नवलवखत विकल्पों में से सही 1
विकल्प का चयन कीवजए-
कर्न I- राजस्ि बजट सरकार की पररसंपवत्त-उत्तरर्दावयत्ि को प्रभावित करता है ।
कर्न II- उच्च पूंजीगत प्रावप्तयां अक्सर उिार लेने की अवनिायथताओं से संबंवित होती हैं।
विकल्प:
(A) कर्न-I सत्य है एिं कर्न-II असत्य है ।
(B) कर्न-I असत्य है एिं कर्न-II सत्य है ।
(C) र्दोनों कर्न सत्य है |
(D) र्दोनों कर्न असत्य है |
Read the following statement-I and statement-II carefully and choose the correct
alternatives from the followings-
Statement-I: Revenue budget impact asset-liability status of the government.
Statement-II: High capital receipts are often related to compulsions of borrowings.
Alternatives:
(A) Statement-I is true and Statement-II is false.
(B) Statement-II is true and Statement-I is false.
(C) Both Statement are true.
(D) Both Statement are false.
Q.2 वनम्नवलवखत अवभकर्न (A) और कारण (R) को ध्यानपूिथक पकढ़ए | 1
वनम्नवलवखत विकल्पों में से एक सही विकल्प का चयन कीवजए |
अवभकर्न (A): वनिेश गुणक का मूल्य एक और अनन्द्त के बीच पररितथनीय है।
कारण (R): जब वनिेश गुणक का मूल्य एक होगा तो सीमान्द्त उपभोग प्रिृवत का भी मूल्य एक होगा।
विकल्प:
(A) अवभकर्न (A) एिं कारण (R) र्दोनों सत्य हैं और कारण (R),अवभकर्न (A) की सही व्याख्या है।
(B) अवभकर्न (A) एिं कारण (R) र्दोनों सत्य हैं और कारण (R), अवभकर्न (A) की सही व्याख्या
नहीं है।
(C) अवभकर्न (A) सत्य है लेककन कारण (R) असत्य है।
(D) अवभकर्न (A) असत्य है एिं कारण (R) सत्य है।
Read the following Assertion (A) and Reason (R).
Choose the correct alternative.
Assertion (A): Value of investment multiplier varies between one and infinity.
Reason (R): When investment multiplier is one, value of MPC is also one.
Alternatives:
(A) Both Assertion (A) and Reason (R) are true and reason (R) is the correct
explanation of Assertion(A).
(B) Both Assertion (A) and Reason (R) are true and reason (R) is not the correct
explanation of Assertion (A).
(C) Assertion (A) is true but Reason (R) is false.
(D) Assertion (A) is false but Reason (R) is true.
Q.3 कॉलम-II में कर्दए गए विकल्पों का कॉलम-I में संबंवित मर्द के सार् वमलान करके सही अनुिम की 1
पहचान कीवजए ।
Identify the correct sequence of the alternatives given in column-II by matching
them with respective items in column-I.
कॉलम-I कॉलम-II
Column-I Column-II
a. ऋण i. सरकारी व्यय > सरकारी प्रावप्तयां
Borrowings Government Expenditure > Government Receipt
b. विवनिेश ii. पूंजीगत प्रावप्त
Disinvestment Capital Receipt
c. पूंजीगत व्यय iii. राज्य सरकारों को ऋण आिंटन
Capital expenditure Loan granted to state government
d. बजट घाटा iv. सरकार द्वारा ककये गये वनिेश की वनकासी
Deficit budget Withdrawal of existing investment by government
(A) a-i, b-ii, c-iii, d-iv (C) a-ii, b-iv, c-iii, d-i
(B) a-ii, b-i, c-vi, d-iii (D) a-iii, b-iv, c-ii, d-ii
Q.4 राम एिं श्याम के मध्य वनम्नवलवखत संिार्द को पकढ़ए और वनम्नवलवखत प्रश्न का उत्तर र्दीवजए- 1
राम - मुझे 1 ककलो चािल चावहए।
श्याम - तुम विवनमय / बर्दले में क्या र्दोगे?
राम - मैं तुम्हे चािल के बर्दले में कु छ गेहं र्दूग
ं ा।
श्याम - मुझे चािल की आिश्यकता नहीं है। मुझे कु छ र्दाल चावहए।
राम एिं श्याम को विवनमय प्रकिया की कौन-सी समस्या का सामना करना पड़ रहा है?
(A) भािी भुगतान के आिार का अभाि (C) मूल्य के भण्डारण का अभाि
(B) र्दोहरी आिश्यकताओं के संयोग का अभाि (D) उपरोक्त में कोई नहीं
Read the following dialogue between Ram and Shyam and answer the following
question -
Ram- I want 1kg of rice.
Shyam- What will you give in exchange?
Ram- I can give you some wheat in return for rice.
Shyam- I don’t need rice. I want some pulse.
Which of the following problems is being faced by Ram and Shyam in their
exchange process?
(A) Lack of standard of deferred payment (C) Lack of store of value
(B) Lack of double coincidence of wants (D) None of these
Q.5 भारत में वस्र्त एक कं पनी को विर्देश में वस्र्त एक कं पनी से ऋण प्राप्त होता है। यह लेन-र्देन भारत के 1
भुगतान खाते के संतुलन में ककस प्रकार से र्दजथ ककया जाता है? (सही विकल्प का चयन कीवजए)
(A) चालू खाते का िे वडट पक्ष में (C) पूंजी खाते का िे वडट पक्ष में
(B) चालू खाते का डेवबट पक्ष में (D) पूंजी खाते का डेवबट पक्ष में
A company located in India receives a loan from a company located abroad. How is
this transaction recorded in India’s balance of payments account?
(Choose the correct alternative)
(A) Credit side of current account (C) Credit side of capital account
(B) Debit side of current account (D) Credit side of capital account
अर्िा / OR
भुगतान शेष अवभलेखों का चालू खाता र्दजथ करता है: (सही विकल्प का चयन कीवजए)
(A) िस्तुओं का वनयाथत और आयात (C) ितथमान अंतरण
(B) वनयाथत और सेिाओं का आयात। (D) ये सभी
Current account of BOP records: (Choose the correct alternative)
(A) Exports and imports of goods (C) Current transfer
(B) Exports and imports of services. (D) All of these
Q.6 वनम्नवलवखत अवभकर्न (A) और कारण (R) को ध्यानपूिथक पकढ़ए | 1
वनम्नवलवखत विकल्पों में से एक सही विकल्प का चयन कीवजए |
अवभकर्न (A): सािथजावनक ऋणों पर ककए गया व्याज भुगतान राजस्ि व्यय है।
कारण (R): राजस्ि व्यय या तो सरकार के वलए पररसंपवत्त का वनमाथण करता है या कफर सरकार के
उत्तरर्दावयत्िों में कटौती करता है।
विकल्प:
(A) अवभकर्न (A) एिं कारण (R) र्दोनों सत्य हैं और कारण (R),अवभकर्न (A) की सही व्याख्या है।
(B) अवभकर्न (A) एिं कारण (R) र्दोनों सत्य हैं और कारण (R), अवभकर्न (A) की सही व्याख्या
नहीं है।
(C) अवभकर्न (A) सत्य है लेककन कारण (R) असत्य है।
(D) अवभकर्न (A) असत्य है एिं कारण (R) सत्य है।
Read the following Assertion (A) and Reason (R).
Choose the correct alternative.
Assertion (A): Expenditure on interest payment on public loans is a revenue
expenditure.
Reason (R): Revenue expenditure either creates assets or causes a reduction in
liabilities for the government.
Alternatives:
(A) Both Assertion (A) and Reason (R) are true and reason (R) is the correct
explanation of Assertion (A).
(B) Both Assertion (A) and Reason (R) are true and reason (R) is not the correct
explanation of Assertion (A).
(C) Assertion (A) is true but Reason (R) is false.
(D) Assertion (A) is false but Reason (R) is true.
Q.7 वनम्नवलवखत कर्न-I एिं कर्न-II को साििानीपूिथक पकढ़ए और वनम्नवलवखत विकल्पों में से सही 1
विकल्प का चयन कीवजए -
कर्न I- लचीली विवनमय र्दर सरकार द्वारा वनिाथररत की जाती है।
कर्न II- प्रबंवित लचीली विवनमय र्दर, विवनमय र्दर को िांवछत सीमाओं के भीतर रखने का एक
प्रयास है ।
विकल्प:
(A) कर्न-I सत्य है एिं कर्न-II असत्य है ।
(B) कर्न-I असत्य है एिं कर्न-II सत्य है ।
(C) र्दोनों कर्न सत्य है |
(D) र्दोनों कर्न असत्य है |
Read the following statement-I and statement-II carefully and choose the correct
alternatives from the followings-
Statement-I: Flexible exchange rate is determinates by the government.
Statement-II: Managed floating exchange rate is an attempt to keep the exchange
rate within the desired limits.
Alternatives:
(A) Statement-I is true and Statement-II is false.
(B) Statement-II is true and Statement-I is false.
(C) Both Statement are true.
(D) Both Statement are false.
Q.8 वनम्नवलवखत अवभकर्न (A) और कारण (R) को ध्यानपूिथक पकढ़ए | 1
वनम्नवलवखत विकल्पों में से एक सही विकल्प का चयन कीवजए |
अवभकर्न (A): र्देश में मुरा की आपूर्तथकताथ द्वारा रखा गया मुरा का स्टॉक मुरा की आपूर्तथ का एक
वहस्सा है।
कारण (R): भारत में, "भारतीय ररज़िथ बैंक" मुरा का प्रमुख आपूर्तथकताथ है।
विकल्प:
(A) अवभकर्न (A) एिं कारण (R) र्दोनों सत्य हैं और कारण (R),अवभकर्न (A) की सही व्याख्या है।
(B) अवभकर्न (A) एिं कारण (R) र्दोनों सत्य हैं और कारण (R), अवभकर्न (A) की सही व्याख्या
नहीं है।
(C) अवभकर्न (A) सत्य है लेककन कारण (R) असत्य है।
(D) अवभकर्न (A) असत्य है एिं कारण (R) सत्य है।
Read the following Assertion (A) and Reason (R).
Choose the correct alternative.
Assertion (A): Stock of money is held by the supplier of money is a part of the
money supply in the country.
Reason (R): In India, “Reserve bank of India” is the principal supplier of money.
Alternatives:
(A) Both Assertion (A) and Reason (R) are true and reason (R) is the correct
explanation of Assertion (A).
(B) Both Assertion (A) and Reason (R) are true and reason (R) is not the correct
explanation of Assertion (A).
(C) Assertion (A) is true but Reason (R) is false.
(D) Assertion (A) is false but Reason (R) is true.
Q.9 कॉलम- I और कॉलम- II में कर्दए गए कर्नों के समूह में से, कर्नों के सही जोड़े का चयन कीवजए- 1
From the set of statements given in Column-I and Column-II, Choose the correct
pair of statements-
कालम –I कालम-II
Column-I Column-II
a. अल्परोजगार संतुलन i. अर्थव्यिस्र्ा में कोई बेरोजगारी नहीं।
Underemployment equilibrium No unemployment in the economy.
b. ऐवच्छक बेरोजगारी ii. अर्थव्यिस्र्ा में कोई अवतररक्त क्षमता नहीं है।
Involuntary unemployment No excess capacity in the economy.
c. बेरोजगारी की समस्या iii. ऐवच्छक बेरोजगारी की समस्या।
Problem of unemployment Problem of involuntary unemployment
d. पूणथ रोजगार iv. शून्द्य बेरोजगारी की वस्र्वत।
Full Employment A situation of zero unemployment.
(A) a-I (B) b-ii (C) c-iii (D) d-iv
Q.10 यकर्द मौकरक सकल घरे लू उत्पार्द 240 हो और सकल घरे लू उत्पार्द अपस्फीवतक अन्द्तराल का मूल्य 1
120 तो िास्तविक सकल घरे लू उत्पार्द के मूल्य की गणना कीवजए।
If nominal GDP is 240 and the value of GDP deflator is 120, find the value of real GDP.
(A) 0.5 (B) 2 (C) 20 (D) 288
अर्िा / OR
एक मशीन का बाजार मूल्य 2020 में खरीर्द के समय ₹ 500000 र्ा। 2023 में इसका बाजार मूल्य
₹ 350000 होने का अनुमान है। आपके अनुसार मशीन के बाजार मूल्य में वगरािट का कारण क्या
है?
(A) सामान्द्य टूट-फू ट (C) (A) एिं (B) र्दोनों
(B) आकवस्मक क्षवत (D) उपरोक्त में कोई नहीं
Market value of machine at the time of purchase is ₹ 500000 in 2020. In 2023 its
market value is estimated to be ₹ 350000. What do you think is the reason for the
fall in the market value of machine?
(A) Normal wear and tear (C) (A)and (B) Both
(B) Accidental damages (D) None of the Above.
Q.11 वनम्नवलवखत गवतविवियों को मध्यिती एिं अवन्द्तम िस्तुओं में िगीकृ त कीवजए । उवचत कारण 3
बताते हुए अपने उत्तर को वसि कीवजए।
Classify the following activities into intermediate goods or final goods. Justify your
answer with valid reason.
(A) स्टेशनरी र्दुकानर्दार द्वारा पुस्तकों की खरीर्दर्दारी।
Purchase of books by a stationary shopkeeper.
(B) सुरक्षा उपायों हेतु अस्पताल द्वारा सैवनटाजर एिं मास्क की खरीर्दर्दारी।
Sanitizer and Mask purchase by a hospital for precautionary measures.
(C) वमल्क के क बनाने के वलए वमठाई की र्दुकान द्वारा र्दूि खरीर्दना।
Milk purchase by sweet shop to make milk cake.
अर्िा / OR
वनम्नवलवखत की व्याख्या कीवजए-
Explain the following :
(i) संचलन आविक्य
Operating Surplus.
(ii) सामान्द्य वनिासी
Normal Resident.
(iii) घरे लू सीमा
Domestic Territory.
Q.12 “बजट नीवत के माध्यम से सरकार र्देश में आिश्यकता के अनुसार संसािनों का आिंटन करती है”। 3
क्या आप इस कर्न से सहमत है? उवचत कारण बताते हुए अपने उत्तर को वसि कीवजए।
“Through the budgetary policy government allocates resources in
accordance with requirement of the country.” Do you agree with the given
statement? Justify your answer with valid reason.
Q.13 उपभोग िि की सहायता से बचत िि व्युत्पन्न ककया जा सकता है अर्िा नहीं। एक काल्पवनक वचत्र 4
की सहायता से अपने उत्तर को वसि कीवजए।
Saving curve can be derived from consumption curve or not. Justify your answer
with a hypothetical diagram.
अर्िा / OR
यकर्द एक अर्थव्यिस्र्ा संतुलन में हो तो, वनम्नवलवखत जानकारी से वनिेश व्यय की गणना कीवजए।
Calculate investment expenditure from the following data about an economy
which is in equilibrium.
(i) राष्ट्रीय आय 10000
National Income
(ii) सीमान्द्त बचत प्रिृवत 0.25
MPS
(ii) स्ितंत्र उपभोग 2000
Autonomous Consumption
Q.14 (A) भुगतान संतुलन के स्िायत्त एिं समायोवजत मर्दों के मध्य अन्द्तर स्पि कीवजए। 2+2
Distinguish between autonomous and accommodating items of balance of
payments.
(B) प्रबंवित वितरण विवनमय र्दर क्या है?
What is the managed floating exchange rate system?
Q.15 के न्द्रीय बैंक के ककन्द्हीं र्दो कायों की व्याख्या कीवजए। 2+2
Explain any two functions of central bank.
Q.16 सकल घरे लू उत्पार्द की गणना करने की तीनो विवियों पर एक संवक्षप्त लेख वलवखए। तीनो विवियों से 4+2
पररकवलत सकल घरे लू उत्पार्द का मूल्य तीनो विवियों से एकसमान क्यों प्राप्त होता है, इसकी संवक्षप्त
व्यख्या भी कीवजए।
Write a short note on three methods of calculating the GDP of a country. Also
explain in brief why each of these methods should give the same value of GDP?
अर्िा / OR
उपलब्ि जानकारी का प्रयोग करके वनम्नवलवखत की गणना कीवजए :
Calculate the followings by using given data-
(A) विर्देशों से प्राप्त वनिल सािन आय
Net Factor Income from Abroad (NFIA).
(B) राष्ट्रीय आय
National Income.
ि. वििरण िनरावश (करोड़ में)
Sr. Particulars Amount (in Crore)
1 सािन लागत पर सकल राष्ट्रीय उत्पार्द 6150
National Product at Factor Cost
2 वनिल वनयाथत (-)50
Net Export
3 कमचाररयों को क्षवतपूर्तथ 1000
Compensation of Employees
4 सकल वस्र्र पूँजी वनमाथण 850
Gross fixed capital formation
5 स्टॉक में पररितथन 50
Change in stock
6 लाभांश 300
Dividend
7 वनिल अप्रत्यक्ष कर 300
Net Indirect Tax
8 ककराया 800
Rent
9 ब्याज 900
Interest
10 विर्देशों को सािन आय 80
Factor Income To Abroad
11 लाभ 1300
Profit
12 वनिल घरे लू पूँजी वनमाथण 800
Net Domestic Capital Formation
Q.17(A) नीचे कर्दए गए आंकड़ों के आिार पर वनम्नवलवखत प्रश्नों के उत्तर र्दीवजए: 3+3
वनिेश का वनयोवजत स्तर = 200 करोड़ रुपये
C = 100 + 0.8 Y
(i) आय का संतुलन स्तर का वनिाथरण कीवजए।
(ii) आय के संतुलन के स्तर पर बचत और उपभोग व्यय की गणना कीवजए।
(A) Answer the following questions based on the data given below :
Planned level of investment = Rs.200 crores
C = 100 + 0·8 Y
(i) Determine the equilibrium level of income.
(ii) Calculate the saving and consumption expenditure at equilibrium level of income.
(B) अिस्फीवतक अंतराल से क्या अवभप्राय है? अिस्फीवतक अंतराल की वस्र्वत को ठीक करने के वलए
ककन्द्हीं र्दो राजकोषीय उपायों का िणथन कीवजए।
What is meant by deflationary gap? State any two fiscal measures to correct the
situation of deflationary gap.
खण्ड-ब (भारतीय अर्थव्यिस्र्ा का विकास)
SECTION–B (Indian Economic Development)
Q.18 वनम्नवलवखत में से कौन-सा भारत में आर्र्थक वनयोजन के संबंि में सही नहीं है ? 1
(A) वमवित अर्थव्यिस्र्ा भारत में वनयोजन की रूपरे खा रही है।
(B) भारत में योजना का उद्देश्य समानता रहा है।
(C) भारत में योजना की शुरुआत सािथजवनक क्षेत्र पर भारी वनभथरता के सार् हुई र्ी।
(D) जिाहरलाल नेहरू को भारत में वनयोजन की िास्तुकला के रूप में जाना जाता है।
Which of the following is not correct about the economic planning in India?
(A) Mixed economy has been framework of the planning in India.
(B) Equity has been an objective of planning in India.
(C) Planning in India was started with a heavy dependency on public sector.
(D) Jawaharlal Nehru is known as architecture of planning in India.
Q.19 वनम्नवलवखत अवभकर्न (A) और कारण (R) को ध्यानपूिथक पकढ़ए | 1
वनम्नवलवखत विकल्पों में से एक सही विकल्प का चयन कीवजए |
अवभकर्न (A): विरटश शासन के र्दौरान भारतीय उद्योगों का व्यिवस्र्त रूप से विऔद्योगीकरण
ककया गया।
कारण (R): विरटशों द्वारा भारतीय अर्थव्यिस्र्ा शोषण करने के वलए इसका उपयोग कच्चे माल के
स्रोत और तैयार माल के बाजार के रूप में ककया जाता र्ा |
विकल्प:
(A) अवभकर्न (A) एिं कारण (R) र्दोनों सत्य हैं और कारण (R),अवभकर्न (A) की सही व्याख्या है।
(B) अवभकर्न (A) एिं कारण (R) र्दोनों सत्य हैं और कारण (R), अवभकर्न (A) की सही व्याख्या
नहीं है।
(C) अवभकर्न (A) सत्य है लेककन कारण (R) असत्य है।
(D) अवभकर्न (A) असत्य है एिं कारण (R) सत्य है।
Read the following Assertion (A) and Reason (R).
Choose the correct alternative.
Assertion (A)-There was a systematic deindustrialization of Indian industries
during British rule.
Reason (R) – Indian economy was used as a source of raw material and market
for finished goods in order to exploit by Britishers.
Alternatives:
(A) Both Assertion (A) and Reason (R) are true and reason (R) is the correct
explanation of Assertion (A).
(B) Both Assertion (A) and Reason (R) are true and reason (R) is not the correct
explanation of Assertion (A).
(C) Assertion (A) is false but Reason (R) is true.
(D) Assertion (A) is true but Reason (R) is false.
अर्िा / OR
विरटश शासन के र्दौरान वनम्नवलवखत में से कौन-सा कायथ नहीं ककया गया र्ा?
(A) रे लिे की स्र्ापना (C) तार रवहत प्रणाली की स्र्ापना
(B) टेलीफोन प्रणाली की स्र्ापना (D) कर्दल्ली में मेरो की स्र्ापना
During British regime which one of the following work was not performed?
(A) Establishment of railways (C) Establishment of wire-less system
(B) Establishment of telephone system (D) Establishment of Metro in Delhi
Q.20 वनम्नवलवखत कर्न-I एिं कर्न-II को साििानीपूिथक पकढ़ए और वनम्नवलवखत विकल्पों में से सही 1
विकल्प का चयन कीवजए -
कर्न I- भारत में मानिपूंजी को वनखारनेएिं िम शवक्त को और अविक उत्पार्दक के वलए कौशल-
अंतराल की खाई को कम करने की जरूरत है।
कर्न II- कौशल प्रकृ वत में मूतथ हैं, उन्द्हें उनके मावलक से अलग ककया जा सकता है।
विकल्प:
(A) कर्न-I सत्य है एिं कर्न-II असत्य है ।
(B) कर्न-I असत्य है एिं कर्न-II सत्य है ।
(C) र्दोनों कर्न सत्य है |
(D) र्दोनों कर्न असत्य है |
Read the following statement-I and statement-II carefully and choose the correct
alternatives from the followings-
Statement-I: There is a need to bridge the skill gap in India in order to enhance
human capital and bring about more productive labour force.
Statement-II: Skills are tangible in nature can be separated from its owner.
Alternatives:
(A) Statement-I is true and Statement-II is false.
(B) Statement-II is true and Statement-I is false.
(C) Both Statement are true.
(D) Both Statement are false.
Q.21 कॉलम- I और कॉलम- II में कर्दए गए कर्नों के समूह में से, कर्नों के सही युग्मों का चयन कीवजए - 1
From the set of statements given in Column-I and Column-II, Choose the correct
pair of statements-
कालम –I कालम-II
Column-I Column-II
a. वित्तीय क्षेत्र सुिार i. सरकार के राजस्ि एिं व्यय नीवत में सुिार।
Financial sector reforms Reforms in revenue and expenditure policy of
government.
b. राजकोषीय क्षेत्रक सुिार ii. FDI एिं FII क्षेत्र में सुिार।
Fiscal sector reforms Reforms in FDI and FII
c. अिमूल्यन iii. र्दूसरे र्देशों मुरा की तुलना में घरे लू मुरा के मूल्य में कटौती
Devaluation Lowering the value of domestic currency in
relation to currencies of other countries.
d. विर्देशी वनिेश iv. बैंक एिं गैर बैंक संस्र्ाएं
Foreign investment Banking and non-banking institutions
(A) a-I (B) b-ii (C) c-iii (D) d-iv
Q.22 वनम्नवलवखत अवभकर्न (A) एिं कारण (R) को ध्यान से पकिए और वनम्नवलवखत विकल्पों में से सही 1
विकल्प का चयन कीवजए।
अवभकर्न (A): सतत विकास उस पररितथन के बारे में है जो गरीब लोगों, समुर्दायों और र्देशों को
जागरुक, वशवक्षत, स्िस्र् और उत्पार्दक समाज में बर्दल र्देता है।
कारण (R) : सतत िारणीय विकास िन सृजन की िकालत करता है जो सामावजक-आर्र्थक समानता
उत्पन्न करता है।
विकल्प:
(A) अवभकर्न (A) एिं कारण (R) र्दोनों सत्य हैं और कारण (R),अवभकर्न (A) की सही व्याख्या है।
(B) अवभकर्न (A) एिं कारण (R) र्दोनों सत्य हैं और कारण (R), अवभकर्न (A) की सही व्याख्या
नहीं है।
(C) अवभकर्न (A) सत्य है लेककन कारण (R) असत्य है।
(D) अवभकर्न (A) असत्य है एिं कारण (R) सत्य है।
Read the following Assertion (A) and Reason (R).
Choose the correct alternative.
Assertion (A) - Sustainable development is about change that transform
impoverished people, ccommunities and countries into informed, educated,
healthy and productive society.
Reason (R) – Sustainable development advocates about wealth creation that
generates socio-economic equality.
Alternatives:
(A) Both Assertion (A) and Reason (R) are true and reason (R) is the correct
explanation of Assertion (A).
(B) Both Assertion (A) and Reason (R) are true and reason (R) is not the correct
explanation of Assertion (A).
(C) Assertion (A) is false but Reason (R) is true.
(D) Assertion (A) is true but Reason (R) is false.
अर्िा / OR
__________ पयाथिरण क्षरण का कारण नहीं है।
(A) अपवशि प्रबंिन (C) ग्लोबल िार्मिंग
(B) िनों की कटाई (D) हररत आिरण की रक्षा करना
_______________is not a cause for environmental degradation.
(A) Waste management (C) Global warming
(B) Deforestation (D) Guarding green cover
Q.23 वनम्नवलवखत अवभकर्न (A) एिं कारण (R) को ध्यान से पकिए और वनम्नवलवखत विकल्पों में से सही 1
विकल्प का चयन कीवजए।
अवभकर्न (A): मानि पूज ँ ी वनमाथण से भौवतक पूज
ँ ी की उत्पार्दकता में िृवि होती है |
कारण (R): कु शल एिं प्रवशवक्षत लोगों की अविक संख्या होने से निाचार एिं तकनीकी सुिारों की
सम्भािना कम हो जाती है।
विकल्प:
(A) अवभकर्न (A) एिं कारण (R) र्दोनों सत्य हैं और कारण (R) अवभकर्न (A) की सही व्याख्या है।
(B) अवभकर्न (A) एिं कारण (R) र्दोनों सत्य हैं और कारण (R) अवभकर्न (A) की सही व्याख्या
नहीं है।
(C) अवभकर्न (A) सत्य है लेककन कारण (R) असत्य है।
(D) अवभकर्न (A) असत्य है एिं कारण (R) सत्य है।
Read the following Assertion (A) and Reason (R), chose one of the correct
alternative given below.
Assertion (A) - Human capital formation increase productivity of physical
capital.
Reason (R) – Larger the number of skilled and trained personnel, lesser
possibilities of innovation and technical improvements in the area of production.
Alternatives:
(A) Both Assertion (A) and Reason (R) are true and reason (R) is the correct
explanation of Assertion (A).
(B) Both Assertion (A) and Reason (R) are true and reason (R) is not the correct
explanation of Assertion (A).
(C) Assertion (A) is false but Reason (R) is true.
(D) Assertion (A) is true but Reason (R) is false.
Q.24 वनम्नवलवखत कर्न-I एिं कर्न-II को साििानीपूिथक पकढ़ए और वनम्नवलवखत विकल्पों में से सही 1
विकल्प का चयन कीवजए -
कर्न I- बेहतर स्िच्छता सुवििाएँ प्रर्दान करने में पाककस्तान भारत से आगे है |
कर्न II- विकास के अनुभिों के मामले में चीन भारत और पाककस्तान से आगे है।
विकल्प:
(A) कर्न-I सत्य है एिं कर्न-II असत्य है ।
(B) कर्न-I असत्य है एिं कर्न-II सत्य है ।
(C) र्दोनों कर्न सत्य है।
(D) र्दोनों कर्न असत्य है।
Read the following statement-I and statement-II carefully and choose the correct
alternatives from the followings-
Statement-I: Pakistan is ahead of India in providing improved sanitation facilities.
Statement-II: China ranks ahead in terms of development experiences with
respect to India and Pakistan.
Alternatives:
(A) Statement-I is true and Statement-II is false.
(B) Statement-II is true and Statement-I is false.
(C) Both Statement are true.
(D) Both Statement are false.
Q.25 चीन की ग्रेट लीप फॉरिडथ योजना ________________ पर आिाररत र्ी | 1
(A) उद्योगों के विकास (C) अर्थव्यिस्र्ा का वनजीकरण
(B) कृ वष के विकास (D) A और B र्दोनों
The Great Leap Forward plan of China is focused on ___________________.
(A) Promotion of Industrialization (C) Privatization of Economy
(B) Promotion of Agriculture (D) A and B both
Q.26 वनम्नवलवखत में से ककस राष्ट्र के द्वारा एक बच्चे की नीवत अपनाया गया? 1
(A) भारत (C) पाककस्तान
(B) चीन (D) नेपाल
Which of the following country has adopted one child policy?
(A) India (C) Pakistan
(B) China (D) Nepal
Q.27 चीन की वनम्नवलवखत घटनाओं को कालानुिवमक िम में व्यिवस्र्त कीवजए और वनम्नवलवखत सही 1
विकल्पों का चयन कीवजए -
Arrange the following events of china in chronological order and choose the
correct alternatives given below-
i. हररत सिथहारा िांवत अवभयान iii. आर्र्थक सुिार
Green proletarian revolution campaign. Economic reforms
ii. ग्रेट लीप फारिडथ अवभयान iv. प्रर्म पंचिषीय योजना
Great leap forward campaign. First five year plan.
विकल्प:
Alternatives:
(A) iv, ii, i, iii (C) ii, iv, iii, i
(B) i, ii, iii, iv (D) iv, i , ii , iii
Q.28 “विरटश शासनकाल के अंतगथत परम्परागत हस्तवशल्प उद्योग बबाथर्द हो गया र्ा” | क्या आप इस 3
कर्न से सहमत हैं? कारण बताते हुए अपने उत्तर का समर्थन कीवजए।
“The traditional handicrafts industries were ruined under British rule”. Do
you agree with this statement? Give reason and support your answer.
अर्िा / OR
पंचिषीय योजना के सामान्द्य लक्ष्य “समता के सार् िृवि” की व्याख्या कीवजए।
Explain “Growth with Equity” as a common goal of five year plan.
Q.29 अवनयवमत एिं वनयवमत कार्मथकों के मध्य अन्द्तर स्पि कीवजए। 3
Distinguish between casual employee and regular employee.
Q.30 (a) सहकारी ऋण सवमवतयां क्या हैं? 2
What are co-operative credit societies?
(b) ग्रामीण साख के गैर-संस्र्ागत स्त्रोतों के संवक्षप्त व्याख्या कीवजए। 2
Discuss in brief about non-institutional sources of rural credit.
अर्िा / OR
मीना एक गृहणी है। घर के काम वनपटाने के बार्द िह अपने पवत के कपडे के शोरूम में उनकी मर्दर्द 4
करती है। क्या मीना को एक िवमक माना जा सकता है अर्िा नहीं? कारण के सार् व्याख्या कीवजए।
Meena is a housewife. Besides taking care of house she supports her husband in his
cloth showroom. She can be considered as worker or not. Explain with valid reason.
Q.31 वनम्नवलवखत वचत्र के आिार पर बताइए कक ककस प्रकार की बेरोजगारी पररलवक्षत होती है? इस 4
प्रकार की बेरोजगारी को कम करने के वलए कोई र्दो उपाय सुझाइए ।
On the basis of following figure, explain what type of unemployment is reflected?
Suggest any two measures for reducing this kind of unemployment.

Q.32 तावलका में र्दशाथए गए रुझानों को पकढ़ए और उसके अपने विश्लेषण पर एक संवक्षप्त रटप्पणी वलवखए | 4
रोजगार और सकल घरे लू उत्पार्द में क्षेत्रीय योगर्दान (% में)
जीडीपी में योगर्दान (% में) िम-शवक्त वितरण (% में)
क्षेत्रक
भारत चीन पाककस्तान भारत चीन पाककस्तान
कृ वष 17 7 23 44 26 42
उद्योग 29 41 18 25 28 24
सेिा 54 52 59 31 46 34
कु ल 100 100 100 100 100 100
सौजन्द्य से- विश्व विकास सूचक -2019, मुख्य सूचक एवशया एिं प्रशांत
Read the trends as shown in the table and write a short note on your analysis of
the same
Sectoral contribution in employment and GDP (in %)
Contribution to GDP (in %) Distribution of Workforce (in %)
Sectors
India China Pakistan India China Pakistan
Agriculture 17 7 23 44 26 42
Industry 29 41 18 25 28 24
Services 54 52 59 31 46 34
Total 100 100 100 100 100 100
Source: World Development Indicators – 2019, Key Indicators of Asia and Pacific.
Q.33 वनम्नवलवखत पररच्छेर्द को साििानीपूिथक पकढ़ए :
भारत का सूक्ष्म लघु और मध्यम उद्यम (MSME) क्षेत्र 2020 में एक बड़े बर्दलाि के वलए तैयार है ,
वजसमे अलीबाबा जैसे ई माके टप्लेस, लोगों में ितथमान फै शन के अनुसार ककफायती खार्दी उत्पार्द
और उद्यवमयों के वलए िे वडट डेटा आिाररत वडवजटल ऋण उपलब्ि कराने जैसी सुवििाओं की
शुरुआत की जानी है । हालाँकक, सूक्ष्म, लघु और मध्यम उद्यम क्षेत्र को अक्सर अर्थव्यिस्र्ा का आिार
माना जाता है क्योंकक यह सकल घरे लू उत्पार्द में लगभग 29 प्रवतशत और भारतीय वनयाथत में 48
प्रवतशत का योगर्दान र्देता है। भारत में कम लागत िाले ऋण की समय पर उपलब्िता सुवनवित
करने, व्यापार करने में आसानी और तकनीकी उन्नयन में सुिार, लाखों नौकररयों के सृजन की विकट
चुनौती का सामना करने, राष्ट्रीय आय का समान वितरण सुवनवित करने और बड़े पैमाने पर आयात
प्रवतस्र्ापन प्राप्त करने की कर्दशा में प्रमुख सुिारों और नीवतगत हस्तक्षेप की तत्काल आिश्यकता है।
विश्व बैंक ने हाल ही में भारत के सूक्ष्म, लघु और मध्यम उद्यम क्षेत्र जो, कोविड-19 संकटों से गंभीर
रूप से प्रभावित हुआ है की तत्काल तरलता और ऋण जरूरतों को पूरा करने के वलए 750 वमवलयन
डॉलर के ऋण को मंजूरी र्दी है। यह सरकार के आत्मवनभथर भारत विजन को भी बढ़ािा र्देगा ।
वनम्नवलवखत प्रश्नों के उत्तर उपरोक्त पररच्छेर्द एिं सामान्द्य समझ के आिार पर र्दीवजए :
(i) कोविड -19 महामारी के र्दौरान भारत का सूक्ष्म, लघु और मध्यम क्षेत्र ककन समस्याओं से सिाथविक 2
प्रभावित हुआ ?
(ii) अर्थव्यिस्र्ा में सूक्ष्म, लघु और मध्यम क्षेत्र की भूवमका की व्याख्या कीवजए। 1
(iii) “सूक्ष्म, लघु और मध्यम उद्यम के वलए िम की तुलना पूज ं ी की अविक आिश्यकता होती है |” क्या 2
आप इस कर्न से सहमत है ? व्याख्या कीवजए।
(iv) भारत में लघु उद्योगों की भूवमका एिं महत्ि की वसफाररश ककस सवमवत ने की ? 1
Read the following passage carefully:
India’s Micro, Small and Medium Enterprise (MSME) sector is poised for a mega
transformation in 2020, with the launch of an Alibaba-like e-marketplace, trendy
yet affordable khadi products to appeal to the masses and digital data-based credit
ratings to help entrepreneurs avail loans. However, the Micro, Small and Medium
Enterprises sector, often considered the bulwark of the economy as it contributes
around 29% to the GDP and 48% to the Indian export. There is an urgent need of
major reforms and policy interventions towards ensuring timely availability of low
cost credit, improving ease of doing business and technological up gradation, to
take on the formidable challenge of creating millions of jobs, ensure equitable
distribution of national income and achieving large scale import substitution. The
world bank has recently approved loan worth 750 million dollars to address the
immediate liquidity and credit needs of India’s Micro, Small and Medium
Enterprise sector that has been severely impact by COVID-19 crises. This will give
a push to the Atmanirbhar Bharat vision of the government.
On the basis of the given text and common understanding, answer the following
question:
(i) What were the main problems that most affected to MSME sector of India during
COVID-19 pandemic?
(ii) Explain to important role of MSME sector in the economy.
(iii) “Micro, Small and Medium Enterprise requires more capital than labour”. Do
you agree with this statement? Explain.
(iv) Which committee recommended the role and importance of small scale industries
in India?
Q.34 विकास के संसािन के रूप में मानि पूँजी एक महत्िपूणथ भूवमका हो सकती है। क्या आप इस कर्न 6
से सहमत है? अपने उत्तर को वसि कीवजए।
Human capital can play an important role as resource of development. Do you
agree with this statement? Justify your answer.
अर्िा / OR
वनम्नवलवखत की व्याख्या कीवजए –
Explain the following-
(i) कृ वष विपणन
Agricultural Marketing
(ii) नाबाडथ
NABARD
(iii) कृ वष विवििीकरण
Agricultural Diversification.
ROLL NO………………………………….. SUBJECT CODE: - 030
अनुिमांक संख्या ............................ विषय कोड: - 030
KENDRIYA VIDYALAYA SANGATHAN, JAIPUR REGION
कें रीय विद्यालय संगठन, जयपुर संभाग SET-4
Practice Paper
अभ्यास पत्र
SUBJECT: - ECONOMICS CLASS: - XII
विषय: - अर्थशास्त्र कक्षा: - 12
MAXIMUM MARKS: - 80 MARKS TIME: - 3:00 HOURS
अविकतम अंक: - 80 अंक समय: - 3:00 घंटे
-------------------------------------------------------------------------------------------------------------------------------------
General instructions:
सामान्द्य वनर्देश:
(1) This question paper contains two sections:
इस प्रश्न पत्र के र्दो खण्ड है :-
Section–A (Introductory Macro Economics) Section–B (Indian Economic Development)
खण्ड-अ (पररचयात्मक समवि अर्थशास्त्र) खण्ड-ब (भारतीय अर्थव्यिस्र्ा का विकास)
(2) All questions in both the sections are compulsory. However, there is internal choice in some questions.
Marks for questions are indicated against each question.
र्दोनों खण्डों के सभी प्रश्न करना अवनिायथ है | यद्यवप कु छ प्रश्नों में आन्द्तररक चयन प्रर्दान ककया गया है | प्रत्येक प्रश्न के अंक
उस प्रश्न के सामने अंककत है |
(3) Question number 1 –10 and 18 – 27 are very short answer / multiple choice questions carrying 1 mark
each. They are required to be answered in one word or one sentence each.
प्रश्न संख्या 1 से 10 एिं 18 से 27 तक के सभी एक-एक अंक के अवत लघूत्तरात्मक / बहुविकल्पीय प्रश्न है | इन प्रश्नों के उत्तर
एक शब्र्द या एक िाक्य में र्दीवजए |
(4) Question number 11 – 12 and 28 – 29 are short answer questions carrying 3 marks each. Answers to them
should normally not exceed 60-80 words each.
प्रश्न संख्या 11 से 12 एिं 28 से 29 तक के सभी प्रश्न लघूत्तरात्मक प्रश्न है, जो कक तीन-तीन अंक के है | इन प्रश्नों के उत्तर
60-80 शब्र्दों से अविक नहीं होने चावहए |
(5) Question number 13 – 15 and 30 – 32 are also short answer questions carrying 4 marks each. Answers to
them should normally not exceed 80-100 words each.
प्रश्न संख्या 13 से 15 एिं 30 से 32 तक के सभी प्रश्न भी लघूत्तरात्मक प्रश्न है, जो कक चार-चार अंक के है | इन प्रश्नों के
उत्तर 80-100 शब्र्दों से अविक नहीं होने चावहए |
(6) Question number 16 – 17 and 33 – 34 are long answer questions carrying 6 marks each. Answers to them
should normally not exceed 100-150 words each.
प्रश्न संख्या 16 से 17 एिं 33 से 34 तक के सभी प्रश्न र्दीघथ-उत्तरात्मक प्रश्न है, जो कक छह-छह अंक के है | इन प्रश्नों के
उत्तर 100-150 शब्र्दों से अविक नहीं होने चावहए |
(7) Answers should be brief and to the point and the above word limits should be adhered to as far as
possible.
सभी प्रश्नों के उत्तर यर्ा सम्भि प्रर्दत्त शब्र्द सीमा को ध्यान में रखते हुए संवक्षप्त, तथ्यात्मक एिं सटीक होने चावहए |
(8) An additional 15 minutes has been allotted to read the question paper.
इस प्रश्न पत्र को पढ़ने हेतु पंरह (15) वमनट का अवतररक्त समय आिंरटत ककया गया है |
Q.NO. Section-A (Introductory Macro Economics) Marks
प्र. सं. खण्ड-अ (पररचयात्मक समवि अर्थशास्त्र) अंक
1. Read the following Assertion (A) and Reason (R). 1
Choose the correct alternative.
वनम्नवलवखत अवभकर्न (A) और कारण (R) को ध्यानपूिथक पकढ़ए |
वनम्नवलवखत विकल्पों में से एक सही विकल्प का चयन कीवजए |
Assertion (A): Compensation given to victims of Uttarakhand natural disaster
(tragedy) is a commendable welfare measures taken by the government, but it is not
included in the estimation of national income.
अवभकर्न (A): उत्तराखंड प्राकृ वतक आपर्दा (त्रासर्दी) के पीवड़तों को कर्दया गया मुआिजा सरकार
द्वारा उठाया गया एक सराहनीय कल्याणकारी कर्दम है, लेककन यह राष्ट्रीय आय के अनुमान में
शावमल नहीं है।

Reason (R): Transfer payments do not lead to corresponding flow of goods and services
in the economy, hence are not included in the calculation of national income of the
country.
कारण (R): हस्तांतरण भुगतान से अर्थव्यिस्र्ा में िस्तुओं और सेिाओं का तर्दनुरूप प्रिाह नहीं
होता है, इसवलए इसे र्देश की राष्ट्रीय आय की गणना में शावमल नहीं ककया जाता है।
Alternatives:
विकल्प:
(A) Both Assertion (A) and Reason (R) are true and Reason (R) is the correct explanation
of Assertion (A).
अवभकर्न (A) और कारण (R) र्दोनों सत्य हैं और कारण (R) अवभकर्न (A) का सही स्पिीकरण है।
(B) Both Assertion (A) and Reason (R) are true and Reason (R) is not the correct
explanation of Assertion (A).
अवभकर्न (A) और कारण (R) र्दोनों सत्य हैं और कारण (R) अवभकर्न (A) का सही स्पिीकरण
नहीं है।
(C) Assertion (A) is true but Reason (R) is false.
अवभकर्न (A) सत्य है लेककन कारण (R) गलत है।
(D) Assertion (A) is false but Reason (R) is true.
अवभकर्न (A) गलत है लेककन कारण (R) सत्य है।

2. In the context of commercial bank, which of the following alternatives is correct? 1


िावणवज्यक बैंक के संर्दभथ में, वनम्नवलवखत में से कौन-सा विकल्प सही है?
(i) Deposit are liabilities of Commercial bank.
जमाएँ िावणवज्यक बैंक की र्देनर्दाररयां है।
(ii) Commercial banks are creator of money.
िावणवज्यक बैंक िन का सजथन करते है।
(iii) Commercial banks accept deposits of the general public.
िावणवज्यक बैंक आम जनता की जमाएँ स्िीकार करते हैं।
(iv) Accept deposit of World Bank for agriculture and Rural development.
कृ वष एिं ग्रामीण विकास के वलए विश्व बैंक की जमा रावश स्िीकार करते है |
Alternatives:
विकल्प:
(A) (i) and (ii) (B) (ii) and (iii) (C) (i), (ii) and (iii) (D) (i) and (iv)

3. Mr. Rahul Kumar, an economics teacher, was explaining the concept of ‘minimum 1
percentage of the total deposits to be kept by any commercial bank with the Central
Bank of the country, as per norms and statute prevailing in the country’. From the
following, choose the correct alternative which specifies towards the concept
explained by him.

िी राहुल कु मार, एक अर्थशास्त्र वशक्षक, 'र्देश में प्रचवलत मानर्दंडों और कानून के अनुसार, ककसी
भी िावणवज्यक बैंक द्वारा र्देश के कें रीय बैंक के पास रखी जाने िाली कु ल जमा का न्द्यनू तम
प्रवतशत' की अििारणा को समझा रहे र्े। वनम्नवलवखत में से, उस सही विकल्प का चयन कीवजए
जो उनके द्वारा समझाई गई अििारणा को वनर्र्दथि करता है |
(A) Cash Reserve Ratio (B) Repo Rate
नकर्द आरवक्षत अनुपात रे पो र्दर
(C) Bank Rate (D) Statutory Liquidity Ratio
बैंक र्दर िैिावनक तरलता अनुपात
4. Read the following Assertion (A) and Reason (R). 1
Choose the correct alternative.
वनम्नवलवखत अवभकर्न (A) और कारण (R) को ध्यानपूिथक पकढ़ए |
वनम्नवलवखत विकल्पों में से एक सही विकल्प का चयन कीवजए |
Assertion (A): Excess demand refers to the situation when aggregate demand is in
excess of aggregate supply corresponding to full employment in the economy i.e.,
AD>AS, corresponding to full employment. It is when actual level of aggregate
demand is more than required or planned level of aggregate demand to maintain full
employment.
अवभकर्न (A): अवतररक्त मांग उस वस्र्वत को संर्दर्भथत करती है जब कु ल मांग अर्थव्यिस्र्ा में पूणथ
रोजगार यानी AD>AS, पूणथ रोजगार के अनुरूप कु ल आपूर्तथ से अविक होती है। यह तब होता है
जब कु ल मांग का िास्तविक स्तर पूणथ रोजगार बनाए रखने के वलए आिश्यक या कु ल मांग के
वनयोवजत स्तर से अविक होता है।

Reason (R): To correct inflationary gap, Bank rate, Repo rate, Reserve Repo rate and
legal reserve ratio is increased by the central Bank to reduce supply of money so that
purchasing power of people can be curtailed and inflationary gap can be controlled.
कारण (R): मुरास्फीवत अंतर को ठीक करने के वलए, कें रीय बैंक द्वारा िन की आपूर्तथ को कम करने
के वलए बैंक र्दर, रे पो र्दर, ररजिथ रे पो र्दर और कानूनी आरवक्षत अनुपात बढ़ाया जाता है ताकक
लोगों की िय शवक्त को कम ककया जा सके और मुरास्फीवत अंतर को वनयंवत्रत ककया जा सके ।
Alternatives:
विकल्प:
(A) Both Assertion (A) and Reason (R) are true and Reason (R) is the correct explanation
of Assertion (A).
अवभकर्न (A) और कारण (R) र्दोनों सत्य हैं और कारण (R) अवभकर्न (A) का सही स्पिीकरण है।
(B) Both Assertion (A) and Reason (R) are true and Reason (R) is not the correct
explanation of Assertion (A).
अवभकर्न (A) और कारण (R) र्दोनों सत्य हैं और कारण (R) अवभकर्न (A) का सही स्पिीकरण
नहीं है।
(C) Assertion (A) is true but Reason (R) is false.
अवभकर्न (A) सत्य है लेककन कारण (R) गलत है।
(D) Assertion (A) is false but Reason (R) is true.
अवभकर्न (A) गलत है लेककन कारण (R) सत्य है।

5. Match the following items given in Column-A with those in Column-B. Choose the 1
correct alternative.
वनम्नवलवखत में से कॉलम- A में र्दी गई िस्तुओं को कॉलम-B में र्दी गई िस्तुओं से वमलान कीवजए |
वनम्नवलवखत विकल्पों में से एक सही विकल्प का चयन कीवजए |
Column-A Column-B
कॉलम- A कॉलम-B
i Qualitative Measures a Reverse Repo Rate
गुणात्मक उपाय ररिसथ रे पो र्दर
ii Fall in Bank Rate b Controls excess demand
बैंक र्दर में वगरािट अवतररक्त मांग को वनयंवत्रत करना
iii Sale of government securities by RBI C Controls deficient demand
आरबीआई द्वारा सरकारी प्रवतभूवतयों की वबिी कमी िाली मांग को वनयंवत्रत करना
iv Quantitative Measures d Imposing margin requirement
मात्रात्मक उपाय उपांत की आिश्यकता को र्ोपना
v Credit Creation e Commercial bank
साख सृजन िावणवज्यक बैंक
Alternatives:
विकल्प:
(A) i-d ii-a iii-c iv-b v-e (B) i-d ii-a iii-b iv-c v-e
(C) i-d ii-b iii-c iv-a v-e (D) i-d ii-c iii-b iv-a v-e

6. In an economy, the saving and investment functions are given as: 1


S = (–)100 + 0.2Y I = (–) 30 + 0.1Y
The equilibrium levels of income will be______.
एक अर्थव्यिस्र्ा में, बचत और वनिेश कायथ इस प्रकार कर्दए गए हैं:
S = (–)100 + 0.2Y I = (–) 30 + 0.1Y
आय का संतुलन स्तर ______ होगा।
(A) 600 (B) 700 (C) 1,000 (D) 900

7. Ex- ante saving refers to: 1


पूिथ-पूिथ बचत से तात्पयथ है:
(A) Desired saving during the period of one year.
एक िषथ की अिवि के र्दौरान िांवछत बचत
(B) Planned investment during the period of one year.
एक िषथ की अिवि के र्दौरान वनयोवजत वनिेश
(C) Actual saving during the period of one year.
एक िषथ की अिवि के र्दौरान िास्तविक बचत
(D) Both (A) and (B).
(A) और (B) र्दोनों

8. Read the given news taken from the article “Currency caution: On the Indian rupee’s 1
depreciation” published in The Hindu on 23rd July, 2022.
“The Indian rupee is experiencing its worst slump in four years. Since the start of 2022,
the currency has depreciated by more than 7% against the US dollar, weakening past a
historic low of 80 to a dollar mark earlier this week.” In layman terms, which of the
following is the reason for depreciation of Indian rupee?

23 जुलाई, 2022 को र्द बहंर्दू में प्रकावशत लेख "मुरा साििानी: भारतीय रुपये के मूल्यह्रास" से ली
गई र्दी गई खबर को पकढ़ए ।
“भारतीय रुपया चार िषों में सबसे खराब वगरािट का अनुभि कर रहा है। 2022 की शुरुआत के
बार्द से, अमेररकी डॉलर के मुकाबले मुरा में 7% से अविक की वगरािट आई है, जो इस सप्ताह की
शुरुआत में 80 प्रवत डॉलर के ऐवतहावसक वनचले स्तर से कमजोर हो गई है”।
सामान्द्य शब्र्दों में, भारतीय रुपए के मूल्यह्रास का वनम्नवलवखत में से कौन-सा कारण है?
(A) Increase in demand of US dollar and decrease in supply of US dollar.
अमेररकी डॉलर की मांग में िृवि और अमेररकी डॉलर की आपूर्तथ में कमी
(B) Increase in both demand and supply of US dollar.
अमेररकी डॉलर की मांग और आपूर्तथ र्दोनों में िृवि
(C) Increase in supply of US dollar and decrease in demand of US dollar.
अमेररकी डॉलर की आपूर्तथ में िृवि और अमेररकी डॉलर की मांग में कमी
(D) Decrease in both demand and supply of US dollar.
अमेररकी डॉलर की मांग और आपूर्तथ र्दोनों में कमी

9. Two friends Rajesh and Mohit were discussing the exchange rate systems. “Under 1
this system, the exchange rates are determined by the market forces of demand and
supply. However, deliberate efforts are made by the competent authority to keep the
exchange rates within a specific range”. The above mentioned statement was given
by Rajesh, identify the type of exchange rate system was he talking about.

र्दो वमत्र राजेश और मोवहत विवनमय र्दर प्रणाली पर चचाथ कर रहे र्े। “इस प्रणाली के तहत,
विवनमय र्दरें मांग और आपूर्तथ की बाजार शवक्तयों द्वारा वनिाथररत की जाती हैं। हालाँकक, सक्षम
प्राविकारी द्वारा विवनमय र्दरों को एक विवशि सीमा के भीतर रखने के वलए जानबूझकर प्रयास
ककए जाते हैं”। उपयुथक्त कर्न राजेश द्वारा प्रस्तुत ककया गया र्ा, पहचान कीवजए कक िह ककस
प्रकार की विवनमय र्दर प्रणाली के बारे में बात कर रहा र्ा ।
(A) Fixed Exchange Rate (B) Floating Exchange Rate
वनवित विवनमय र्दर फ्लोटटंग विवनमय र्दर
(C) Managed Floating Exchange Rate (D) All of them
प्रबंवित फ्लोटटंग विवनमय र्दर इनमे से सभी
10. Identify the correct pair of items from the following Column-I and Column-II: 1
वनम्नवलवखत कॉलम-I और कॉलम-II में से िस्तुओं के सही युग्म की पहचान कीवजए:
Column-I Column-II
कॉलम-I कॉलम-II
i Exports of merchandise a Debit item in the Current Account of BOP
माल का वनयाथत भुगतान संतुलन के चालू खाते में डेवबट आइटम
ii A gift from foreigner b Credit item in the Capital Account of BOP
विर्देशी व्यवक्त से एक उपहार भुगतान संतुलन के पूंजी खाते में िे वडट आइटम
iii Imports of services c Debit item in the Current Account of BOP
सेिाओं का आयात भुगतान संतुलन के चालू खाते में डेवबट आइटम
iv Lending to abroad d Credit item in the Current Account of BOP
विर्देश को ऋण र्देना भुगतान संतुलन के चालू खाते में िे वडट आइटम
Alternatives:
विकल्प:
(A) (iii)-c (B) (iv)-d (C) (i)-a (D) (ii)-b

11.(A) What is depreciation of domestic currency in terms of US dollar? How it will affect 1+2=3
your cost of education, if you are planning to go to USA for higher studies?

अमेररकी डॉलर के संर्दभथ में घरे लू मुरा का मूल्यह्रास क्या है? यकर्द आप उच्च अध्ययन के वलए
अमेररका जाने की योजना बना रहे हैं तो इसका आपकी वशक्षा की लागत पर क्या प्रभाि पड़ेगा?
OR / अर्िा
(B) Indian investors borrow from abroad.
भारतीय वनिेशक विर्देशों से ऋण लेते हैं।
Answer the following questions:
वनम्नवलवखत प्रश्नों के उत्तर र्दीवजए :
(i) In which sub- account and on which side of the balance of the payments account will 1½
this borrowing be recorded? Give reason.
यह उिार ककस उप-खाते में और भुगतान खाते के शेष के ककस तरफ र्दजथ ककया जाएगा ? कारण
र्दीवजए |

(ii) What is the impact of this borrowing on exchange rate? Explain. 1½


इस उिार का विवनमय र्दर पर क्या प्रभाि पड़ता है? िणथन कीवजए |

12. “Many goods and services which may contribute to welfare, but are not included in 3
estimating Gross Domestic Product (GDP).” Do you agree with the given statement?
Give valid reason in support of your answer.
"कई िस्तुएँ और सेिाएँ जो कल्याण में योगर्दान र्दे सकती हैं, लेककन सकल घरे लू उत्पार्द
(जीडीपी) के आकलन में शावमल नहीं हैं।" क्या आप कर्दए गए कर्न से सहमत हैं? अपने उत्तर के
समर्थन में िैि कारण र्दीवजए।

13. In an economy, the aggregate demand function is given by:


AD = 200 + 0.75Y
The full employment level of income is ₹ 500 crore.

ककसी अर्थव्यिस्र्ा में, कु ल मांग फलन इस प्रकार कर्दया जाता है:


AD = 200 + 0.75Y
पूणथ रोजगार आय का स्तर ₹ 500 करोड़ है।
(i) Calculate equilibrium level of income. 1
आय के संतुलन स्तर की गणना कीवजए |
(ii) What is the situation in the economy – inflationary or deflationary? How much is the 2
inflationary gap / deflationary gap? Give valid reasons in support of your answer.
अर्थव्यिस्र्ा की वस्र्वत क्या है - मुरास्फीवतकारी या अिस्फीवतकारी ? मुरास्फीवत / अपस्फीवत
का अंतर ककतना है ? अपने उत्तर के समर्थन में िैि कारण र्दीवजए ।
(iii) Calculate the change in investment needed to combat the inflationary / deflationary 1
gap in the economy.
अर्थव्यिस्र्ा में मुरास्फीवत / अपस्फीवत अंतर से वनपटने के वलए आिश्यक वनिेश में पररितथन की
गणना कीवजए ।

14.(A) ‘Monetary measures offer a valid solution to the problem of inflationary gap in an 2+2=4
economy’. State and discuss any two monetary measures to justify the given
statement.
'मौकरक उपाय ककसी अर्थव्यिस्र्ा में मुरास्फीवत के अंतर की समस्या का िैि समािान प्रर्दान
करते हैं'। कर्दए गए कर्न को सही ठहराने के वलए ककन्द्हीं र्दो मौकरक उपायों को बताइए एिं उन
पर चचाथ भी कीवजए ।
OR / अर्िा
(B) Can government spending help in the correction of the situation of excess demand? 1
क्या सरकारी खचथ अवतररक्त मांग की वस्र्वत को सुिारने में मर्दर्द कर सकता है?
(C) What measures can the government and Central bank take in order to control the 1½+1½=3

deflationary?
अपस्फीवत को वनयंवत्रत करने के वलए सरकार और कें रीय बैंक क्या उपाय कर सकते हैं?
15.(A) Define Money Multiplier. 1
िन गुणक को पररभावषत कीवजए ।
(B) ‘Credit creation is inversely related to the reserve deposit ratio’. Justify the given 3
statement, using a hypothetical example.
‘साख सजथन विपरीत रूप से आरवक्षत जमा अनुपात से संबवं ित है'। एक काल्पवनक उर्दाहरण का
प्रयोग करते हुए, कर्दए गए कर्न की पुवि कीवजए |
16.(A) Suppose in the budget, the government proposed to raise the excise duty on
cement. It is also proposed to raise the income tax on individuals earning more than
rupees one crore per annum.

मान लीवजए कक बजट में सरकार ने सीमेंट पर उत्पार्द शुल्क बढ़ाने का प्रस्ताि रखा है। इसके सार्
प्रवत िषथ एक करोड़ रुपये से अविक आय िाले व्यवक्तयों पर आयकर बढ़ाने का भी प्रस्ताि ककया
गया है।
(i) Identify and explain the type of taxes proposed by the government. 2
सरकार द्वारा प्रस्तावित करों के प्रकार को पहचावनए एिं समझाइए |
(ii) Was the objective only to earn revenue for the government? 2
क्या इसका उद्देश्य के िल सरकार के वलए राजस्ि कमाना र्ा?
(iii) What possible welfare objective could the government be considering? 2
सरकार ककस संभावित कल्याणकारी उद्देश्य पर विचार कर सकती है?
OR / अर्िा
(B) From the information given below, calculate:
वनम्नवलवखत जानकारी से, गणना कीवजए :
(i) Revenue receipts 1
राजस्ि प्रावप्तयाँ
(ii) Non-debt creating capital receipts 2
ऋण-रवहत पूंजी सजथन प्रावप्तयाँ
(iii) Fiscal deficit 2
राजकोषीय घाटा
(iv) Primary deficit 1
प्रार्वमक घाटा
SR. NO. PARTICULARS AMOUNT (₹ In crore)
ि. स. वििरण रावश (₹ करोड़ में)
1. Revenue deficit 6,000
राजस्ि घाटा
2. Total receipts excluding borrowings 40,000
उिार को छोड़कर कु ल प्रावप्तयाँ
3. Capital expenditure 30,000
पूंजीगत व्यय
4. Interest payment 40% of Revenue deficit
ब्याज भुगतान राजस्ि घाटे का 40%
5. Revenue expenditure 20,000
राजस्ि व्यय
17. From the following data, find the missing value of ‘Government final consumption 3+3=6
expenditure’ and ‘Mixed Income of self- Employed’.

वनम्नवलवखत आंकड़ों को र्देखते हुए, 'सरकारी अंवतम उपभोग व्यय' और 'स्ि-रोज़गार की वमवित
आय' के लुप्त मूल्य ज्ञात कीवजए।
SR. NO. PARTICULARS AMOUNT (₹ In crore)
ि. स. वििरण रावश (₹ करोड़ में)
i National income 71,000
राष्ट्रीय आय
ii Gross domestic capital formation 10,000
सकल घरे लू पूंजी वनमाथण
iii Government final consumption expenditure ?
सरकार का अंवतम उपभोग व्यय
iv Mixed income of self-employed ?
स्ि-रोज़गार की वमवित आय
v Net factor income from abroad 1,000
विर्देशों से शुि कारक आय
vi Net indirect taxes 2,000
शुि अप्रत्यक्ष कर
vii Profits 1,200
लाभ
viii Wages and salaries 15,000
मजर्दूरी और िेतन
ix Net exports 5,000
शुि वनयाथत
x Private final consumption expenditure 40,000
वनजी अंवतम उपभोग व्यय
xi Consumption of fixed capital 3,000
अचल पूंजी की खपत
xii Operating surplus 30,000
पररचालन अविशेष

SECTION–B (Indian Economic Development)


खण्ड-ब (भारतीय अर्थव्यिस्र्ा का विकास)
18. Read the following Assertion (A) and Reason (R). 1
Choose the correct alternative.
वनम्नवलवखत अवभकर्न (A) और कारण (R) को ध्यानपूिथक पकढ़ए |
वनम्नवलवखत विकल्पों में से एक सही विकल्प का चयन कीवजए |
Assertion (A): To increase economic cooperation among nations in the neighbored,
regional and economic grouping are formed such as SAARC, EU and G-8 etc.
अवभकर्न (A): पड़ोसी र्देशों के बीच आर्र्थक सहयोग बढ़ाने के वलए क्षेत्रीय और आर्र्थक समूह
बनाए जाते हैं जैसे साकथ , ईयू और जी-8 आकर्द।

Reason (R): Due to economic transformation and globalization, nations have been
primarily trying to adopt various means which will strengthen their own domestic
economics.
कारण (R): आर्र्थक पररितथन और िैश्वीकरण के कारण, राष्ट्र मुख्य रूप से विवभन्न तरीकों को
अपनाने की कोवशश कर रहे हैं वजनसे स्िंय की घरे लू अर्थव्यिस्र्ा को मजबूत ककया जा सके ।
Alternatives:
विकल्प:
(A) Both Assertion (A) and Reason (R) are true and Reason (R) is the correct explanation
of Assertion (A).
अवभकर्न (A) और कारण (R) र्दोनों सत्य हैं और कारण (R) अवभकर्न (A) का सही स्पिीकरण है।
(B) Both Assertion (A) and Reason (R) are true and Reason (R) is not the correct
explanation of Assertion (A).
अवभकर्न (A) और कारण (R) र्दोनों सत्य हैं और कारण (R) अवभकर्न (A) का सही स्पिीकरण
नहीं है।
(C) Assertion (A) is true but Reason (R) is false.
अवभकर्न (A) सत्य है लेककन कारण (R) गलत है।
(D) Assertion (A) is false but Reason (R) is true.
अवभकर्न (A) गलत है लेककन कारण (R) सत्य है।

19. From the set of events / systems given in Column- I and corresponding relevant fact 1
given in Column -II, about China, choose the correct pair of statement:
चीन के बारे में कॉलम-I में र्दी गई घटनाओं / प्रणावलयों के समूह का कॉलम-II में कर्दए गए संबंवित
प्रासंवगक तथ्यों में से, कर्नों के सही युग्म का चयन कीवजए :
Column- I Column -II
कॉलम-I कॉलम-II
i Commune system a 1966-76
कम्यून प्रणाली
ii Special Economic Zones b To attract foreign investors
विशेष आर्र्थक क्षेत्र विर्देशी वनिेशकों को आकर्षथत करने के वलए
iii One-child policy c Cultivating land collectively
एक बच्चे की नीवत सामूवहक रूप से भूवम पर खेती करना
iv Great Proletarian Cultural Revolution. d Major reason for low population growth.
महान सिथहारा सांस्कृ वतक िांवत कम जनसंख्या िृवि का प्रमुख कारण
Alternatives:
विकल्प:
(A) (iii)-c (B) (iv)-d (C) (i)-a (D) (ii)-b
20. Arrange the following events of China in chronological order: 1
चीन की वनम्नवलवखत घटनाओं को कालानुिवमक िम में व्यिवस्र्त कीवजए :
(i) Great Proletarian Culture Revolution (ii) Great Leap Forward Campaign
महान सिथहारा सांस्कृ वतक िांवत ग्रेट लीप फॉरिडथ अवभयान
(iii) Introduction of Economic Reforms (iv) First Five Year Plan
आर्र्थक सुिारों का पररचय प्रर्म पंचिषीय योजना
Choose the correct alternative:
सही विकल्प का चयन कीवजए :
(A) (ii), (iv), (iii), (i) (B) (iv), (ii), (i), (iii)
(C) (ii), (iv), (i), (iii) (D) (iv), (i), (ii), (iii)
21. Which one of the following is not true about the similar developmental strategies 1
about India and Pakistan for their development paths?
भारत और पाककस्तान के विकास पर् के सन्द्र्दभथ में समान विकासात्मक रणनीवतयों के बारे में
वनम्नवलवखत में से कौन-सा एक सत्य नहीं है?
(A) Both the economies followed the path of mixed economic structure.
र्दोनों अर्थव्यिस्र्ाओं ने वमवित आर्र्थक संरचना के मागथ को अपनाया।
(B) Both the economies started their developmental path at the same time soon after
their independence.
र्दोनों अर्थव्यिस्र्ाओं ने अपनी स्ितंत्रता के तुरंत पिात एक ही समय में अपना विकास पर् शुरू
ककया।
(C) Both the countries announced their first five year plan in the year1951.
र्दोनों र्देशों ने िषथ 1951 में अपनी पहली पंचिषीय योजना की घोषणा की |
(D) Both the countries adopted similar strategies, such as creating a large public sector
and raising public expenditure on social development.
र्दोनों र्देशों ने समान रणनीवतयाँ अपनाईं, जैसे बड़े सािथजवनक क्षेत्र बनाना और जनता के
सामावजक विकास पर व्यय पर अविकता |

22. Read the following statements carefully. 1


वनम्नवलवखत कर्नों को ध्यानपूिथक पकढ़ए |
Statement-I: Since independence, the economic condition of many farmers across
India has improved as they have adopted horticulture as a secondary source of
income.
कर्न-I: आज़ार्दी के बार्द से, भारत भर में कई ककसानों की आर्र्थक वस्र्वत में सुिार हुआ है क्योंकक
उन्द्होंने बागिानी को आय के वद्वतीय स्रोत के रूप में अपनाया है।
Statement-II: Varying climate and soil conditions have given India an added
advantage to be the producer of diverse horticultural crops.
कर्न-II: बर्दलती जलिायु और वमट्टी की वस्र्वतयों ने भारत को विविि बागिानी फसलों के
उत्पार्दक होने का अवतररक्त लाभ कर्दया है।
In the light of given statements, choose the correct alternatives from the following:
कर्दए गए कर्नों के आलोक में वनम्नवलवखत में से सही विकल्प का चयन कीवजए :
(A) Statement-I is true and statement-II is false.
कर्न-I सत्य है और कर्न-II गलत है।
(B) Statement-I is false and statement-II is true.
कर्न-I गलत है और कर्न-II सत्य है।
(C) Both statement-I and statement-II are true.
र्दोनों कर्न-I और कर्न-II सत्य है।
(D) Both statement-I and statement-II are false.
र्दोनों कर्न-I और कर्न-II गलत है।
23. Which of the following sources collect data on unemployment? 1
वनम्नवलवखत में से कौन-सा स्रोत बेरोजगारी पर तथ्य एकत्र करता है?
(A) Census Reports of India.
भारत की जनगणना ररपोटथ
(B) National Sample Survey Organization's Reports of Employment and Unemployment
Situation.
राष्ट्रीय नमूना सिेक्षण संगठन की रोजगार और बेरोजगारी की वस्र्वत की ररपोटथ
(C) Directorate General of Employment and Training Data of Registration with
Employment exchange.
रोजगार और प्रवशक्षण महावनर्देशालय का रोजगार कायाथलय के सार् पंजीकृ त वनर्र्दथि
(D) All of the above.
उपरोक्त सभी

24. Identify the Correctly Matched pair from the items in Column A and by matching 1
them with the items in Column B:
कॉलम-A के तथ्यों की पहचान कॉलम-B में कर्दए गए तथ्यों के सार् करते हुए सही युग्म का
वमलान कीवजए :
Column A Column B
कॉलम-A कॉलम-B
1- Sustainable (a) The process whereby the real per capita income of the
Development country increases over a long period of time.
सतत िारणीय िह प्रकिया वजससे र्देश की िास्तविक प्रवत व्यवक्त आय लंबी अिवि में
विकास बढ़ती है |
2- Economic (b) Development that means the need of the present
Development generation without compromising the ability of the future
generation to meet their own needs.
आर्र्थक विकास विकास का अर्थ है भविष्य की पीढ़ी की अपनी जरूरतों को पूरा करने की
क्षमता से समझौता ककए वबना ितथमान पीढ़ी की आिश्यकताओं को पूणथ
करना है ।
3- Economic (c) Comprehensive concepts including increasing real per
Growth capita income of a Country over a long period of time along
with reduction in poverty, inequality and unemployment.
िास्तविक प्रवत व्यवक्त आय बढ़ाने सवहत व्यापक अििारणाएँ लंबे
आर्र्थक संिृवि समय से र्देश में गरीबी, असमानता और बेरोजगारी में कमी आई है।
4- Environment (d) The total planetary inheritance and the totality of all
resources.
पयाथिरण कु ल ग्रहीय विरासत और सभी संसािनों की समग्रता।
Alternatives:
विकल्प:
(A) 1- (a) (B) 2-(b) (C) 3- (c) (D) 4- (d)

25. Which of the following statements correctly represents actions taken by the 1
government towards liberalization?
वनम्नवलवखत में से कौन-सा कर्न उर्दारीकरण की कर्दशा में सरकार द्वारा उठाए गए कर्दमों का सही
प्रवतवनवित्ि करता है?
P Levying high tariffs to discourage import and promoting the consumption of
domestic goods and services.
आयात को हतोत्सावहत करने और घरे लू िस्तुओं और सेिाओं की खपत को बढ़ािा र्देने के वलए उच्च
प्रशुल्क लगाना।
Q Devaluation of the rupees to encourage inflow of foreign exchange.
विर्देशी मुरा के प्रिाह को प्रोत्सावहत करने के वलए रुपए का अिमूल्यन।
R Allowing for private banks to make decisions independent of the RBI restrictions.
वनजी बैंकों को आरबीआई के प्रवतबंिों से स्ितंत्र वनणथय लेने की अनुमवत र्देना।
S Fixing prices of certain industrial goods in order to support increased consumption of
these goods to boost the manufacturing industry.
विवनमाथण उद्योग को बढ़ािा र्देने के वलए इन िस्तुओं की बढ़ती खपत का समर्थन करने के वलए
कु छ औद्योवगक िस्तुओं की कीमतें तय करना।
Alternatives:
विकल्प:
(A) P, Q and S (B) P, Q and R (C) Q, R and S (D) All of them
इनमे से सभी
26. In India agriculture subsidies have been a topic of debate among the economic 1
thinkers identify, which of the following statement is not an argument in favour of
continuation of the agricultural subsidies in India?
भारत में कृ वष अनुर्दान आर्र्थक विचारकों के मध्य बहस का एक विषय रहा है, वनम्नवलवखत में से
कौन-सा कर्न भारत में कृ वष अनुर्दान जारी रखने के पक्ष में तकथ नहीं है?
(A) Subsidies encourage farmers to adopt new technology.
अनुर्दान ककसानों को नई प्रौद्योवगकी को अपनाने के वलए प्रोत्सावहत करता है |
(B) Subsidies help farmers to face the uncertainties of future.
अनुर्दान ककसान को भविष्य की अवनवितताओं का सामना करने में मर्दर्द करता है |
(C) Subsidies aim to reduce the inequalities between rich and poor farmers.
अनुर्दान का उद्देश्य अमीर और गरीब ककसानों के बीच असमानताओं को कम करना है |
(D) Subsidies mainly benefits the fertilizer industry and big farmers.
अनुर्दान मुख्य रूप से उिथरक उद्योग और बड़े ककसानों को लाभ पहुंचाता है |
27. Identify the correct statement from the following: 1
वनम्नवलवखत में से सही कर्न की पहचान कीवजए:
(A) Restrictive policies of commodity production, trade and tariff pursued by the colonial
government adversely affected the structure, composition and volume of India’s
foreign trade.
औपवनिेवशक सरकार द्वारा अपनाई गई िस्तु उत्पार्दन, व्यापार और प्रशुल्क की प्रवतबंिात्मक
नीवतयों ने भारत के विर्देशी व्यापार की संरचना, संरचना और मात्रा पर प्रवतकू ल प्रभाि डाला।
(B) Restrictive policies of commodity production, trade and tariff pursued by the colonial
government favorable affected the structure, composition and volume of India’s
foreign trade.
औपवनिेवशक सरकार द्वारा अपनाई गई िस्तु उत्पार्दन, व्यापार और प्रशुल्क की प्रवतबंिात्मक
नीवतयों ने भारत के विर्देशी व्यापार की संरचना, संरचना और मात्रा पर अनुकूल प्रभाि डाला।
(C) Effective trade policies of commodity production, trade and tariff pursued by the
colonial government favorable affected the structure, composition and volume of
India’s foreign trade.
औपवनिेवशक सरकार द्वारा अपनाई गई िस्तु उत्पार्दन, व्यापार और प्रशुल्क की प्रभािी व्यापार
नीवतयों ने भारत के विर्देशी व्यापार की संरचना, संरचना और मात्रा पर अनुकूल प्रभाि डाला।
(D) Liberal policies of commodity production, trade and tariff pursued by the colonial
government adversely affected the structure, composition and volume of India’s
foreign trade.
औपवनिेवशक सरकार द्वारा अपनाई गई िस्तु उत्पार्दन, व्यापार और प्रशुल्क की उर्दार नीवतयों ने
भारत के विर्देशी व्यापार की संरचना, संरचना और मात्रा पर प्रवतकू ल प्रभाि डाला।
28. Economics policy of 1991 stressed on removing unnecessary restriction and creating 3
a more competitive environment to increase its interaction with the world economy.
Which values in your opinion have been realized as per New economic policy?
1991 की अर्थशास्त्र नीवत में विश्व अर्थव्यिस्र्ा के सार् संपकथ बढ़ाने के वलए अनािश्यक प्रवतबंि
हटाने और अविक प्रवतस्पिी माहौल बनाने पर जोर कर्दया गया। आपकी राय में नई आर्र्थक नीवत
के अनुसार ककन मूल्यों को साकार ककया गया है?

29.(A) Highlight the role of Human capital formation in the economic growth and 3
development of a country.
ककसी र्देश की आर्र्थक िृवि और विकास में मानि पूंजी वनमाथण की भूवमका पर प्रकाश डावलए |
OR / अर्िा
(B) Highlight the impacts of organic farming in sustainable development process. 3
सतत िारणीय विकास की प्रकिया में जैविक खेती के प्रभािों पर प्रकाश डावलए |
30. Answer the following questions on the basis of the following data:
वनम्नवलवखत आँकड़ों के आिार पर वनम्नवलवखत प्रश्नों के उत्तर र्दीवजए:
Demographic Indicators - 2017-18
जनसांवख्यकीय संकेतक - 2017-18
Country Estimated Annual Density Sex Fertility Urbanization
Population Growth of per sq. km Ratio Ratio
(in million) Population
राष्ट्र संभावित जनसँख्या की घनत्ि बलंग जननक्षमता शहरीकरण
जनसँख्या िार्षथक िृवि प्रवत िगथ कक.मी. अनुपात अनुपात
(र्दस लाख में) र्दर
India 1352 1.03 455 924 2.2 34
भारत
China 1393 0.46 148 949 1.7 59
चीन
Pakistan 212 2.05 275 943 3.6 37
पाककस्तान
I Comment upon the population growth rates among the three countries 2
तीनों राष्ट्रों में जनसंख्या िृवि र्दर पर रटप्पणी कीवजए |
II Which country has less density among the three countries, give appropriate reason? 2
तीनों राष्ट्रों में से ककस राष्ट्र का घनत्ि कम है, उवचत कारण र्दीवजए ?

31.(A) What is seasonal unemployment? Suggest measures for reducing this kind of 1+3=4
unemployment in India.
मौसमी बेरोजगारी क्या है ? भारत में इस प्रकार की बेरोजगारी को कम करने के उपाय सुझाइए |
OR / अर्िा
(B) What do you mean by rural development? Bring out the key issues in rural 1+3=4
development.
ग्रामीण विकास से आप क्या समझते हैं ? ग्रामीण विकास के प्रमुख मुद्दों को उजागर कीवजए |

32. “Recently the government of India has taken a few steps to reach the 6% growth goal
as recommended by the Education Commission, 1964.” Do you agree with the 4
statement give valid reasons in support of your answer?

"हाल ही में भारत सरकार ने वशक्षा आयोग, 1964 द्वारा अनुशवं सत 6% विकास लक्ष्य तक पहुंचने
के वलए कु छ कर्दम उठाए हैं।" क्या आप कर्न से सहमत हैं, अपने उत्तर के समर्थन में िैि कारण
बताइए?

33.(A) Though public sector is very essential for industries, many public sector undertakings 6
incur huge losses and are a drain on the economy’s resources. Discuss the usefulness
of public sector undertakings in the light of this fact.

यद्यवप सािथजवनक क्षेत्र उद्योगों के वलए बहुत आिश्यक है, कई सािथजवनक क्षेत्र के उपिम भारी
घाटे में हैं और अर्थव्यिस्र्ा के संसािनों को बबाथर्द कर रहे हैं। इस तथ्य के आलोक में सािथजवनक
क्षेत्र के उपिमों की उपयोवगता पर चचाथ कीवजए |
OR / अर्िा
(B) Explain how import substitution can protect industry? Why did the policy makers 3
adopt such policy of protection?
बताइए कक आयात प्रवतस्र्ापन ककस प्रकार से उद्योग की रक्षा कर सकता है? नीवत वनमाथताओं ने
संरक्षण की ऐसी नीवत क्यों अपनाई?

(C) Do you think banned of Chinese products and apps will help in the growth of 3
Indigenous products? Establish the rationale of the given statement with valid
reasons.
क्या आपको लगता है कक चीनी उत्पार्दों और ऐप्स पर प्रवतबंि लगाने से स्िर्देशी उत्पार्दों के विकास
में मर्दर्द वमलेगी? िैि कारणों की सहायता से कर्दए गए कर्न का औवचत्य स्र्ावपत कीवजए |

34. Read the following text carefully and answer the given questions on the basis of the
same and common understanding:
The rising population of developing countries, affluent consumption and production
standards has put huge stress on the environment. Many resources have been
become extinct and the wastes generated are beyond ‘Absorptive Capacity’ of the
environment. The population explosion and the advent of industrial revolution has
increased the demand for environmental resources but their supply is limited due to
overuse and misuse. The intensive and extensive extraction of both renewable and
non-renewable resources has exhausted some of the vital resources. Due to this,
huge amount of money is spent on technology and research to explore new
resources.
The development process has polluted the atmosphere and waters and there is
decline in air and water quality. The expenditure on health is also rising. Global
environmental issues such as global warming and ozone depletion also contribute to
the increased financial commitments for the government.

वनम्नवलवखत पररच्छेर्द को ध्यानपूिथक पकढ़ए और कर्दए गए प्रश्नों के उत्तर पररच्छेर्द एिं सामान्द्य
समझ के आिार पर र्दीवजए :
विकासशील र्देशों की बढ़ती जनसंख्या, समृि उपभोग और उत्पार्दन मानकों ने पयाथिरण पर भारी
र्दबाि डाला है। कई संसािन विलुप्त हो गए हैं और उत्पन्न अपवशि पयाथिरण की 'अिशोषण
क्षमता' से परे हैं। जनसंख्या विस्फोट और औद्योवगक िांवत के आगमन से पयाथिरणीय संसािनों
की मांग में िृवि हुई है लेककन अत्यविक उपयोग और र्दुरुपयोग के कारण उनकी आपूर्तथ सीवमत
है। निीकरणीय और गैर-निीकरणीय र्दोनों संसािनों के गहन और व्यापक वनष्कषथण ने कु छ
महत्त्िपूणथ संसािनों को समाप्त कर कर्दया है। इसके कारण, नए संसािनों की खोज के वलए
प्रौद्योवगकी और अनुसंिान पर भारी मात्रा में िन खचथ ककया जाता है।
विकास प्रकिया ने िातािरण और पानी को प्रर्दूवषत कर कर्दया है और हिा एिं पानी की गुणित्ता
में वगरािट आई है। इससे स्िास्थ्य पर खचथ भी बढ़ रहा है | िैवश्वक ताप और ओजोन ररक्तीकरण
जैसे िैवश्वक पयाथिरणीय मुद्दे भी सरकार के वलए बढ़ती वित्तीय प्रवतबिताओं में योगर्दान करते हैं।

(I) Two major environmental issues facing the world today are _____ and ______. ½+½=1
आज सम्पूणथ विश्व के सामने र्दो प्रमुख पयाथिरणीय मुद्दे ____________ और ____________है |

(II) Opportunity cost of negative environment impact are high. Explain. 2


नकारात्मक पयाथिरणीय प्रभाि की अिसर लागत अविक है | िणथन कीवजए |

(III) “Environmental problems are new to this century.” Justify the statement with valid 3
arguments.
“इस सर्दी के वलए पयाथिरणीय समस्याएं नई है” | इस कर्न की िैि तकों की सहायता से पुवि
कीवजए |

********************
Kendriya Vidyalaya Sangthan, Jaipur Region SET-5
के न्द्रीय विद्यालय संगठन, जयपुर सम्भाग
Subject: Economics
विषय: अर्थशास्त्र
Practice Paper
अभ्यास पत्र
Class: XII Time: 3:00 Hours Maximum Marks: 80 Marks
कक्षा: XII समय: 3:00 घंटे अविकतम अंक: 80 अंक
General Instructions:
सामान्द्य वनर्देश:
(1) This question paper contains two sections:
इस प्रश्न पत्र के र्दो खण्ड है :-
Section–A (Introductory Macro Economics) Section–B (Indian Economic Development)
खण्ड-अ (पररचयात्मक समवि अर्थशास्त्र) खण्ड-ब (भारतीय अर्थव्यिस्र्ा का विकास)
(2) All questions in both the sections are compulsory. However, there is internal choice in some
questions. Marks for questions are indicated against each question.
र्दोनों खण्डों के सभी प्रश्न करना अवनिायथ है | यद्यवप कु छ प्रश्नों में आन्द्तररक चयन प्रर्दान ककया गया है | प्रत्येक प्रश्न
के अंक उस प्रश्न के सामने अंककत है |
(3) Question number 1 –10 and 18 – 27 are very short-answer / multiple choice questions carrying
1 mark each. They are required to be answered in one word or one sentence each.
प्रश्न संख्या 1 से 10 एिं 18 से 27 तक के सभी एक-एक अंक के अवत लघूत्तरात्मक / बहुविकल्पीय प्रश्न है | इन
प्रश्नों के उत्तर एक शब्र्द या एक िाक्य में र्दीवजए |
(4) Question number 11 – 12 and 28 – 29 are short-answer questions carrying 3 marks each.
Answers to them should normally not exceed 60-80 words each.
प्रश्न संख्या 11 से 12 एिं 28 से 29 तक के सभी प्रश्न लघूत्तरात्मक प्रश्न है, जो कक तीन-तीन अंक के है | इन प्रश्नों
के उत्तर 60-80 शब्र्दों से अविक नहीं होने चावहए |
(5) Question number 13 – 15 and 30 – 32 are also short-answer questions carrying 4 marks
each. Answers to them should normally not exceed 80-100 words each.
प्रश्न संख्या 13 से 15 एिं 30 से 32 तक के सभी प्रश्न भी लघूत्तरात्मक प्रश्न है, जो कक चार-चार अंक के है | इन
प्रश्नों के उत्तर 80-100 शब्र्दों से अविक नहीं होने चावहए |
(6) Question number 16 – 17 and 33 – 34 are long-answer questions carrying 6 marks each.
Answers to them should normally not exceed 100-150 words each.
प्रश्न संख्या 16 से 17 एिं 33 से 34 तक के सभी प्रश्न र्दीघथ उत्तरात्मक प्रश्न है, जो कक छह-छह अंक के है | इन
प्रश्नों के उत्तर 100-150 शब्र्दों से अविक नहीं होने चावहए |
(7) Answers should be brief and to the point and the above word limits should be adhered to as far
as possible.
सभी प्रश्नों के उत्तर यर्ा सम्भि प्रर्दत्त शब्र्द सीमा को ध्यान में रखते हुए संवक्षप्त, तथ्यात्मक एिं सटीक होने
चावहए |
(8) An additional 15 minutes has been allotted to read the question paper.
इस प्रश्न पत्र को पढ़ने हेतु पंरह (15) वमनट का अवतररक्त समय आिंरटत ककया गया है |
Q. NO. Section-A (Introductory Macro Economics) Marks
प्र. सं. खण्ड-अ (पररचयात्मक समवि अर्थशास्त्र) अंक
1 Mrs. Madhu, an Economics teacher, was explaining ‘the concept of 1
minimum percentage of total deposits to be kept by any commercial
bank with the central bank of country, as per norms and statute
prevailing in the country'. From the following, choose the correct
alternative which specifies towards the concept explained by her.

िीमती मिु, एक अर्थशास्त्र की वशवक्षका, “र्देश में प्रचवलत मानर्दंडों और कानून के अनुसार,
ककसी भी िावणवज्यक बैंक द्वारा र्देश के कें रीय बैंक के पास रखे जाने िाले कु ल जमा के
न्द्यूनतम प्रवतशत की अििारणा को समझा रही र्ीं”। वनम्नवलवखत में से, उस सही विकल्प का
चयन कीवजए जो उनके द्वारा समझाई गई अििारणा को वनर्र्दथि करता है |
(A) Repo Rate (B) Cash Reserve Ratio
रे पो र्दर नकर्द आरवक्षत अनुपात
(C) Bank Rate (D) Statutory Liquidity Ratio
बैंक र्दर िैिावनक तरलता अनुपात

2 Which one of the following saving functions corresponds to the investment 1


multiplier of 5?

वनम्नवलवखत में से कौन-सा बचत फलन 5 के वनिेश गुणक से मेल खाता है?
(A) S = (-) 28 + 0.25Y (B) S = (-) 40 + 0.75Y
(C) S = (-) 75 + 0.30Y (D) S = (-) 60 + 0.20Y

3 A company located in India receives a loan from a company located abroad. 1


How is this transaction recorded in India’s balance of payments account?
(Choose the correct alternative)

भारत में वस्र्त कं पनी को विर्देश में वस्र्त कं पनी से ऋण प्राप्त होता है। यह लेन-र्देन भारत के
भुगतान संतुलन खाते में ककस प्रकार से र्दजथ ककया जायेगा?
(सही विकल्प का चयन कीवजए)
(A) Credit side of current account (B) Debit side of current account
चालू खाते के जमा पक्ष चालू खाते के नामे पक्ष
(C) Credit side of capital account (D) Credit side of capital account
पूंजी खाते के जमा पक्ष पूंजी खाते के जमा पक्ष

4 The relation between Cash Reserve Ratio, Statutory Liquidity Ratio and 1
Legal Reserve Ratio is define as__________________.

नकर्द आरवक्षत अनुपात, िैिावनक तरलता अनुपात और विविक आरवक्षत अनुपात के मध्य
सम्बन्द्िों को ________________ के रूप में पररभावषत ककया गया है।
(A) Legal Reserve Ratio is the product of Cash Reserve Ratio and Statutory
Liquidity Ratio.
विविक आरवक्षत अनुपात, नकर्द आरवक्षत अनुपात और िैिावनक तरलता अनुपात का उत्पार्द है।
(B) Legal Reserve Ratio is the ratio of Cash Reserve Ratio and Statutory Liquidity
Ratio.
विविक आरवक्षत अनुपात, नकर्द आरवक्षत अनुपात और िैिावनक तरलता अनुपात का अनुपात
है।
(C) Legal Reserve Ratio is the difference of Cash Reserve Ratio and Statutory
Liquidity Ratio.
विविक आरवक्षत अनुपात, नकर्द आरवक्षत अनुपात और िैिावनक तरलता अनुपात का अंतर है।
(D) Legal Reserve Ratio is the sum of Cash Reserve Ratio and Statutory Liquidity
Ratio.
विविक आरवक्षत अनुपात, नकर्द आरवक्षत अनुपात और िैिावनक तरलता अनुपात का योग है |

5 Read the following Assertion (A) and Reason (R). 1


Choose the correct alternative.
वनम्नवलवखत अवभकर्न (A) और कारण (R) को ध्यानपूिथक पकढ़ए |
वनम्नवलवखत विकल्पों में से एक सही विकल्प का चयन कीवजए |
Assertion (A): Managed Floating exchange rate system is also called as
'Dirty Floating'.
अवभकर्न (A): प्रबंवित फ्लोटटंग विवनमय र्दर प्रणाली को 'डटी फ्लोटटंग' भी कहा जाता है।

Reason (R): The Managed Floating rate is influenced by the intervention of


the Central Bank in the Foreign Exchange Market.
कारण (R): प्रबंवित फ्लोटटंग र्दर विर्देशी मुरा बाजार में कें रीय बैंक के हस्तक्षेप से प्रभावित
होती है।
Alternatives:
विकल्प:
(A) Both Assertion (A) and Reason (R) are true and Reason (R) is the correct
explanation of Assertion (A).
अवभकर्न (A) और कारण (R) र्दोनों सत्य हैं और कारण (R) अवभकर्न (A) का सही
स्पिीकरण है।
(B) Both Assertion (A) and Reason (R) are true and Reason (R) is not the correct
explanation of Assertion (A).
अवभकर्न (A) और कारण (R) र्दोनों सत्य हैं और कारण (R) अवभकर्न (A) का सही
स्पिीकरण नहीं है।
(C) Assertion (A) is true but Reason (R) is false.
अवभकर्न (A) सत्य है लेककन कारण (R) गलत है।
(D) Assertion (A) is false but Reason (R) is true.
अवभकर्न (A) गलत है लेककन कारण (R) सत्य है।

6 Read the following statements carefully. 1


वनम्नवलवखत कर्नों को ध्यानपूिथक पकढ़ए |
Statement-1: When investment multiplier is 1, the value of MPC is zero.
कर्न-1: जब वनिेश गुणक 1 होता है, तो एमपीसी का मूल्य शून्द्य होता है।

Statement-2: When investment multiplier is zero, the value of MPC is one.


कर्न-2: जब वनिेश गुणक शून्द्य होता है, तो एमपीसी का मूल्य एक होता है।
In light of the given statements, choose the correct alternative from the
following:
कर्दए गए कर्नों के आलोक में वनम्नवलवखत में से सही विकल्प का चयन कीवजए:
(A) Statement-1 is true and Statement-2 is false.
कर्न 1 सत्य है और कर्न 2 गलत है |
(B) Statement-1 is false and Statement-2 is true.
कर्न 1 गलत है और कर्न 2 सत्य है |
(C) Both Statement-1 and Statement-2 is true.
कर्न 1 और कर्न 2 र्दोनों सत्य हैं |
(D) Both Statement-1 and Statement-2 is false.
कर्न 1 और कर्न 2 र्दोनों गलत हैं |

7 Which of the following will lead to an appreciation of domestic currency? 1


वनम्नवलवखत में से कौन-सा घरे लू मुरा को अविमूल्यन की ओर अग्रवसत करे गा?
(P) Increased demand for foreign exchange
विर्देशी मुरा की बढ़ी हुई मांग
(Q) Decreased demand for foreign exchange
विर्देशी मुरा की कम हुई मांग
(R) Increased supply for foreign exchange
विर्देशी मुरा के वलए आपूर्तथ में िृवि
(S) Decreased supply for foreign exchange
विर्देशी मुरा के वलए आपूर्तथ में कमी
Alternatives:
विकल्प:
(A) P and R (B) P and S (C) Q and R (D) Q and S
8 If factor cost is > market price, then it means that : 1
यकर्द कारक लागत > बाजार मूल्य है, तब इसका अर्थ है कक:
(A) Indirect Taxes > Subsidies (B) Indirect Taxes = Subsidies
अप्रत्यक्ष कर > अनुर्दान अप्रत्यक्ष कर = अनुर्दान
(C) Indirect Taxes < Subsidies (D) Indirect Taxes ≥ Subsidies
अप्रत्यक्ष कर < अनुर्दान अप्रत्यक्ष कर ≥ अनुर्दान

9 Read the following Assertion (A) and Reason (R). 1


Choose the correct alternative.
वनम्नवलवखत अवभकर्न (A) और कारण (R) को ध्यानपूिथक पकढ़ए |
वनम्नवलवखत विकल्पों में से एक सही विकल्प का चयन कीवजए |
Assertion (A): Increase in reverse repo rate helps to correct the excess
demand.
अवभकर्न (A): ररिसथ रे पो र्दर में िृवि से अवतररक्त मांग को ठीक करने में मर्दर्द वमलती है।

Reason (R): The Credit creation capacity of the commercial banks can be
increased by raising the reverse repo rate.
कारण (R): ररिसथ रे पो र्दर बढ़ाकर िावणवज्यक बैंकों की साख सजथन क्षमता बढ़ाई जा सकती है|
Alternatives:
विकल्प:
(A) Both Assertion (A) and Reason (R) are true and Reason (R) is the correct
explanation of Assertion (A).
अवभकर्न (A) और कारण (R) र्दोनों सत्य हैं और कारण (R) अवभकर्न (A) का सही
स्पिीकरण है।
(B) Both Assertion (A) and Reason (R) are true and Reason (R) is not the correct
explanation of Assertion (A).
अवभकर्न (A) और कारण (R) र्दोनों सत्य हैं और कारण (R) अवभकर्न (A) का सही
स्पिीकरण नहीं है।
(C) Assertion (A) is true but Reason (R) is false.
अवभकर्न (A) सत्य है लेककन कारण (R) गलत है।
(D) Assertion (A) is false but Reason (R) is true.
अवभकर्न (A) गलत है लेककन कारण (R) सत्य है।

10 Welfare of the people in an economy is measured in terms of ______________. 1


ककसी अर्थव्यिस्र्ा में लोगों का कल्याण ______________ के संर्दभथ में मापा जाता है |
(A) Rise in National Income
राष्ट्रीय आय में िृवि
(B) Availability of goods and services for the rich per person
अमीरों के वलए प्रवत व्यवक्त िस्तुओं और सेिाओं की उपलब्िता
(C) Overall price and degree of profit maximisation
कु ल मूल्य और लाभ अविकतमकरण की वडग्री
(D) Overall price level in an economy.
एक अर्थव्यिस्र्ा में समग्र मूल्य स्तर।

11 Recently Government of India has doubled the import duty on gold. What 1+1+1
impact is it likely to have on foreign exchange rate, import-export and how?

हाल ही में भारत सरकार ने सोने पर आयात शुल्क र्दोगुना कर कर्दया है। इसका विर्देशी
विवनमय र्दर, आयात- वनयाथत पर क्या प्रभाि पड़ने की संभािना है और कै से?

12. (A) Giving reasons explain the treatment assigned to the following while
estimating National Income of India.
भारत की राष्ट्रीय आय का अनुमान लगाते समय वनम्नवलवखत के सार् ककए जाने व्यिहार का
कारण बताते हुए िणथन कीवजए |
(i) Interest paid by banks on deposits by individuals. 1
व्यवक्तयों द्वारा जमा रावश पर बैंकों द्वारा भुगतान ककए जाने िाला ब्याज।
(ii) Payment of interest by an individual to a bank 1
ककसी व्यवक्त द्वारा बैंक को ब्याज का भुगतान
(iii) National debt interest 1
राष्ट्रीय ऋण का ब्याज
OR / अर्िा
(B) How will you treat the following in the calculation of National Income? Give
reasons for your answer.
आप राष्ट्रीय आय की गणना करते समय वनम्नवलवखत के सार् ककस प्रकार का व्यिहार करें गे ?
अपने उत्तर के वलए कारण र्दीवजए |
(i) Wheat grown by a farmer but used entirely for family’s consumption. 1½
ककसान द्वारा पूणथ रूप से अपने पररिार के उपभोग हेतु उगाया गया गेहँ |

(ii) Rent- free house given to an employee by an employer. 1½


अपने कमथचारी को वनयोक्ता द्वारा ककराया-मुक्त घर प्रर्दान करना |

13. If in an economy, Consumption Function is given by C = 100 + 0.75Y and


autonomous investment = ₹150. Calculate:
यकर्द ककसी अर्थव्यिस्र्ा में, उपभोग फलन C = 100 + 0.75Y और स्िायत्त वनिेश = ₹ 150 है।
गणना कीवजए :
(i) Equilibrium level of income 2
आय का संतुलन स्तर
(ii) Consumption and savings at equilibrium level of income 2
आय के संतुलन स्तर पर उपभोग और बचत

14. Explain the “Banker’s bank” function of a central bank. 4


कें रीय बैंक के ‘बैंको के बैंक' कायथ की व्याख्या कीवजए ।

15. (A) ‘Monetary measures offer a valid solution to the in a problem of 2+2=4
inflationary gap economy'. State and discuss any two monetary measures
to justify the given statement.

“मौकरक उपाय, स्फीवत अन्द्तराल अर्थव्यिस्र्ा की समस्या का एक िैि समािान प्रर्दान करते
हैं ”। कर्दए गए कर्न को उवचत ठहराने हेतु ककन्द्हीं र्दो मौकरक उपायों को बताइए और उन पर
चचाथ कीवजए ।
OR / अर्िा
(B) 'An economy is operating at under-employment level of income'. What is 2+2=4
meant by the given statement? Explain it. Discuss any one fiscal measure to
tackle the situation.

'एक अर्थव्यिस्र्ा आय के अल्प-रोज़गार स्तर पर चल रही है।' कर्दए गए कर्न का क्या अर्थ है?
इसका िणथन कीवजए | इस प्रकार की वस्र्वत से वनपटने के वलए ककसी एक राजकोषीय उपाय
की चचाथ कीवजए ।

16. (i) The government under Uijwala Yojna, is providing free LPG gas connection to 3
the families 'below the poverty line'. What objective is government trying
to fulfil through the government budget and how? Explain it.
सरकार उज्िला योजना के तहत ‘गरीबी रे खा से नीचे' रहने िाले पररिारों को मुफ्त एलपीजी
गैस उपलब्ि करा रही है | इससे सरकार द्वारा सरकारी बजट से ककस उद्देश्य को पूरा करने
की कोवशश की जा रही है और कै से? इसका िणथन कीवजए ।

(ii) Two friends, Deepak and Ranjeet were discussing the impact of increase in 3
Goods and Service Tax (GST) rates on luxury items, as recently undertaken
by the Government. Ranjeet was of the view that most of the luxury items
(like foreign travel, imported cigarettes, etc.) should be taxed exorbitantly,
while the items related to daily consumption of poor and middle class should
be tax-free. Identify and explain the objective of the Government budget
Ranjeet is suggesting.

र्दो र्दोस्त, र्दीपक और रणजीत, हाल ही में सरकार द्वारा विलावसता की िस्तुओं पर िस्तु एिं
सेिा कर (जीएसटी) की र्दरों में की गई िृवि के प्रभािों पर चचाथ कर रहे र्े। रणजीत का
विचार र्ा कक अविकांश विलावसता की िस्तुओं (जैसे विर्देश यात्रा, आयावतत वसगरे ट आकर्द)
पर अत्यविक कर लगाया जाना चावहए, जबकक गरीब और मध्यम िगथ की र्दैवनक उपभोग से
संबंवित िस्तुओं को कर-मुक्त ककया जाना चावहए। रणजीत द्वारा वजस सरकारी बजट के
उर्दर्देश्य का सुझाि कर्दया जा रहा हैं, उसकी पहचान कीवजए एिं िणथन कीवजए ।

17. (A) From the following data, show that the National Income will be same from 3+3=6
both Income method and Expenditure method.

वनम्नवलवखत आँकड़ों से, र्दशाथइए कक आय विवि एिं व्यय विवि र्दोनों से राष्ट्रीय आय समान
होगी।
SR. NO. PARTICULARS AMOUNT (₹ In crore)
ि. स. वििरण रावश (₹ करोड़ में)
1 Net Exports
(-) 60
शुि वनयाथत
2 Net Indirect tax
200
शुि अप्रत्यक्ष कर
3 Operating surplus
740
पररचालन अविशेष
4 Compensation of employees
1400
कमथचाररयों को भुगतान
5 Net factor income from abroad
40
विर्देश से शुि कारक आय
6 Mixed income of self employed
1000
स्ि-रोज़गार की वमवित आय
7 Net Domestic Fixed Capital formation
500
शुि घरे लू वस्र्र पूंजी वनमाथण
8 Change in stock
(-)100
स्टॉक में पररितथन
9 Depreciation 100
मूल्यह्रास
10 Private final consumption expenditure 2000
वनजी अंवतम उपभोग व्यय
11 Govt. final consumption expenditure 1000
सरकार का अंवतम उपभोग व्यय
OR / अर्िा
(B) (i) How Nominal GDP is different from Real GDP? Which one is better and why? 3

सांकेवतक जीडीपी ककस प्रकार से िास्तविक जीडीपी से वभन्न है? इनमे से कौन-सी बेहतर है
और क्यों?
(ii) Briefly explain the problem of Double counting with the help of an Example. 3
How can it be avoided?

एक उर्दाहरण की सहायता से र्दोहरी गणना की समस्या को संवक्षप्त में समझाइए। इसे ककस
प्रकार से टाला जा सकता है?

SECTION–B (Indian Economic Development)


खण्ड-ब (भारतीय अर्थव्यिस्र्ा का विकास)
18. Read the following Assertion (A) and Reason (R). 1
Choose the correct alternative.
वनम्नवलवखत अवभकर्न (A) और कारण (R) को ध्यानपूिथक पकढ़ए |
वनम्नवलवखत विकल्पों में से एक सही विकल्प का चयन कीवजए |
Assertion (A): Industry provides employment which is more stable than the
employment in agriculture. It promotes modernisation and overall
prosperity.
अवभकर्न (A): उद्योग द्वारा प्रर्दत्त रोजगार कृ वष में रोजगार की तुलना में अविक वस्र्र है।
यह आिुवनकीकरण और समग्र समृवि को बढ़ािा र्देता है।

Reason (R): Economists have found that poor nations can progress if they
have a good industrial sector.
कारण (R): अर्थशावस्त्रयों ने पाया है कक गरीब राष्ट्र तभी प्रगवत कर सकते हैं जब उनके पास
एक अच्छा औद्योवगक क्षेत्र हो।
Alternatives:
विकल्प:
(A) Both Assertion (A) and Reason (R) are true and Reason (R) is the correct
explanation of Assertion (A).
अवभकर्न (A) और कारण (R) र्दोनों सत्य हैं और कारण (R) अवभकर्न (A) का सही
स्पिीकरण है।
(B) Both Assertion (A) and Reason (R) are true and Reason (R) is not the correct
explanation of Assertion (A).
अवभकर्न (A) और कारण (R) र्दोनों सत्य हैं और कारण (R) अवभकर्न (A) का सही
स्पिीकरण नहीं है।
(C) Assertion (A) is true but Reason (R) is false.
अवभकर्न (A) सत्य है लेककन कारण (R) गलत है।
(D) Assertion (A) is false but Reason (R) is true.
अवभकर्न (A) गलत है लेककन कारण (R) सत्य है।

19. Economists have stressed the need for expanding educational opportunities 1
in a nation as it accelerates the development process. Which of the following
factor justify the above findings?

अर्थशावस्त्रयों ने ककसी राष्ट्र में शैवक्षक अिसरों के विस्तार की आिश्यकता पर बल कर्दया है


क्योंकक यह विकास प्रकिया को गवत र्देता है। वनम्नवलवखत में से कौन-सा कारक उपरोक्त
वनष्कषथ को उवचत ठहराता है?

(A) Education confers higher earning capacity on people.


वशक्षा लोगों को उच्च कमाई की क्षमता प्रर्दान करती है |
(B) It gives one a better social standing and pride and enables one to make
better choices in life.
यह व्यवक्त को बेहतर सामावजक प्रवतष्ठा और गौरि प्रर्दान करता है और जीिन में बेहतर
विकल्प के चयन में सक्षम बनाता है |
(C) It provides knowledge to understand the changes taking place in society; it
also stimulates innovations.
यह समाज में हो रहे पररितथनों को समझने के वलए ज्ञान प्रर्दान करता है; यह निाचारों को
भी प्रोत्सावहत करता है।
(D) All of the above
उपरोक्त सभी

20. Deepti is a house lady but knows the skill of tailoring. Her brother has a cloth 1
shop. Customers buy cloth materials from her brother’s shop and get it
stitched from Deepti who is assisting her brother in the business. Can she be
considered as a worker?

र्दीवप्त एक घरे लू मवहला हैं लेककन वसलाई का हुनर जानती हैं। उसके भाई की कपड़े की र्दुकान
है। ग्राहक उसके भाई की र्दुकान से कपड़े की सामग्री खरीर्दते हैं और उसे र्दीवप्त से वसलिाते हैं
जो कक व्यिसाय में अपने भाई की सहायता कर रही है। क्या उसको एक िवमक के रूप में
माना जा सकता है?
(A) Yes (B) No
हाँ नहीं
(C) Both (A) and (B) (D) None of these
(A) और (B) र्दोनों इनमे से कोई नहीं

21. Arrange the following in chronological order and choose the correct 1
alternative:
वनम्नवलवखत को कालानुिवमक िम में व्यिवस्र्त कीवजए और सही विकल्प का चयन कीवजए:
(i) Year of great divide
महान विभाजन का िषथ
(ii) Establishment of Tata Iron and steel company
टाटा आयरन एिं स्टील कं पनी की स्र्ापना
(iii) Introduction of Railways in India by British
अंग्रेजों द्वारा भारत में रे लिे की शुरूआत
(iv) Opening of Suez Canal
स्िेज नहर का खुलना
Alternatives:
विकल्प:
(A) (iv), (ii), (i), (iii) (B) (i), (iv), (iii), (ii)
(C) (ii), (iii), (iv), (i) (D) (iii), (iv), (ii), (i)

22. Read the following Assertion (A) and Reason (R). 1


Choose the correct alternative.
वनम्नवलवखत अवभकर्न (A) और कारण (R) को ध्यानपूिथक पकढ़ए |
वनम्नवलवखत विकल्पों में से एक सही विकल्प का चयन कीवजए |
Assertion (A): Organic farming generates more employment opportunities
than conventional farming.
अवभकर्न (A): जैविक खेती पारं पररक खेती की तुलना में अविक रोजगार के अिसर उत्पन्न
करती है।

Reason (R): The production of organic farming is pesticide-free.


कारण (R): जैविक खेती का उत्पार्द कीटनाशक से मुक्त है।
Alternatives:
विकल्प:
(A) Both Assertion (A) and Reason (R) are true and Reason (R) is the correct
explanation of Assertion (A).
अवभकर्न (A) और कारण (R) र्दोनों सत्य हैं और कारण (R) अवभकर्न (A) का सही
स्पिीकरण है।
(B) Both Assertion (A) and Reason (R) are true and Reason (R) is not the correct
explanation of Assertion (A).
अवभकर्न (A) और कारण (R) र्दोनों सत्य हैं और कारण (R) अवभकर्न (A) का सही
स्पिीकरण नहीं है।
(C) Assertion (A) is true but Reason (R) is false.
अवभकर्न (A) सत्य है लेककन कारण (R) गलत है।
(D) Assertion (A) is false but Reason (R) is true.
अवभकर्न (A) गलत है लेककन कारण (R) सत्य है।

23. Which of the following statements gives the correct meaning of absorptive 1
capacity?
वनम्नवलवखत में से कौन-सा कर्न अिशोषण क्षमता का सही अर्थ प्रस्तुत करता है?
(A) Absorptive capacity is defined as the ability of our environment to absorb
degradation.
अिशोषण क्षमता को हमारे पयाथिरण की वगरािट को अिशोवषत करने की क्षमता के रूप में
पररभावषत ककया गया है |
(B) Absorptive capacity is defined as the method of resource generation that is
within the assimilating capacity of our environment.
अिशोषण क्षमता को संसािन उत्पार्दन की विवि के रूप में पररभावषत ककया गया है जो
हमारे पयाथिरण की आत्मसात क्षमता के भीतर है |
(C) Absorptive capacity is defined as the average number of organisms that are
living within a given environment.
अिशोषण क्षमता को ककसी कर्दए गए िातािरण में रहने िाले जीिों की औसत संख्या के रूप
में पररभावषत ककया गया है |
(D) Absorptive capacity is defined as the minimum population that is currently
surviving in the world.
अिशोषण क्षमता को उस न्द्यूनतम जनसंख्या के रूप में पररभावषत ककया गया है जो ितथमान
में विश्व में जीवित है |

24. Identify the issue depicted in the given image. 1


प्रर्दर्शथत ककए गए छायावचत्र में र्दशाथए गए मुद्दे की पहचान कीवजए |

(A) Demographic dividend (B) Exponential population growth


जनसांवख्यकीय लाभांश घातीय जनसंख्या िृवि
(C) Aging of population (D) None of these
जनसंख्या की उम्र बढ़ना इनमें से कोई नहीं

25. Which of the following is a means by which countries are trying to 1


strengthen their own domestic economies?
वनम्नवलवखत में से कौन-सा एक सािन है वजसके द्वारा र्देश अपनी घरे लू अर्थव्यिस्र्ा को
मजबूत करने का प्रयास कर रहे हैं।

(A) Formation of regional and economic groups. Such as SAARC, G-8, G-21 etc.
क्षेत्रीय और आर्र्थक समूहों का गठन। जैसे की साकथ , जी-8, जी-21 आकर्द।
(B) Understanding the development per process pursued by their neighbouring
nations.
अपने पड़ोसी र्देशों द्वारा अपनाई गई विकास की प्रकिया को समझना।
(C) Both A and B
A और B र्दोनों।
(D) Comprehend their own strength and weakness with vis a vis their neighbours.
अपने पड़ोवसयों के सार् उनकी ताकत और कमजोरी को सवम्मवलत करना ।

26. Micro credit programmes play a crucial role in the development of rural 1
economy because:
सूक्ष्म ऋण कायथिम ग्रामीण अर्थव्यिस्र्ा के विकास में महत्त्िपूणथ भूवमका वनभाते हैं क्योंकक:

(i) They help in women empowerment.


िे मवहला सशवक्तकरण में मर्दर्द करते हैं |
(ii) They increase the reach of formal credit system.
िे औपचाररक ऋण प्रणाली की पहुंच को बढ़ाते हैं |
(iii) Credit is made available at concessional rate of interest.
ररयायती ब्याज र्दर पर ऋण उपलब्ि कराया जाता है |
Alternatives:
विकल्प:
(A) (i) and (ii) (B) (i), (ii) and (iii)
(C) (i) and (iii) (D) Only (i)

27. Arrange the following events of China in Chronological order: 1


चीन की वनम्नवलवखत घटनाओं को कालानुिवमक िम में व्यिवस्र्त कीवजए:
(i) Special Economic Zones
विशेष आर्र्थक क्षेत्र
(ii) Great Proletarian Cultural Revolution
महान सिथहारा सांस्कृ वतक िांवत
(iii) Commune System
कम्यून प्रणाली
(iv) Establishment of People‘s Republic of China
पीपुल्स ररपवब्लक ऑफ चाइना की स्र्ापना
Choose the Correct Alternatives:
सही विकल्प का चयन कीवजए:
(A) (ii),(iv),(iii),(i) (B) (iv),(iii),(ii),(i)
(C) (ii),(iv),(i),(iii) (D) (iv),(i),(ii),(iii)

28. ‘Amaazon.com, a vast internet-based enterprise has been outsourcing 1+1+1


to various customer support companies in India to accommodate more
local and international buyers and sellers.’ Considering the above
statement, how has the process of globalisation impacted the Indian
economy?

'अमेज़न. कॉम, एक विशाल इं टरनेट-आिाररत उद्यम है, जो कक अविक मात्रा में स्र्ानीय और
अंतराथष्ट्रीय खरीर्दारों और वििे ताओं को समायोवजत करने के वलए भारत में विवभन्न ग्राहक
सहायता कं पवनयों को आउटसोर्सिंग कर रहा है।' उपरोक्त कर्न पर विचार करते हुए,
िैश्वीकरण की प्रकिया ने भारतीय अर्थव्यिस्र्ा को ककस प्रकार से प्रभावित ककया है?

29.(A)(i) What are co-operative credit societies? 1½


सहकारी ऋण सवमवतयाँ क्या हैं?

(ii) Discuss in brief about non-institutional sources of rural credit. 1½


ग्रामीण ऋण के गैर-संस्र्ागत स्रोतों के बारे में संवक्षप्त में चचाथ कीवजए ।
OR / अर्िा
(B) A farmer has four acres of land and he actually needs only two workers and
himself to carry out various operations on his farm in a year, but if he
employs five workers and his family members such as his wife and children.
एक ककसान के पास चार एकड़ भूवम है और िास्ति में उसे एक िषथ में अपनी भूवम पर
विवभन्न कायों को पूरा करने के वलए स्ियं की और के िल र्दो ही िवमकों आिश्यकता होती है,
लेककन यकर्द िह पांच िवमकों और अपने पररिार के सर्दस्यों जैसे की अपनी पत्नी और बच्चों को
कायथ के वलए वनयोवजत करता है ।
(i) This type of situation is known as ____________. 1
इस प्रकार की वस्र्वत को ____________ के रूप में जाना जाता है।
(ii) What will be the marginal productivity of the three extra people? 1
तीन अवतररक्त व्यवक्तयों की सीमांत उत्पार्दकता क्या होगी?
(iii) Explain the different ways to tackle this type of problem. 1
इस प्रकार की समस्या से वनपटने के विवभन्न तरीकों की व्याख्या कीवजए |

30. Interpret the given picture on account of the role of human capital formation 4
enhancing productivity of physical capital.

Source: NCERT
भौवतक पूंजी की उत्पार्दकता बढ़ाने में मानि पूंजी वनमाथण की भूवमका के आिार पर कर्दए गए
वचत्र की व्याख्या कीवजए ।

सौजन्द्य से : एन सी ई आर टी
31. Compare and analyse the given data with valid reasons. 4
कर्दए गए आंकड़ों की िैि कारणों के सार् तुलना और विश्लेषण कीवजए ।
Distribution of workforce (in 2018-19)
कायथबल का वितरण (2018-19 में)
Country Agriculture (%) Industry (%) Services (%)
र्देश कृ वष (%) उद्योग (%) सेिाएँ (%)
India
43 25 32
भारत
China
26 28 46
चीन
Pakistan
41 24 35
पाककस्तान
Source: NCERT
सौजन्द्य से : एन सी ई आर टी
32. (A) “Regulation of Agricultural Marketing is pre-requisite for rural 4
development“. Illustrate.
"कृ वष विपणन की विवनयमन की ग्रामीण विकास के वलए पूिथ-आिश्यकता है"| स्पि कीवजए।
OR / अर्िा
(B) ‘The challenge is to build cost effective Information Technology (IT) 4
based system to improve the living standard of Indian rural
population.’ Justify the statement.
'भारतीय ग्रामीण आबार्दी के जीिन स्तर में सुिार के वलए लागत प्रभािी सूचना
प्रौद्योवगकी (आई.टी.) आिाररत प्रणाली का वनमाथण करना एक चुनौती है।' कर्न का
औवचत्य वसि कीवजए |

33. (A) What do you mean by demographic transition? Relate it with the year great 6
divide and small divide. Throw a light on quantitative appraisal of India’s
demographic profile during colonial period.
जनसांवख्यकीय पररितथन से आप क्या समझते हैं? इसे िषथ िृहत विभाजन और लघु विभाजन
से संबि कीवजए । औपवनिेवशक काल के र्दौरान भारत की जनसांवख्यकीय रूपरे खा के
मात्रात्मक मूल्यांकन पर प्रकाश डावलए ।
OR / अर्िा
(B) “We have everything by globalization, we have noting by globalization”. 6
Explain.
"िैश्वीकरण से हमारे पास सब कु छ है, िैश्वीकरण से ही हमारे पास कु छ नहीं है।" व्याख्या
कीवजए |

34. Read the following passage on the causes of the environmental crisis:
The world is facing an unprecedented environmental crisis characterized by
various interconnected challenges. Several factors have contributed to this
alarming situation, threatening the delicate balance of ecosystems and the
well-being of both humans and wildlife. One of the primary causes of the
environmental crisis is the rapid growth of human population. The
increasing demand for resources, food, and energy has led to extensive
deforestation, habitat destruction, and overconsumption of natural
resources. Industrialization and modernization have played a significant role
in exacerbating the environmental crisis. The reliance on fossil fuels and the
emission of greenhouse gases have resulted in global warming and climate
change, leading to extreme weather events, rising sea levels, and disruptions
in natural cycles. The unchecked discharge of pollutants and waste from
industries has further contaminated air, water, and soil, endangering human
health and biodiversity. Another critical factor contributing to the
environmental crisis is the expansion of agriculture and urbanization. The
conversion of forests and natural habitats into agricultural lands and urban
areas has led to the loss of biodiversity and fragmentation of ecosystems.
This has disrupted the natural habitats of numerous species, leading to the
extinction of many plants and animal species. Moreover, human activities
such as irresponsible waste management and improper disposal of plastics
have resulted in the accumulation of plastic waste in oceans and landfills,
causing harm to marine life and contaminating the environment. The
excessive use of chemical fertilizers and pesticides in agriculture has also led
to soil degradation, water pollution, and loss of biodiversity. In summary, the
environmental crisis is a complex issue with various interconnected causes.
The rapid growth of human population, industrialization, urbanization, and
irresponsible waste management practices are some of the major
contributors to this crisis. Addressing these causes requires a collective
effort from governments, industries, and individuals to adopt sustainable
practices and promote environmental conservation.
पयाथिरणीय संकट के कारणों पर आिाररत वनम्नवलवखत पररच्छेर्द को पकढ़ए :
र्दुवनया एक अभूतपूिथ पयाथिरणीय संकट का सामना कर रही है, वजसमें विवभन्न परस्पर जुड़ी
चुनौवतयाँ शावमल हैं। इस बचंताजनक वस्र्वत में कई कारकों ने योगर्दान कर्दया है, वजससे
पाररवस्र्वतक तंत्र के नाजुक संतलु न और मनुष्यों और िन्द्यजीिों र्दोनों की भलाई को खतरा है।
पयाथिरण संकट का एक प्रमुख कारण मानि जनसंख्या की तीव्र िृवि है। संसािनों, भोजन और
ऊजाथ की बढ़ती मांग के कारण व्यापक िनों की कटाई, आिास विनाश और प्राकृ वतक संसािनों
की अत्यविक खपत हुई है। औद्योगीकरण और आिुवनकीकरण ने पयाथिरण संकट को बढ़ाने में
महत्त्िपूणथ भूवमका वनभाई है। जीिाश्म ईंिन पर वनभथरता और ग्रीनहाउस गैसों के उत्सजथन के
पररणामस्िरूप िैवश्वक ताप और जलिायु पररितथन हुआ है, वजससे चरम मौसम की घटनाएं,
समुर के स्तर का बढ़ना और प्राकृ वतक चिों में व्यििान उत्पन्न हुआ है। उद्योगों से प्रर्दूषकों
और अपवशिों के अवनयंवत्रत वनिथहन ने हिा, पानी एिं वमट्टी को और अविक प्रर्दूवषत कर
कर्दया है, वजससे मानि स्िास्थ्य और जैि विवििता खतरे में पड़ गई है। पयाथिरणीय संकट में
योगर्दान र्देने िाला एक अन्द्य महत्त्िपूणथ कारक कृ वष और शहरीकरण का विस्तार है। िनों और
प्राकृ वतक आिासों को कृ वष भूवम और शहरी क्षेत्रों में बर्दलने से जैि विवििता का नुकसान हुआ
है और पाररवस्र्वतक तंत्र का विखंडन हुआ है। इससे कई प्रजावतयों के प्राकृ वतक आिास बावित
हो गए हैं, वजससे कई पौिों और जानिरों की प्रजावतयां विलुप्त हो गईं। इसके अलािा, गैर-
वजम्मेर्दार अपवशि प्रबंिन और प्लावस्टक के अनुवचत वनपटान जैसी मानिीय गवतविवियों के
पररणामस्िरूप महासागरों और गड्ढों की गहराई में प्लावस्टक कचरा जमा हो गया है, वजससे
समुरी जीिन को नुकसान हो रहा है और पयाथिरण र्दूवषत हो रहा है। कृ वष में रासायवनक
उिथरकों और कीटनाशकों के अत्यविक उपयोग से भी वमट्टी का क्षरण, जल प्रर्दूषण और जैि
विवििता की हावन हुई है। संक्षेप में, पयाथिरण संकट विवभन्न परस्पर जुड़े कारणों के सार् एक
जरटल मुद्दा है। मानि जनसंख्या की तीव्र िृवि, औद्योगीकरण, शहरीकरण और गैर-वजम्मेर्दार
अपवशि प्रबंिन प्रर्ाएँ इस संकट के कु छ प्रमुख योगर्दानकताथ हैं। इन कारणों को संबोवित
करने के वलए रटकाऊ प्रर्ाओं को अपनाने और पयाथिरण संरक्षण को बढ़ािा र्देने के वलए
सरकारों, उद्योगों और व्यवक्तयों के सामूवहक प्रयास की आिश्यकता है।

On the basis of the given text and common understanding, answer the
following question:
वनम्नवलवखत प्रश्नों के उत्तर उपरोक्त पररच्छेर्द एिं सामान्द्य समझ के आिार पर र्दीवजए :

(i) What are the primary causes of the environmental crisis described in the 2
passage?
पयाथिरणीय संकट के पररच्छेर्द में िर्णथत प्रार्वमक कारण क्या हैं?

(ii) How has industrialization contributed to the environmental crisis? 2


औद्योगीकरण ने पयाथिरण संकट में ककस प्रकार से योगर्दान कर्दया है?

(iii) Explain the impact of agriculture and urbanization on the environment. 2


पयाथिरण पर कृ वष और शहरीकरण के प्रभािों की व्याख्या कीवजए |
**************************************
Kendriya Vidyalaya Sangthan, Jaipur Region
SET-6
के न्द्रीय विद्यालय संगठन, जयपुर सम्भाग
Subject: Economics
विषय: अर्थशास्त्र
Practice Paper
अभ्यास पत्र
Class: XII Time: 3:00 Hours Maximum Marks: 80 Marks
कक्षा: XII समय: 3:00 घंटे अविकतम अंक: 80 अंक
General Instructions:
सामान्द्य वनर्देश:
(1) This question paper contains two sections:
इस प्रश्न पत्र के र्दो खण्ड है :-
Section–A (Introductory Macro Economics) Section–B (Indian Economic Development)
खण्ड-अ (पररचयात्मक समवि अर्थशास्त्र) खण्ड-ब (भारतीय अर्थव्यिस्र्ा का विकास)
(2) All questions in both the sections are compulsory. However, there is internal choice in some questions.
Marks for questions are indicated against each question.
र्दोनों खण्डों के सभी प्रश्न करना अवनिायथ है | यद्यवप कु छ प्रश्नों में आन्द्तररक चयन प्रर्दान ककया गया है | प्रत्येक प्रश्न के
अंक उस प्रश्न के सामने अंककत है |
(3) Question number 1 –10 and 18 – 27 are very short-answer / multiple choice questions carrying 1 mark
each. They are required to be answered in one word or one sentence each.
प्रश्न संख्या 1 से 10 एिं 18 से 27 तक के सभी एक-एक अंक के अवत लघूत्तरात्मक / बहुविकल्पीय प्रश्न है | इन प्रश्नों के
उत्तर एक शब्र्द या एक िाक्य में र्दीवजए |
(4) Question number 11 – 12 and 28 – 29 are short-answer questions carrying 3 marks each. Answers to
them should normally not exceed 60-80 words each.
प्रश्न संख्या 11 से 12 एिं 28 से 29 तक के सभी प्रश्न लघूत्तरात्मक प्रश्न है, जो कक तीन-तीन अंक के है | इन प्रश्नों के
उत्तर 60-80 शब्र्दों से अविक नहीं होने चावहए |
(5) Question number 13 – 15 and 30 – 32 are also short-answer questions carrying 4 marks each. Answers to
them should normally not exceed 80-100 words each.
प्रश्न संख्या 13 से 15 एिं 30 से 32 तक के सभी प्रश्न भी लघूत्तरात्मक प्रश्न है, जो कक चार-चार अंक के है | इन प्रश्नों
के उत्तर 80-100 शब्र्दों से अविक नहीं होने चावहए |
(6) Question number 16 – 17 and 33 – 34 are long-answer questions carrying 6 marks each. Answers to
them should normally not exceed 100-150 words each.
प्रश्न संख्या 16 से 17 एिं 33 से 34 तक के सभी प्रश्न र्दीघथ उत्तरात्मक प्रश्न है, जो कक छह-छह अंक के है | इन प्रश्नों के
उत्तर 100-150 शब्र्दों से अविक नहीं होने चावहए |
(7) Answers should be brief and to the point and the above word limits should be adhered to as
far as possible.
सभी प्रश्नों के उत्तर यर्ा सम्भि प्रर्दत्त शब्र्द सीमा को ध्यान में रखते हुए संवक्षप्त, तथ्यात्मक एिं सटीक होने चावहए |
(8) An additional 15 minutes has been allotted to read the question paper.
इस प्रश्न पत्र को पढ़ने हेतु पंरह (15) वमनट का अवतररक्त समय आिंरटत ककया गया है |
Q. NO. Section-A (Introductory Macro Economics) Marks
प्र. सं. खण्ड-अ (पररचयात्मक समवि अर्थशास्त्र) अंक
1 Read the following statements carefully. 1
वनम्नवलवखत कर्नों को ध्यानपूिथक पकढ़ए |
Statement 1: New addition to capital stock in an economy is measured by net
investment or new capital formation.
कर्न 1: ककसी अर्थव्यिस्र्ा में पूंजी स्टॉक में नई िृवि को शुि वनिेश या नए पूंजी वनमाथण द्वारा
मापा जाता है।

Statement 2: The Central Statistics Office (CSO) of the Government of India has been
reporting the GDP at factor cost and at market prices.
कर्न 2: भारत सरकार का कें रीय सांवख्यकी कायाथलय (सीएसओ) कारक लागत और बाजार
कीमतों पर जीडीपी की ररपोटथ करता रहा है।
In light of the given statements, choose the correct alternative from the following:
कर्दए गए कर्नों के आलोक में वनम्नवलवखत में से सही विकल्प का चयन कीवजए:
(A) Statement 1 is true and Statement 2 is false.
कर्न 1 सत्य है और कर्न 2 गलत है |
(B) Statement 1 is false and Statement 2 is true.
कर्न 1 गलत है और कर्न 2 सत्य है |
(C) Both Statement 1 and Statement 2 is true.
कर्न 1 और कर्न 2 र्दोनों सत्य हैं |
(D) Both Statement 1 and Statement 2 is false.
कर्न 1 और कर्न 2 र्दोनों गलत हैं |

2 Read the following Assertion (A) and Reason (R). 1


Choose the correct alternative.
वनम्नवलवखत अवभकर्न (A) और कारण (R) को ध्यानपूिथक पकढ़ए |
वनम्नवलवखत विकल्पों में से एक सही विकल्प का चयन कीवजए |
Assertion (A): Ex-ante depicts what has been planned, and ex-post depicts what has
actually happened.
अवभकर्न (A): पूिथ-पूिथ र्दशाथता है कक क्या योजना बनाई गई है, और पूिथ-पिात यह र्दशाथता है
कक िास्ति में क्या हुआ है।

Reason (R): The 45 degree line has the feature that every point on it has not the
same horizontal and vertical coordinates.
कारण (R): 45 वडग्री रे खा की यह विशेषता है कक इसके प्रत्येक बबंर्दु समान क्षैवतज और ऊध्िाथिर
वनर्देशांक नहीं है।

Alternatives:
विकल्प:
(A) Both Assertion (A) and Reason (R) are true and Reason (R) is the correct explanation
of Assertion (A).
अवभकर्न (A) और कारण (R) र्दोनों सत्य हैं और कारण (R) अवभकर्न (A) का सही स्पिीकरण है।
(B) Both Assertion (A) and Reason (R) are true and Reason (R) is not the correct
explanation of Assertion (A).
अवभकर्न (A) और कारण (R) र्दोनों सत्य हैं और कारण (R) अवभकर्न (A) का सही स्पिीकरण
नहीं है।
(C) Assertion (A) is true but Reason (R) is false.
अवभकर्न (A) सत्य है लेककन कारण (R) गलत है।
(D) Assertion (A) is false but Reason (R) is true.
अवभकर्न (A) गलत है लेककन कारण (R) सत्य है।

3 State Bank of India receives a deposit of ₹ 90,000 and the reserve requirement is 1
10%. If the bank decides to hold reserves equal to 15% of the deposit instead of the
required amount, how much excess reserve does the bank hold?

भारतीय स्टेट बैंक को ₹ 90,000 की जमा रावश प्राप्त होती है और आरवक्षत आिश्यकता 10%
है। यकर्द बैंक आिश्यक आरवक्षत रावश के बजाय जमा के 15% के बराबर आरवक्षत रखने का
वनणथय लेता है, तब बैंक अपने पास ककतना अवतररक्त आरवक्षत भंडार रखता है?
(A) ₹ 4,500 (B) ₹ 9,000 (C) ₹ 13,500 (D) ₹ 18,000

4 An Indian real estate company receives rent from Google in New York. This 1
transaction would be recorded on ____________________ side of
___________________ account. (Fill up the blanks with correct alternative)

एक भारतीय ररयल एस्टेट कं पनी को न्द्यूयॉकथ में गूगल से ककराया की प्रावप्त होती है। यह लेन-र्देन
_______ पक्ष पर ______________ खाते में र्दजथ ककया जाएगा।
(ररक्त स्र्ान को पूर्तथ सही विकल्प से कीवजए)
(A) credit, current (B) credit, capital
िे वडट, चालू िे वडट, पूंजी
(C) debit, capital (D) debit, current
डेवबट, पूंजी डेवबट, चालू
5 What is the value of marginal propensity to consume, when 1
सीमांत उपभोग प्रिृवत का मान क्या होगा, जब
S = (-) 200 + 0.6Y
(A) 0.6 (B) (-) 0.6 (C) (-) 0.4 (D) 0.4

6 Identify the correct reasons that may affect the demand for foreign exchange in an 1
economy.
उन सही कारणों की पहचान कीवजए जो ककसी अर्थव्यिस्र्ा में विर्देशी मुरा की मांग को प्रभावित
कर सकते हैं।
I Imports of visible
र्दृश्यमान िस्तुओं का आयात
II Exports of invisibles
अर्दृश्यमान िस्तुओं का वनयाथत
III Remittances by residents working abroad
विर्देश में काम करने िाले वनिावसयों द्वारा विप्रेषण
IV Purchase of assets in abroad
विर्देश में संपवत्तयों की खरीर्द
Alternatives:
विकल्प:
(A) I and IV (B) II and III (C) III and IV (D) I and II

7 The influence exerted by international borrowing on the foreign exchange supply 1


within an economy is indicated when it:
ककसी अर्थव्यिस्र्ा के भीतर विर्देशी मुरा आपूर्तथ पर अंतराथष्ट्रीय उिार द्वारा डाले गए प्रभाि का
संकेत तब वमलता है जब:
(A) Minimally affects the supply of foreign exchange.
विर्देशी मुरा की आपूर्तथ को न्द्यूनतम रूप से प्रभावित करता है |
(B) Tends to diminish the supply of foreign exchange.
विर्देशी मुरा की आपूर्तथ कम हो जाती है |
(C) Intends to increase the supply of foreign exchange.
विर्देशी मुरा की आपूर्तथ बढ़ाने का इरार्दा रखता है |
(D) Confined solely to alterations in the supply of domestic currency.
के िल घरे लू मुरा की आपूर्तथ में पररितथन तक ही सीवमत है |

8 The difference by which actual Aggregate Demand exceeds the Aggregate Demand, 1
required to establish full employment equilibrium is known as
_________________________ (inflationary gap / deflationary gap).
(Choose the correct alternative)
िह अंतर वजसके द्वारा िास्तविक समग्र मांग, पूणथ रोजगार संतल
ु न स्र्ावपत करने के वलए
आिश्यक समग्र मांग से अविक हो जाती है, तो इसे ______________ (मुरास्फीवत अंतर /
अपस्फीवत अंतर) के रूप में जाना जाता है। (सही विकल्प का चयन कीवजए)
9 Supply of money refers to___________________________. 1
(Choose the correct alternative)
मुरा आपूर्तथ से तात्पयथ _________________ से है | (सही विकल्प का चयन कीवजए)

(A) Currency held by the public


जनता के पास मौजूर्द मुरा
(B) Currency held by Reserve Bank of India (RBI)
भारतीय ररज़िथ बैंक (RBI) द्वारा रखी गई मुरा
(C) Currency held by the public and demand deposits with commercial banks
जनता के पास मौजूर्द मुरा और िावणवज्यक बैंकों के पास मांग जमा
(D) Currency held in the government account.
सरकारी खाते में रखी गई मुरा

10 Which of the following statements is true? (Choose the correct alternative) 1


वनम्नवलवखत में से कौन-सा कर्न सत्य है? (सही विकल्प का चयन कीवजए)
(A) The flexible exchange rate system gives the government more flexibility to maintain
large stocks of foreign exchange reserves.
विर्देशी मुरा भंडार के बड़े स्टॉक में लचीलापन बनाए रखने में लचीली विवनमय र्दर प्रणाली
सरकार को अविक लाभ र्देती है ।
(B) In the Managed floating exchange rate system, the government intervenes to buy
and sell foreign currencies.
प्रबंवित फ्लोटटंग विवनमय र्दर प्रणाली में, सरकार विर्देशी मुरा खरीर्दने और बेचने में हस्तक्षेप
करती है।
(C) In the Managed floating exchange rate system, the central bank intervenes to
moderate exchange rate fluctuations.
प्रबंवित फ्लोटटंग विवनमय र्दर प्रणाली में, कें रीय बैंक विवनमय र्दर में उतार-चढ़ाि को वनयंवत्रत
करने में हस्तक्षेप करता है।
(D) In the Fixed exchange rate system, market forces fix the exchange rate.
वनवित विवनमय र्दर प्रणाली में, बाज़ार की ताकतें विवनमय र्दर को तय करती हैं ।

11 State, giving valid reasons, whether the following statements are true or false:
िैि कारण बताते हुए वनम्नवलवखत कर्न को सत्य अर्िा असत्य के रूप में बताइए:
(i) Gross Value Added at market price and Gross Domestic Product at market price are 1½
one and the same thing.
बाजार मूल्य पर सकल मूल्य िर्िथत और सकल घरे लू उत्पार्द पर बाज़ार मूल्य एक ही और समान
चीज़ हैं।
(ii) Intermediate goods are always durable in nature. 1½
मध्यिती िस्तुएँ हमेशा प्रकृ वत में रटकाऊ होती हैं।

12. (A) Trade Deficit must exist if a country is facing a situation of Current Account Deficit. 3
Defend or refute the statement, with valid argument.
यकर्द कोई र्देश चालू खाते में घाटे की वस्र्वत का सामना कर रहा है, तब व्यापार घाटा भी
आिश्यक रूप में मौजूर्द होना चावहए। िैि तकथ के सार् कर्न का बचाि या खंडन कीवजए ।
OR / अर्िा
(B) State, giving valid reasons, whether the following statements are true or false:
िैि कारण बताते हुए वनम्नवलवखत कर्न को सत्य अर्िा असत्य के रूप में बताइए:
(i) Current account in Balance of Payments records only the exports and imports of 1½
goods and services.
भुगतान संतुलन में चालू खाता के िल िस्तुओं और सेिाओं के वनयाथत और आयात को ररकॉडथ
करता है।
(ii) Borrowings from abroad are recorded in the Capital Account of the Balance of 1½
Payments on the debit side.
विर्देश से वलए गए उिार को पूंजी खाते में डेवबट पक्ष पर भुगतान संतुलन में र्दजथ ककया जाता है |

13.(A) Explain the process of money creation by bank with the help of a numerical example. 4
बैंक द्वारा ककए जाने िाली साख सजथन की प्रकिया का िणथन एक संख्यात्मक उर्दाहरण की
सहायता से कीवजए |
OR / अर्िा
(B) Elaborate the ‘Banker’s Bank’ and ‘Supervisor’ function performed by the Reserve 2+2=4
Bank of India.
भारतीय ररज़िथ बैंक द्वारा वनष्पाकर्दत 'बैंकसथ बैंक’ और ‘पयथिक्ष
े क' कायथ का विस्तार से िणथन
कीवजए ।

14 In an economy C= 200 + 0.5Y is the consumption function where C is the 4


consumption expenditure and Y is the national income. Investment expenditure is ₹
400 crores. Is the economy in equilibrium at an income level ₹1500 crores? Justify
your answer.
एक अर्थव्यिस्र्ा में C= 200 + 0.5 Y उपभोग फलन है, जहां C उपभोग व्यय है और Y राष्ट्रीय
आय है। वनिेश व्यय ₹ 400 करोड़ है। क्या अर्थव्यिस्र्ा ₹ 1500 करोड़ के आय स्तर पर संतल
ु न में
है? आपने जिाब का औवचत्य सावबत कीवजए ।

15 State the meaning of the following: 1x4=4


वनम्नवलवखत का अर्थ बताइए:
(i) Ex-ante savings (ii) Full Employment
वनयोवजत बचत पूणथ रोजगार
(iii) Autonomous Consumption (iv) Deficient Demand
स्िायत्त उपभोग न्द्यून मांग
16. (A) Giving valid reasons, explain how the following would be treated while estimating
domestic income?
िैि कारणों की सहायता से िणथन कीवजए कक घरे लू आय का अनुमान लगाते समय वनम्नवलवखत
का आकलन ककस प्रकार से ककया जाएगा?
(i) Payment made by American tourist for goods purchased in India. 1½
भारत में खरीर्दे गए सामान के वलए अमेररकी पयथटक द्वारा ककया गया भुगतान।
(ii) Tomatoes grown by Ms. Madhu in her kitchen garden. 1½
सुिी मिु द्वारा अपने रसोई उद्यान में उगाए गए टमाटर।

(B) "Machine purchased by a firm is always a capital good." Do you agree with the given 3
statement? Give valid reasons for your answer.
"ककसी फमथ द्वारा खरीर्दी गई मशीन हमेशा पूज
ं ीगत िस्तु होती है।" क्या आप कर्दए गए कर्न से
सहमत हैं? अपने उत्तर के वलए िैि कारण र्दीवजए।

17. (A) Utilizing the provided data, compute


कर्दए गए वनर्र्दथि का उपयोग करके , गणना कीवजए
i Revenue Deficit 1
राजस्ि घाटा
ii Fiscal Deficit 1
राजकोषीय घाटा
iii Primary Deficit 1
प्रार्वमक घाटा
SR. NO. PARTICULARS AMOUNT (₹ In crore)
ि. स. वििरण रावश (₹ करोड़ में)
i Revenue expenditure 80,350
राजस्ि व्यय
ii Capital expenditure 92,000
पूंजीगत व्यय
iii Interest payments 18,000
ब्याज भुगतान
iv Revenue receipts 63,875
राजस्ि प्रावप्तयां
v Borrowings 59,640
उिार
vi Capital receipts 74,000
पूंजीगत प्रावप्तयां
(B) Outline a hypothetical scenario where a government is considering implementing a 3
new tax policy. In this scenario, explain how the introduction of a Progressive Tax
would affect individuals with low, middle and high incomes.
एक काल्पवनक पररर्दृश्य की रूपरे खा तैयार कीवजए जहां सरकार एक नई कर नीवत लागू करने
पर विचार कर रही है। इस पररर्दृश्य में, बताइए कक प्रगवतशील कर की शुरूआत वनम्न, मध्यम
और उच्च आय िाले व्यवक्तयों को ककस प्रकार से प्रभावित करे गी ?
OR / अर्िा
(C) Explain the Government's Budget and its various sources of revenue. Present three 3
actual instances of non-tax revenue earnings that a government could accumulate
during its fiscal year.
सरकार के बजट और उसके राजस्ि के विवभन्न स्रोतों की व्याख्या कीवजए । गैर-कर राजस्ि आय
के तीन िास्तविक उर्दाहरण प्रस्तुत कीवजए वजन्द्हें एक सरकार अपने वित्तीय िषथ के र्दौरान जमा
कर सकती है।

(D) Highlight three main objectives that guide the government to frame their policies 3
and decisions. Explain these three objectives providing examples.
सरकार को अपनी नीवतयों और वनणथयों को तैयार करने में मागथर्दशथन करने िाले तीन मुख्य
उद्देश्यों पर प्रकाश डावलए । उर्दाहरण र्देकर इन तीन उद्देश्यों की व्याख्या कीवजए ।

SECTION–B (Indian Economic Development)


खण्ड-ब (भारतीय अर्थव्यिस्र्ा का विकास)
18 India entered the __________________________ stage of Demographic Transition 1
after the year 1921. (Choose the correct alternative).
िषथ 1921 के बार्द भारत ने जनसांवख्यकीय संिमण के _________________ चरण में प्रिेश
ककया। (सही विकल्प का चयन कीवजए)
(A) Forth (B) Second (C) Third (D) First
चौर्े र्दूसरे तीसरे पहले
19 _______________________________ was the Indian Finance Minister in 1991, 1
acknowledged for his capabilities to steer away the economic crisis looming large on
the erstwhile Indian Economy. (Choose the correct alternative).
1991 में भारतीय वित्त मंत्री _____________ र्े, वजन्द्हें तत्कालीन भारतीय अर्थव्यिस्र्ा पर
मंडरा रहे आर्र्थक संकट को र्दूर करने की उनकी क्षमताओं के वलए जाना जाता है।
(सही विकल्प का चयन कीवजए)
(A) Dr. Subramanian Swamy (B) Dr. Urjit Patel
डॉ. सुिमण्यम स्िामी डॉ उर्जथत पटेल
(C) Pranab Mukherjee (D) Dr. Manmohan Singh
प्रणब मुखजी डॉ. मनमोहन बसंह
20 The main objective of China's One Child Policy, which was implemented from 1979 1
to 2016 was:
1979 से 2016 तक लागू की गई चीन की एक बच्चे की नीवत का मुख्य उद्देश्य र्ा:
(A) To provide financial incentives for families with multiple children.
एकाविक बच्चों िाले पररिारों के वलए वित्तीय प्रोत्साहन प्रर्दान करना
(B) To promote gender equality by limiting the number of male children.
पुरुष बच्चों की संख्या को सीवमत करके लैंवगक समानता को बढ़ािा र्देना
(C) To control the population growth and address overpopulation concerns.
जनसंख्या िृवि को वनयंवत्रत करने और अविक जनसंख्या संबि ं ी बचंताओं का समािान करने के वलए
(D) To encourage families to have more children and increase the population.
पररिारों को अविक बच्चे पैर्दा करने और जनसंख्या बढ़ाने के वलए प्रोत्सावहत करना

21 Read the following statements carefully. 1


वनम्नवलवखत कर्नों को ध्यानपूिथक पकढ़ए |
Statement 1: India became an exporter of primary products and an importer of
finished consumer and capital goods produced in Britain.
कर्न 1: भारत प्रार्वमक उत्पार्दों का वनयाथतक और विटेन में उत्पाकर्दत तैयार उपभोक्ता एिं
पूंजीगत िस्तुओं का आयातक बन गया।

Statement 2: Restrictive policies of commodity production, trade and tariff pursued


by the colonial government adversely affected the structure, composition and
volume of India’s foreign trade.
कर्न 2: औपवनिेवशक सरकार द्वारा अपनाई गई िस्तु उत्पार्दन, व्यापार और प्रशुल्क की
प्रवतबंिात्मक नीवतयों ने भारत के विर्देशी व्यापार की संरचना, संरचना और मात्रा पर प्रवतकू ल
प्रभाि डाला।
In light of the given statements, choose the correct alternative from the following:
कर्दए गए कर्नों के आलोक में वनम्नवलवखत में से सही विकल्प का चयन कीवजए:
(A) Statement 1 is true and Statement 2 is false
कर्न 1 सत्य है और कर्न 2 गलत है |
(B) Statement 1 is false and Statement 2 is true
कर्न 1 गलत है और कर्न 2 सत्य है |
(C) Both Statement 1 and Statement 2 are true
र्दोनों कर्न 1 और कर्न 2 सत्य हैं |
(D) Both Statement 1 and Statement 2 are false
र्दोनों कर्न 1 और कर्न 2 गलत हैं |

22 Read the following Assertion (A) and Reason (R). 1


Choose the correct alternative.
वनम्नवलवखत अवभकर्न (A) और कारण (R) को ध्यानपूिथक पकढ़ए |
वनम्नवलवखत विकल्पों में से एक सही विकल्प का चयन कीवजए |
Assertion (A): The major policy initiatives i.e. land reforms and green revolution
helped India to become self-sufficient in food grains production.
अवभकर्न (A) : प्रमुख नीवतगत पहल यानी भूवम सुिार और हररत िांवत ने भारत को खाद्यान्न
उत्पार्दन में आत्मवनभथर बनने में मर्दर्द की।

Reason (R): The proportion of people depending on agriculture did not decline as
expected.
कारण (R): कृ वष पर वनभथर लोगों के अनुपात में उम्मीर्द के अनुरूप वगरािट नहीं आई |
Alternatives:
विकल्प:
(A) Both Assertion (A) and Reason (R) are true and Reason (R) is the correct explanation
of Assertion (A).
अवभकर्न (A) और कारण (R) र्दोनों सत्य हैं और कारण (R) अवभकर्न (A) का सही स्पिीकरण है।
(B) Both Assertion (A) and Reason (R) are true and Reason (R) is not the correct
explanation of Assertion (A).
अवभकर्न (A) और कारण (R) र्दोनों सत्य हैं और कारण (R) अवभकर्न (A) का सही स्पिीकरण
नहीं है।
(C) Assertion (A) is true but Reason (R) is false.
अवभकर्न (A) सत्य है लेककन कारण (R) गलत है।
(D) Assertion (A) is false but Reason (R) is true.
अवभकर्न (A) गलत है लेककन कारण (R) सत्य है।

23 A farmer named Chandan practices organic farming and uses crop rotation and 1
natural predators to control pests. Is there an advantage of this approach provided
as compared to conventional pesticides use? Choose the correct alternative from
the following:

चन्द्र्दन नामक एक ककसान जैविक खेती करता हैं और कीटों को वनयंवत्रत करने के वलए फसल चि
और प्राकृ वतक जानिरों का उपयोग करता हैं। क्या पारं पररक कीटनाशकों के उपयोग की तुलना में
इस प्रकार के र्दृविकोण का कोई लाभ है? वनम्नवलवखत में से सही विकल्प का चयन कीवजए:
(A) Faster and predictable pest eradication
तेज और पूिाथनुमावनत कीट उन्द्मल
ू न
(B) Lower labor costs for pest management
कीट प्रबंिन के वलए कम िम लागत
(C) Increased risk of crop failure and reduced yields
फसल की विफलता और कम पैर्दािार का खतरा बढ़ गया
(D) Reduced harm to beneficial insects and pollinators
लाभकारी कीड़ों और परागणकों को कम नुकसान

24 From the set of events / systems given in Column-I and corresponding relevant 1
fact given in Column-II, about China, choose the correct pair of statement:
चीन के बारे में कॉलम-I और कॉलम-II में कर्दए गए तथ्यों में से संबंवित प्रासंवगक घटनाओं /
प्रणावलयों के सही कर्न युग्म का चयन कीवजए :
Column I Column II
कॉलम-I कॉलम-II
i Great Leap Forward a Students were sent to work and learn
from the countryside.
ग्रेट लीप फॉरिडथ छात्रों को ग्रामीण इलाकों में काम करने और
सीखने के वलए भेजा गया
ii Commune System b Cultivating land Collectively.
कम्यून व्यिस्र्ा सामूवहक रूप से भूवम पर खेती करना
iii Proletarian Cultural Revolution c Opening of the Industries in their homes.
सिथहारा सांस्कृ वतक िांवत अपने घरों में उद्योग खोलना
iv Economic Reforms in China d 1988
आर्र्थक सुिार में चीन में
Alternatives:
विकल्प:
(A) i -a (B) ii-b (C) iii-c (D) iv-d

25 Read the following Assertion (A) and Reason (R). 1


Choose the correct alternative.
वनम्नवलवखत अवभकर्न (A) और कारण (R) को ध्यानपूिथक पकढ़ए |
वनम्नवलवखत विकल्पों में से एक सही विकल्प का चयन कीवजए |
Assertion (A): In the past few decades, primary sector has created maximum jobs in
India.
अवभकर्न (A): वपछले कु छ र्दशकों में प्रार्वमक क्षेत्र ने भारत में सिाथविक रोजगार सृवजत ककए हैं।

Reason (R): Agriculture is not a major source of employment in urban areas.


कारण (R): शहरी क्षेत्रों में कृ वष रोजगार का प्रमुख स्रोत नहीं है।

Alternatives:
विकल्प:
(A) Both Assertion (A) and Reason (R) are true and Reason (R) is the correct explanation
of Assertion (A).
अवभकर्न (A) और कारण (R) र्दोनों सत्य हैं और कारण (R) अवभकर्न (A) का सही स्पिीकरण है।
(B) Both Assertion (A) and Reason (R) are true and Reason (R) is not the correct
explanation of Assertion (A).
अवभकर्न (A) और कारण (R) र्दोनों सत्य हैं और कारण (R) अवभकर्न (A) का सही स्पिीकरण
नहीं है।
(C) Assertion (A) is true but Reason (R) is false.
अवभकर्न (A) सत्य है लेककन कारण (R) गलत है।
(D) Assertion (A) is false but Reason (R) is true.
अवभकर्न (A) गलत है लेककन कारण (R) सत्य है।

26 At the G-20 Summit year 2023, under the theme of 1


____________________important issues such as food security, climate and energy,
development, health and digitalization were discussed. (Choose the correct
alternative to fill up the blank)
िषथ 2023 के जी-20 वशखर सम्मेलन में, __________________ की र्ीम के तहत खाद्य
सुरक्षा, जलिायु और ऊजाथ, विकास, स्िस्थ्य और वडवजटलीकरण जैसे मुद्दों पर चचाथ की गई |
(ररक्त स्र्ान की पूर्तथ करने के वलए सही विकल्प का चयन कीवजए)
(A) Shaping an Interconnected World
एक अंतसिंबंवित विश्व को आकार र्देना |
(B) Fighting poverty with rigidity
कठोरतापूिथक गरीबी से लड़ना |
(C) One Earth-One Family-One Future
एक पृथ्िी-एक पररिार-एक भविष्य |
(D) Making the world together
विश्व को एकीकृ त करना |

27 Read the following statements carefully. 1


वनम्नवलवखत कर्नों को ध्यानपूिथक पकढ़ए |
Statement 1: It is easy to prove cause and effect relation between human capital and
economic growth.
कर्न 1 : मानि पूँजी और आर्र्थक विकास के बीच कारण और प्रभाि संबि ं को वसि करना सरल
है |

Statement 2: Human capital formation promotes inventions, innovations and


technological improvements.
कर्न 2 : मानि पूँजी वनमाथण, अविष्कारों, निाचारों और तकनीकी सुिारों को बढ़ािा र्देता है |
In light of the given statements, choose the correct alternative from the following:
कर्दए गए कर्नों के आलोक में वनम्नवलवखत में से सही विकल्प का चयन कीवजए:
(A) Both the statements 1 and 2 are true.
कर्न 1 और कर्न 2 र्दोनों सत्य हैं |
(B) Both the statements 1 and 2 are false.
कर्न 1 और कर्न 2 र्दोनों असत्य हैं |
(C) Statement 1 is true and statement 2 is false.
कर्न 1 सत्य है और कर्न 2 असत्य हैं |
(D) Statement 1 is false and statement 2 is true.
कर्न 1 असत्य है और कर्न 2 सत्य हैं |
28. (A) "India has emerged as a hotspot for medical tourism”. Defend the statement with valid 3
arguments.
"भारत वचककत्सा पयथटन के वलए एक मुख्य के न्द्र वबन्द्र्दु के रूप में उभरा है"। िैि तकों के सार्
कर्न का समर्थन कीवजए ।
OR / अर्िा
(B) Argue in favour of the need for different forms of government intervention in 3
education and health sector.
वशक्षा एिं स्िस्थ्य क्षेत्रों में विविि प्रकार के सरकारी हस्तक्षेप की आिश्यकता के पक्ष में तकथ
र्दीवजए |

29 “Agriculture sector appears to be adversely affected by the economic reform 3


process.” Explain the given statement.
"आर्र्थक सुिार प्रकिया से कृ वष क्षेत्र पर प्रवतकू ल प्रभाि पड़ता कर्दख रहा है।" कर्दए गए कर्न का
िणथन कीवजए।

30 State, giving valid reasons, whether the following statements are true or false:
िैि कारण बताते हुए वनम्नवलवखत कर्न को सत्य अर्िा असत्य के रूप में बताइए:
(i) Since independence, the benefits of the increase in economic growth in India have 2
trickled down to the people at the bottom of population pyramid.
आजार्दी के बार्द से, भारत में आर्र्थक विकास में िृवि का लाभ जनसंख्या वपरावमड के वनचले स्तर
पर मौजूर्द लोगों तक पहुंचा है।

(ii) SHG has emerged as one of the major micro finance programme in the country in 2
recent years.
हाल के िषों में स्िंय सहायता समूह सूक्ष्म वित्त कायथिम र्देश में अनेकों कायथिमों में से एक प्रमुख के
रूप में उभरा है |

31 “Since independence, the government has been playing an important role in 4


generating employment directly or indirectly.” Discuss.

"आजार्दी के बार्द से, सरकार प्रत्यक्ष या परोक्ष रूप से रोजगार उत्पन्न करने में महत्त्िपूणथ भूवमका
वनभा रही है।" चचाथ कीवजए ।

32. (A) “The Prime Minister urged to increase the rural income by increasing non- farm 4
activities.” Explain how non-farm activities can lead to rise in income of people in
rural sector?

“प्रिानमंत्री जी ने गैर-कृ वषक गवतविवियों में बढ़ोतरी करके ग्रामीण आय को बढ़ाने का आग्रह
ककया है”| िणथन कीवजए कक गैर-कृ वषक गवतविवियां ककस प्रकार से ग्रामीण क्षेत्र में लोगों की आय
में िृवि कर सकती है ?
OR / अर्िा
(B) “Do you think the various measures taken by the government to improve agricultural 4
marketing system are sufficient”? Discuss.

“क्या आपको लगता है कक कृ वष विपणन प्रणाली में सुिार के वलए सरकार द्वारा उठाए गए
विवभन्न कर्दम पयाथप्त है” ? चचाथ कीवजए |

33. (A) “If the rate of resource extraction exceeds the rate of regeneration, it leads to 3
reduction in carrying capacity of the environment.” Discuss the rationale of the given
statement with valid reasons.
"यकर्द संसािन वनष्कषथण की र्दर पुनजथनन की र्दर से अविक है, तो इससे पयाथिरण की िहन क्षमता
में कमी आती है।" कर्दए गए कर्न के औवचत्य पर िैि कारणों सवहत चचाथ कीवजए ।
(B) “Calorie-based norm is not an adequate measure to identify the poor”. Establish the 3
rationale of the given statement with valid reasons.
“गरीबों की पहचान के वलए कै लोरी आिाररत मानर्दंड पयाथप्त उपाय नहीं है।“ िैि कारणों के सार्
कर्दए गए कर्न का औवचत्य स्र्ावपत कीवजए ।
OR / अर्िा
(C) Answer the following questions on the basis of the following data:
वनम्नवलवखत आँकड़ों के आिार पर वनम्नवलवखत प्रश्नों के उत्तर र्दीवजए:
India: Trends in Employment Pattern (Sector wise), 1993-2012 (in %)
भारत: रोजगार प्रवतमान में रुझान (क्षेत्रिार), 1993-2012 (% में)
Sector 1993-94 1999-2000 2011-2012
क्षेत्र
Primary 64 60.4 24.3
प्रार्वमक
Secondary 16 15.8 24.3
वद्वतीयक
Services 20 23.8 26.8
सेिाएं
I Why are less women found in regular salaried employment? 1
वनयवमत िेतनभोगी रोज़गार में मवहलाएँ कम क्यों पाई जाती हैं?
II Discuss any two disadvantages of casualization of the workforce 2
कायथबल के अकवस्मकीकरण के ककन्द्हीं र्दो नुकसानों पर चचाथ कीवजए |
III Analyze the recent trends in sectoral distribution of workforce in India. 3
भारत में कायथबल के क्षेत्रिार वितरण में हावलया रुझानों का विश्लेषण कीवजए |
34 Read the following text carefully and answer the given questions on the basis of the
same and common understanding:
Global warming is a gradual increase in the average temperature of the earth’s lower
atmosphere as a result of the increase in greenhouse gases since the Industrial
Revolution. Much of the recent observed and projected global warming is human-
induced. It is caused by man-made increases in carbon dioxide and other greenhouse
gases through the burning of fossil fuels and deforestation. Adding carbon dioxide,
methane and such other gases (that have the potential to absorb heat) to the
atmosphere with no other changes will make our planet’s surface warmer. The
atmospheric concentrations of carbon dioxide and CH4 have increased by 31 per cent
and 149 per cent respectively above pre-industrial levels since 1750. During the past
century, the atmospheric temperature has risen by 1.1°F (0.6°C) and sea level has
risen several inches. Some of the longer-term results of global warming are melting of
polar ice with a resulting rise in sea level and coastal flooding; disruption of drinking
water supplies dependent on snow melts; extinction of species as ecological niches
disappear; more frequent tropical storms; and an increased incidence of tropical
diseases. Among factors that may be contributing to global warming are the burning
of coal and petroleum products (sources of carbon dioxide, methane, nitrous oxide,
ozone); deforestation, which increases the amount of carbon dioxide in the
atmosphere; methane gas released in animal waste; and increased cattle production,
which contributes to deforestation, methane production, and use of fossil fuels. A UN
Conference on Climate Change, held in Kyoto, Japan, in 1997, resulted in an
international agreement to fight global warming which called for reductions in
emissions of greenhouse gases by industrialized nations.
वनम्नवलवखत पररच्छेर्द को ध्यानपूिथक पकढ़ए और कर्दए गए प्रश्नों के उत्तर पररच्छेर्द एिं सामान्द्य
समझ के आिार पर र्दीवजए :
औद्योवगक िांवत के पिात से ग्रीनहाउस गैसों में िृवि के पररणाम स्िरूप िैवश्वक ताप से पृथ्िी के
वनचले िायुमंडल के औसत तापमान में िवमक िृवि हुई है। हाल ही में र्देखी गई और अनुमावनत
की गई िैवश्वक ताप में से अविकांश मानि-प्रेररत है। यह जीिाश्म ईंिन के जलने और िनों की
कटाई के माध्यम से काबथन डाइऑक्साइड और अन्द्य ग्रीनहाउस गैसों में मानि वनर्मथत िृवि के
कारण होता है। वबना ककसी अन्द्य पररितथन के िायुमंडल में काबथन डाइऑक्साइड, मीर्ेन और
ऐसी अन्द्य गैसें (वजनमें गमी को अिशोवषत करने की क्षमता होती है) वमलाने से हमारे ग्रह की
सतह गमथ हो जाएगी। 1750 के बार्द से काबथन डाइऑक्साइड और CH4 की िायुमंडलीय सांरता
पूिथ-औद्योवगक स्तरों से िमशः 31 प्रवतशत और 149 प्रवतशत बढ़ गई है। वपछली शताब्र्दी के
र्दौरान, िायुमंडलीय तापमान 1.1°F (0.6°C) बढ़ गया है और समुर का स्तर कई इं च ऊपर उठ
गया है | िैवश्वक ताप के कु छ र्दीघथकावलक पररणाम हैं जैसे ध्रुिीय बफथ का वपघलना वजसके
पररणामस्िरूप समुर के स्तर में िृवि और तटीय बाढ़; बफथ वपघलने पर वनभथर पेयजल आपूर्तथ में
व्यििान; पाररवस्र्वतक क्षेत्रों के लुप्त होने से प्रजावतयों का विलुप्त होना; अविक लगातार
उष्णकरटबंिीय तूफान; और उष्णकरटबंिीय रोगों की घटनाओं में िृवि हुई है। िैवश्वक ताप में
योगर्दान र्देने िाले कारकों में कोयला और पेरोवलयम उत्पार्दों (काबथन डाइऑक्साइड, मीर्ेन,
नाइरस ऑक्साइड, ओजोन के स्रोत) का जलना शावमल है; िनों की कटाई, वजससे िातािरण में
काबथन डाइऑक्साइड की मात्रा बढ़ जाती है; पशु अपवशि में उत्सर्जथत मीर्ेन गैस; और मिेशी
उत्पार्दन में िृवि हुई, जो िनों की कटाई, मीर्ेन उत्पार्दन और जीिाश्म ईंिन के उपयोग में
योगर्दान र्देता है। 1997 में क्योटो, जापान में आयोवजत जलिायु पररितथन पर संयुक्त राष्ट्र
सम्मेलन के पररणामस्िरूप िैवश्वक ताप से लड़ने के वलए एक अंतराथष्ट्रीय समझौता हुआ, वजसमें
औद्योवगक र्देशों द्वारा ग्रीनहाउस गैसों के उत्सजथन में कटौती का आहिान ककया गया।
(I) Define Global Warming. 2
िैवश्वक ताप को पररभावषत कीवजए ।
(II) Briefly elaborate two causes and consequences of Global Warming. 2+2=4
िैवश्वक ताप के र्दो कारणों और पररणामों का संक्षप े में िणथन कीवजए ।
**************************************
ROLL NO………………………………….. SUBJECT CODE: 030
अनुिमांक संख्या ............................ विषय कोड : 030
KENDRIYA VIDYALAYA SANGATHAN, JAIPUR REGION
कें रीय विद्यालय संगठन, जयपुर संभाग SET-7
Practice Paper
अभ्यास पत्र
SUBJECT: ECONOMICS CLASS- XII
विषय :- अर्थशास्त्र कक्षा :- 12
M.M. 80 MARKS TIME: 3 HOURS
अविकतम अंक :- 80 अंक समय :- 3 घंटे
-------------------------------------------------------------------------------------------------------------------------------------
General instructions:
सामान्द्य वनर्देश:
(1) This question paper contains two sections:
इस प्रश्न पत्र के र्दो भाग है :-
Section–A Macro Economics
भाग अ- समवि अर्थशास्त्र
Section–B Indian Economy
भाग ब- भारतीय अर्थव्यिस्र्ा
All the questions in both the sections are compulsory. Marks for questions are indicated against each question.
र्दोनों खंडों में सभी प्रश्न अवनिायथ हैं। प्रत्येक प्रश्न के अंक उसके सामने कर्दए गए हैं।
(2) Question number 1-10 and 18-27 are very short-answer questions carrying 1 mark each. They are required to be
answered in one word or one sentence each.
प्रश्न संख्या 1–10 एिं 18–27 अवत लघु उत्तरीय प्रश्न / बहुविकल्पीय प्रश्न हैं प्रत्येक के 1 अंक हैं | उनका उत्तर एक शब्र्द या िाक्य में
कर्दया जाना चावहए ।
(3) Question number 11-12 and 28-29 are short-answer questions caring 3 marks each. Answers to them should not
normally exceed 60-80 words each.
प्रश्न संख्या 11-12 एिं 28-29 लघु उत्तरीय प्रश्न हैं वजनमें प्रत्येक के 3 अंक हैं। उनके उत्तर सामान्द्यतः 60-80 शब्र्द से अविक नहीं होने
चावहए।
(4) Question number 13-15 and 30-32 are also short-answer questions carrying 4 marks each. Answers to them should
not normally exceed 80-100 words each.
प्रश्न संख्या 13–15 एिं 30–32 भी लघु उत्तरीय प्रश्न हैं, वजनमें से प्रत्येक के 4 अंक हैं। उनके उत्तर सामान्द्यतः 80-100 शब्र्द से अविक
नहीं होना चावहए।
(5) Question number 16-17 and 33-34 are long answer questions carrying 6 marks each. Answers to them should not
normally exceed 100-150 words each.
प्रश्न संख्या 16–17 एिं 33–34 र्दीघथ उत्तरीय प्रश्न हैं वजनमें से प्रत्येक में 6 अंक हैं। उनके उत्तर सामान्द्यतः 100-150 शब्र्दों से अविक
नहीं होने चावहए।
(6) Answer should be brief and to the point and the word limit be adhered to as far as possible.
उत्तर संवक्षप्त तर्ा तथ्यात्मक होने चावहए एिं शब्र्द सीमा का यर्ासंभि पालन ककया जाना चावहए।
(7) An additional 15 minutes has been allotted to read the question paper.
इस प्रश्न पत्र को पढ़ने हेतु पंरह (15) वमनट का अवतररक्त समय आबंरटत ककया गया है |
SECTION – A (Introductory Macro Economics)
खण्ड: क (पररचयात्मक समवि अर्थशास्त्र)
1- Read the following statements carefully: (1)
Statement 1: The consumption curve is an upward sloping straight line curve due to the direct relationship between
income and consumption and the assumption of constant Marginal Propensity to Consume
Statement 2: Aggregate Demand curve and Consumption curve are parallel to each other.
In the light of the given statements, choose the correct alternative from the following:
Alternatives:
(A) Statement 1 is true and statement 2 is false (B) Statement 1 is false and statement 2 is true
(C) Both statements 1 and 2 are true (D) Both statements 1 and 2 are false

वनम्नवलवखत कर्नों को ध्यानपूिथक पकढ़ए :


कर्न-1: आय और उपभोग के बीच सीिे संबंि और वनरं तर सीमांत उपभोग प्रिृवत्त की िारणा के कारण उपभोग िि एक ऊपर की ओर
झुका हुआ सीिी रे खा िि है।
कर्न-2: समग्र मांग िि और उपभोग िि एक र्दूसरे के समानांतर होते हैं।
कर्दए गए कर्नों के सन्द्र्दभथ में, वनम्नवलवखत में से सही विकल्प का चयन कीवजए:
विकल्प:
(A) कर्न 1 सत्य है और कर्न 2 गलत है | (B) कर्न 1 गलत है और कर्न 2 सत्य है |
(C) कर्न 1 और 2 र्दोनों सत्य हैं | (D) कर्न 1 और 2 र्दोनों गलत हैं |

2- Read the following statements carefully: (1)


Statements 1: Gross Domestic Product (GDP) is the sum total of the gross market value of all the final goods and services
added by all the sectors in the economy during a fiscal year.
Statements 2: Gross Value Added at Market Price (GVAMP) is equal to the excess of value of output over intermediate
consumption.
In the light of given statements, choose the correct alternatives from the following:
Alternatives:
(A) Statement 1 is true and statement 2 is false (B) Statement 1 is false and statement 2 is true
(C) Both statements 1 and 2 are true (D) Both statements 1 and 2 are false

वनम्नवलवखत कर्नों को ध्यानपूिथक पकढ़ए:


कर्न 1: सकल घरे लू उत्पार्द (GDP) एक वित्तीय िषथ के र्दौरान अर्थव्यिस्र्ा के सभी क्षेत्रों द्वारा जोड़े गए सभी अंवतम िस्तुओं और सेिाओं
के सकल बाजार मूल्य का योग है।
कर्न 2: बाजार मूल्य पर सकल मूल्य िर्िथत (GVAMP) मध्यिती उपभोग पर उत्पार्द के मूल्य की अविकता के बराबर है।
कर्दए गए कर्नों के सन्द्र्दभथ में, वनम्नवलवखत में से सही विकल्प का चयन कीवजए:
विकल्प:
(A) कर्न 1 सत्य है और कर्न 2 गलत है | (B) कर्न 1 गलत है और कर्न 2 सत्य है |
(C) कर्न 1 और 2 र्दोनों सत्य हैं | (D) कर्न 1 और 2 र्दोनों गलत हैं |

3- Read the following statements – Assertion (A) and Reason (R). Choose one of the correct alternatives (1) given below:
Assertion (A): Ex-ante savings and Ex-post savings are always equal.
Reason (R): Ex-ante savings are those which all the households plan to make at different levels of income during a
period, whereas ex-post savings are the actual amount of savings made in the economy during a period.
Alternatives:
(A) Both Assertion (A) and Reason (R) are true and Reason (R) is the correct explanation of Assertion (A).
(B) Both Assertion (A) and Reason (R) are true and Reason (R) is not the correct explanation of Assertion (A).
(C) Assertion (A) is true but Reason (R) is false.
(D) Assertion (A) is false but Reason (R) is true.
वनम्नवलवखत अवभकर्न (A) और कारण (R) को ध्यानपूिथक पकढ़ए |
वनम्नवलवखत विकल्पों में से एक सही विकल्प का चुनाि कीवजए | अवभकर्न
(A): पूि-थ पूिथ बचत और पूिथ-पिात बचत हमेशा बराबर होती हैं।
कारण (R): पूि-थ पूिथ बचत िे हैं जो सभी पररिार एक अिवि के र्दौरान आय के विवभन्न स्तरों पर करने की योजना बनाते हैं, जबकक पूिथ-
पोस्ट बचत एक अिवि के र्दौरान अर्थव्यिस्र्ा में की गई बचत की िास्तविक रावश है।
विकल्प:
(A) अवभकर्न (A) और कारण (R) र्दोनों सत्य हैं और कारण (R) अवभकर्न (A) का सही स्पिीकरण है।
(B) अवभकर्न (A) और कारण (R) र्दोनों सत्य हैं और कारण (R) अवभकर्न (A) का सही स्पिीकरण नहीं है।
(C) अवभकर्न (A) सत्य है लेककन कारण (R) गलत है।
(D) अवभकर्न (A) गलत है लेककन कारण (R) सत्य है।

4- Read the following statement carefully and choose the correct alternative among those given below: (1)
Statement 1: Imports of crude oil by India will have a favorable impact on balance of payment. Statement 2: Import of
goods are recorded on the debit side of current account as it leads to outflow of foreign exchange.
In the light of the given statements, choose the correct alternative from the following:
Alternatives:
(A) Statement 1 is true and statement 2 is false (B) Statement 1 is false and statement 2 is true
(C) Both statements 1 and 2 are true (D) Both statements 1 and 2 are false

वनम्नवलवखत कर्नों को ध्यानपूिथक पकढ़ए और नीचे कर्दए गए विकल्पों में से सही विकल्प का चयन कीवजए:
कर्न 1: भारत द्वारा कच्चे तेल के आयात के विकल्प का भुगतान संतुलन पर अनुकूल प्रभाि पड़ेगा |
कर्न 2: िस्तुओं के आयात को चालू खाते के नाम पक्ष (ऋणांकन) में र्दजथ ककया जाता है क्योंकक इससे विर्देशी मुरा का बवहिाथह होता है।
कर्दए गए कर्नों के सन्द्र्दभथ में, वनम्नवलवखत में से सही विकल्प का चयन कीवजए:
विकल्प:
(A) कर्न 1 सत्य है और कर्न 2 गलत है | (B) कर्न 1 गलत है और कर्न 2 सत्य है |
(C) कर्न 1 और 2 र्दोनों सत्य हैं | (D) कर्न 1 और 2 र्दोनों गलत हैं |
OR (अर्िा)
Identify the correct pair from column I and II. (Choose the correct alternative)
Column I Column II
(a) Capital Account Surplus (i) Capital account inflow < Capital account outflow
(b) Current Account Surplus (ii) Current account inflow > current account outflow
(c) Balance Current Account (iii) Current account inflow = Capital account outflow
(d) Current Account Deficit (iv) Current account inflow ≤ Current account outflow
Alternatives:
(A) a - (i) (B) b - (ii) (C) c - (iii) (D) d - (iv)
वनम्नवलवखत कॉलम I और II से सही युग्म की पहचान कीवजए : (सही विकल्प का चयन कीवजए |)
कॉलम I कॉलम II
(a) पूंजी खाता अविशेष (i) पूंजी खाता अन्द्तरप्रिाह < पूंजी खाता बह्यप्रिाह
(b) चालू खाता अविशेष (ii) चालू खाता अन्द्तरप्रिाह > चालू खाता बह्यप्रिाह
(c) चालू खाता संतुलन (iii) चालू खाता अन्द्तरप्रिाह = पूंजी खाता बह्यप्रिाह
(d) चालू खाता घाटा (iv) चालू खाता अन्द्तरप्रिाह ≤ चालू खाता बह्यप्रिाह
विकल्प :-
(A) a - (i) (B) b - (ii) (C) c - (iii) (D) d - (iv)
5- Money supply in India may increase if, ________________ (Choose the correct alternative) (1)
1. Reserve Bank of India (RBI) injects more money in circulation
2. The commercial banks expand their credit operation
3. Tax rates are reduced by the Central Government
4. Reserve Bank of India increases the Bank Rate
Alternatives:
(A) 1, 2 and 4 are correct (B) 2, 3 and 4 are correct
(C) 1, 3 and 4 are correct (D) 1, 2 and 3 are correct
भारत में मुरा आपूर्तथ बढ़ सकती है यकर्द, ________________ (सही विकल्प का चुनाि कीवजए |)
1. भारतीय ररज़िथ बैंक (RBI) प्रचलन में अविक िन डालता है |
2. िावणवज्यक बैंक अपने ऋण संचालन का विस्तार करते हैं |
3. कें र सरकार के द्वारा कर की र्दरें कम की जाती हैं |
4. भारतीय ररजिथ बैंक के द्वारा बैंक र्दर बढ़ाई जाती है |
विकल्प:
(A) 1, 2 और 4 सही हैं | (B) 2, 3 और 4 सही हैं |
(C) 1, 3 और 4 सही हैं | (D) 1, 2 और 3 सही हैं |

6- Identify the correct pair from column I and II. (Choose the correct alternative) (1)
Column I Column II
(a) Open market operations (i) Component of fiscal policy
(b) Margin Requirement (ii) Minimum ratio of total bank deposits which they have to keep with
themselves
(c) Banker’s bank (iii) Function of Central Bank
(d) Reserve Requirement (iv) Sale and Purchase of Securities
Alternatives:
(A) a - (i) (B) b - (ii) (C) c - (iii) (D) d - (iv)

वनम्नवलवखत कॉलम I और II से सही युग्म की पहचान कीवजए : (सही विकल्प का चयन कीवजए |)
कॉलम I कॉलम II
(a) खुले बाजार की कियाएँ (i) राजकोषीय नीवत का घटक
(b) सीमांत आिश्यकताएं (ii) कु ल बैंक जमाओं का न्द्यूनतम अनुपात जो उन्द्हें अपने पास रखना होता है।
(c) बैंकों का बैंक (iii) कें रीय बैंक का कायथ
(d) आरवक्षत आिश्यकता (iv) प्रवतभूवतयों की वबिी और खरीर्द
विकल्प:
(A) a - (i) (B) b - (ii) (C) c - (iii) (D) d - (iv)
7- Read the following statements – Assertion (A) and Reason (R). Choose one of the correct (1)
alternatives given below:
Assertion (A): At break- even level of income, the value of Average Propensity of Consume (APC) is zero.
Reason (R): Sum of Average Propensity of Consume (APC) and Average Propensity of Save (APS) is always equal to one.
Alternatives:
(A) Both Assertion (A) and Reason (R) are true and Reason (R) is the correct explanation of Assertion (A).
(B) Both Assertion (A) and Reason (R) are true and Reason (R) is not the correct explanation of Assertion (A).
(C) Assertion (A) is true but Reason (R) is false.
(D) Assertion (A) is false but Reason (R) is true.

वनम्नवलवखत अवभकर्न (A) और कारण (R) को ध्यानपूिथक पकढ़ए |


वनम्नवलवखत विकल्पों में से एक सही विकल्प का चुनाि कीवजए |
अवभकर्न (A): आय के सम-लाभ स्तर पर, उपभोग की औसत प्रिृवत्त (APC) का मूल्य शून्द्य है।
कारण (R): उपभोग की औसत प्रिृवत्त (APC) और बचत की औसत प्रिृवत्त (APS) का योग हमेशा एक के बराबर होता है।
विकल्प:
(A) अवभकर्न (A) और कारण (आर) र्दोनों सत्य हैं और कारण (R) अवभकर्न (A) का सही स्पिीकरण है।
(B) अवभकर्न (A) और कारण (आर) र्दोनों सत्य हैं और कारण (R) अवभकर्न (A) का सही स्पिीकरण नहीं है।
(C) अवभकर्न (A) सत्य है लेककन कारण (R) गलत है।
(D) अवभकर्न (A) गलत है लेककन कारण (R) सत्य है।
8- Anshna, an economics teacher was discussing the concepts of following with the students. “Make in (1) India programm
will cause rise in __________________ of foreign exchange and hence
___________________ in the balance of payment situation of the country.”
(A) Supply, degradation (B) Demand, improvement
(C) Supply, improvement (D) Demand, degradation.

अर्थशास्त्र की वशवक्षका अंशना छात्रों के सार् वनम्न अििारणाओं पर चचाथ कर रही र्ीं। "मेक इन इंवडया कायथिम विर्देशी मुरा के
_________ में िृवि करे गा और इसवलए र्देश के भुगतान संतल
ु न की वस्र्वत में _________ होगा।"
(A) आपूर्तथ, वगरािट (B) मांग, सुिार
(C) आपूर्तथ, सुिार (D) मांग, वगरािट
9- The impact of “Excess Demand” under Keynesian theory of income and employment in an economy (1)
are ______________________ .
(A) Decrease in income, output, employment and general price level
(B) Decrease in nominal income, but no change in real output
(C) Increase in income, output, employment and general price level
(D) No change in output employment but increase in general price level.

कीन्द्स की आय और रोजगार अििारणा के अनुसार आविक्य मांग का अर्थव्यिस्र्ा पर _____________ प्रभाि पड़ता है |
(A) आय , उत्पार्दन , रोजगार और कीमत स्तर में कमी होगी
(B) गणनात्मक आय में कमी लेककन िास्तविक आय में कोई पररितथन नही होगा
(C) आय , उत्पार्दन , रोजगार और कीमत स्तर में िृवि होगी
(D) उत्पार्दन और रोजगार में कोई पररितथन नही लेककन सामान्द्य कीमत स्तर में िृवि होगी

10- Read the following statement carefully and choose the correct alternative among those given below. (1)
Statement 1 : Appreciation of foreign currency induces FDI from rest of the world.
Statement 2 : Appreciation of foreign currency implies depreciation of domestic currency.
In the light of the given statements, choose the correct alternative from the following:
Alternatives:
(A) Statement 1 is true and statement 2 is false (B) Statement 1 is false and statement 2 is true
(C) Both statements 1 and 2 are true (D) Both statements 1 and 2 are false

वनम्नवलवखत कर्नों को ध्यानपूिथक पकढ़ए और नीचे कर्दए गए विकल्पों में से सही विकल्प का चयन कीवजए:
कर्न 1 : विर्देशी मुरा का पुनमूथल्यन शेष विश्व से प्रत्यक्ष विर्देशी वनिेश को प्रेररत करता है।
कर्न 2 : विर्देशी मुरा के पुनमूथल्यन का तात्पयथ घरे लू मुरा के मूल्यह्रास से है।
कर्दए गए कर्नों के आलोक में, वनम्नवलवखत में से सही विकल्प का चयन कीवजए:
विकल्प:
(A) कर्न 1 सत्य है और कर्न 2 गलत है (B) कर्न 1 गलत है और कर्न 2 सत्य है
(C) कर्न 1 और 2 र्दोनों सत्य हैं (D) कर्न 1 और 2 र्दोनों गलत हैं

11-‘Current account deficit in an economy must be financed by a corresponding capital account (3)
surplus’. Do you agree with the given statement? Give valid reason(s) in support of your answer.
‘ककसी अर्थव्यिस्र्ा में चालू खाता घाटा को संबंवित पूज
ं ी खाता अविशेष द्वारा वित्तपोवषत ककया जाना चावहए' क्या आप कर्दए गए कर्न
से सहमत हैं? अपने उत्तर के समर्थन में िैि कारण/ कारणों को र्दीवजए।
OR (अर्िा)
(A) In recent times, the Indian rupee depreciated to an all-time low against the US $ .Discuss its (1.5+1.5)
impact on India’s imports.
हाल के समय में भारतीय रुपया, अमेररकन डॉलर के मुकाबले अपने वनचले स्तर पर है | भारत के आयात पर इसके प्रभाि की चचाथ
कीवजए |

(B) “A country with trade deficit cannot have current account surplus in its balance of payments”. Do you agree with given
statement? Discuss with reasons.
“व्यापार घाटे िाले र्देश के पास भुगतान संतल
ु न में चालू खाता अविशेष नहीं हो सकता है” | क्या आप कर्दए गए कर्न से सहमत हैं?
कारणों के सार् चचाथ कीवजए ।

12- Suppose in a hypothetical economy there are only two Firms A and B. Firm A sold goods for ₹ 2,000 (3) to Firm B and
purchased goods for ₹ 1,000. Firm B exported goods for ₹ 2,500 and had domestic sales of ₹ 1,500. Calculate Net Domestic
Product at market price, if consumption of fixed capital is ₹ 200.

मान लीवजए कक एक काल्पवनक अर्थव्यिस्र्ा में के िल र्दो फमथ अ और ब ही हैं | फमथ अ ने फमथ ब को ₹ 2,000 का सामान बेचा और ₹
1,000 का माल खरीर्दा। फमथ ब ने ₹ 2,500 का माल वनयाथत ककया और ₹ 1,500 की घरे लू वबिी की। यकर्द वस्र्र पूजं ी की खपत ₹ 200 है,
तो बाजार मूल्य पर शुि घरेलू उत्पार्द की गणना कीवजए I

13- What is the ‘inflationary gap’? Explain the role of reserve ratio in removing this gap. (1+3)
‘वस्फवतक अन्द्तराल' क्या है? इस अंतर को र्दूर करने में नकर्द आरवक्षत अनुपात की भूवमका स्पि कीवजए ।
OR (अर्िा)
What is the “deflationary gap”? Explain the role of reserve ratio in removing this gap.
“अपवस्फवत अन्द्तराल” क्या है? इस अंतर को र्दूर करने में आरवक्षत अनुपात की भूवमका स्पि कीवजए ।

14- For a hypothetical economy, assuming there is an increase in the Marginal Propensity to (4)
Consume (MPC) from 75% to 90% and change in investment to be ₹1,000 crore. Using the concept of investment multiplier,
calculate the increase in income due to change in Marginal Propensity to Consume (MPC).
एक काल्पवनक अर्थव्यिस्र्ा के वलए, यह मानते हुए कक सीमांत उपभोग प्रिृवत्त (MPC) में 75% से 90% तक की िृवि हुई है और वनिेश
में पररितथन ₹ 1,000 करोड़ हो गया है। वनिेश गुणक की अििारणा का उपयोग करते हुए, सीमांत उपभोग प्रिृवत्त (MPC) में पररितथन के
कारण आय में िृवि की गणना कीवजए ।

15- Read the following text carefully; discuss briefly the relevant function of the Central Bank, indicated: (4)
Recently, Reserve Bank of India (RBI) conducted a statutory inspection for supervisory evaluation against a Commercial
Bank. The commercial bank was imposed with stringent penalties, owing to deficiencies in regulatory compliances.
As per the Central Bank, the inspection revealed non-compliances vis-à-vis different directions issued by RBI, on the
following fronts:
i. ATM Card frauds
ii. Ensuring integrity and quality of data
iii. Loans to small borrowers adapted/moderated (Source-livemint.com)

वनम्नवलवखत पद्यांश को ध्यानपूिथक पकढ़ए, संकेवतत कें रीय बैंक के प्रासंवगक कायथ पर संक्षेप में चचाथ कीवजए :
हाल ही में, भारतीय ररजिथ बैंक (RBI) ने एक िावणवज्यक बैंक के वखलाफ पयथिेक्षी मूल्यांकन के वलए एक िैिावनक वनरीक्षण ककया।
वनयामक अनुपालन में कवमयों के कारण िावणवज्यक बैंक पर कड़ा जुमाथना लगाया गया र्ा। के न्द्रीय बैंक के अनुसार, वनरीक्षण में
वनम्नवलवखत मोचों पर भारतीय ररजिथ बैंक (RBI) द्वारा जारी विवभन्न वनर्देशों के गैर-अनुपालन का पता चला:
i. एटीएम काडथ िोखािड़ी
ii. डेटा की अखंडता और गुणित्ता सुवनवित करना
iii. छोटे उिारकताथओं को ऋण अनुकूवलत/ संचावलत (सौजन्द्य से -livemint.com)

16- (A) Explain the Government's Budget and its various sources of revenue. Present three actual (3+3) instances of
non-tax revenue earnings that a government could accumulate during its fiscal year.
सरकार के बजट और उसके राजस्ि के विवभन्न स्त्रोतों की व्याख्या कीवजए | एक वित्तीय िषथ के र्दौरान सरकार द्वारा जमा ककए जाने िाले
गैर –कर राजस्ि आय के तीन िास्तविक उर्दाहरण प्रस्तुत कीवजए |
(B) Outline a hypothetical scenario where a government is considering implementing a new tax policy. In this scenario,
explain how the introduction of a Progressive Tax would affect individuals with low, middle and high incomes.
एक काल्पवनक पररर्दृश्य की रुपरे खा तैयार कीवजए वजसमे सरकार एक नई कर नीवत लागू करने पर विचार कर रही है | िणथन कीवजए कक
इस पररर्दृश्य में एक प्रगवतशील कर की शुरुआत ककस प्रकार से वनम्न, मध्यम एिं उच्च आय िाले व्यवक्तयों को प्रभावित करती है |
OR (अर्िा)
Using budget estimates of government of India for the year 2008-09 calculate: (2+2+2)
(i) Revenue deficit (ii) Fiscal deficit (iii) Primary deficit
Sr. No. Receipts (Rs. In Crores)
1 Revenue receipts(=2+3) 602935
2 Tax revenue (net to centre) 507150
3 Non-tax revenue 95785
4 Capital receipts(=5+6+7) 147949
5 Recovery of loans 4497
6 Other receipts 10165
7 Borrowings and other liabilities 133287
8 Total receipts (=1+4) 750884
Expenditures (Rs. In Crores)
9 Non –plan expenditure (=10+12) 507498
10 On revenue account 448352
11 Interest payment 190807
12 On capital account 59146
13 Plan expenditure (=14+15) 243386
14 On revenue account 209767
15 On capital account 33619
16 Total expenditure (=9+13) 750884
17 Revenue expenditure (=10+14) 658119
18 Capital expenditure (=12+15) 92765

िषथ 2008-09 के वलए भारत सरकार के बजट अनुमानों का उपयोग करते हुए गणना कीवजए :
(i) राजस्ि घाटा (ii) राजकोषीय घाटा (iii) प्रार्वमक घाटा
क्र.सं. प्रावप्तयां (रूपये करोड़ों में)
1 राजस्ि प्रावप्तयाँ (=2+3) 602935
2 कर राजस्ि (कें र को वनिल) 507150
3 गैर कर राजस्ि 95785
4 पूंजीगत प्रावप्तयाँ (=5+6+7) 147949
5 ऋण की िसूली 4497
6 अन्द्य रसीर्दें 10165
7 उिार और अन्द्य र्देनर्दाररयाँ 133287
8 कु ल प्रावप्तयाँ (=1+4) 750884
व्यय (रूपये करोड़ों में)
9 गैर-योजना व्यय (=10+12) 507498
10 राजस्ि खाते पर 448352
11 ब्याज भुगतान 190807
12 पूंजी खाते पर 59146
13 योजना व्यय (=14+15) 243386
14 राजस्ि खाते पर 209767
15 पूंजी खाते पर 33619
16 कु ल व्यय (=9+13) 750884
17 राजस्ि व्यय (=10+14) 658119
18 पूंजीगत व्यय (=12+15) 92765

17- (A) In the estimation of a country's National Income, are the following items included? Provide (1+1+1) reasons for each
ककसी र्देश की राष्ट्रीय आय के आकलन में वनम्नवलवखत मर्दों को शावमल ककया जाता है? प्रत्येक के वलए कारण बताइए |
(i) Rent-free house to an employee by an employer
ककसी वनयोक्ता द्वारा ककसी कमथचारी को ककराया-मुक्त घर
(ii) Purchases by foreign tourists
विर्देशी पयथटकों द्वारा खरीर्दारी
(iii) Purchase of a truck to carry goods by a production unit
ककसी उत्पार्दन इकाई द्वारा माल ले जाने के वलए रक की खरीर्द
(B) Elaborate on the concepts of Nominal Gross National Product and Real Gross National Product with (3) examples to
illustrate the significance of these measures in economic decision-making
आर्र्थक वनणथय लेने में इन उपायों के महत्ि को स्पि करने के वलए नाममात्र सकल राष्ट्रीय उत्पार्द और िास्तविक सकल राष्ट्रीय उत्पार्द की
अििारणाओं को उर्दाहरणों के सार् समझाइए।

SECTION – B (Indian Economy)


खण्ड: ख (भारतीय अर्थव्यिस्र्ा)
18- Read the following statements: Assertion (A) and Reason (R). Choose the correct alternative from (1) those given belo
Assertion (A): The industrial sector reforms implemented under liberalization policies in a country led to an increase in
Foreign Direct Investment in the manufacturing sector.
Reason (R): Liberalization policies aimed to toughen trade barriers, regulate industries, and provide a conducive
environment for Indigenous investors, making the manufacturing sector more attractive for FDI.
Alternatives:
(A) The Assertion (A) is true, but the Reason (R) is false.
(B) The Assertion (A) is false, but the Reason (R) is true.
(C) Both the Assertion (A) and Reason (R) are true, but the Reason (R) is not the correct explanation of the assertion.
(D) Both the Assertion (A) and Reason (R) are true, and the Reason (R) is the correct explanation of the assertion

वनम्नवलवखत कर्न पकढ़ए: अवभकर्न (A) और कारण (R) । नीचे कर्दए गए विकल्पों में से सही विकल्प चुवनए |
अवभकर्न (A): ककसी र्देश में उर्दारीकरण नीवतयों के तहत लागू ककए गए औद्योवगक क्षेत्र के सुिारों से विवनमाथण क्षेत्र में प्रत्यक्ष विर्देशी
वनिेश में िृवि हुई।
कारण (R): उर्दारीकरण नीवतयों का उद्देश्य व्यापार बािाओं को सख्त करना, उद्योगों को विवनयवमत करना और स्िर्देशी वनिेशकों के
वलए अनुकूल िातािरण प्रर्दान करना है, वजससे विवनमाथण क्षेत्र को एफडीआई के वलए अविक आकषथक बनाया जा सके ।
विकल्प:
(A) अवभकर्न (A) सत्य है, लेककन कारण (R) गलत है।
(B) अवभकर्न (A) गलत है, लेककन कारण (R) सत्य है।
(C) अवभकर्न (A) और कारण (R) र्दोनों सत्य हैं, लेककन कारण (R) अवभकर्न (A) का सही स्पिीकरण नहीं है।
(D) अवभकर्न (A) और कारण (R) र्दोनों सत्य हैं, और कारण (R) अवभकर्न (A) का सही स्पिीकरण है

19- _________ and _____ are the reasons for the slowdown of the Pakistan economy since independence. (1)
I. Political instability
II. Over-dependence on remittances from abroad
III. Stable performance of agriculture sector
IV. Growth of service sector
Alternatives:
(A) I and II (B) II and III (C) III and IV (D) I and IV
आजार्दी के बार्द से पाककस्तान की अर्थव्यिस्र्ा में मंर्दी का कारण _________ और _____ हैं।
I. राजनीवतक अवस्र्रता
II. विर्देश से प्राप्त िन पर अत्यविक वनभथरता
III. कृ वष क्षेत्र का वस्र्र प्रर्दशथन
IV. सेिा क्षेत्र का विकास
विकल्प:
(A) I और II (B) II और III (C) III और IV (D) I और IV

20- From the set of the events given in Column-I and the corresponding year in Column-II, Choose the (1) incorrect pair:
Column-I Column-II
a Introduction of railways in India (i) 1853
b Incorporation of TISCO (ii) 1907
c First Official Census of India (iii) 1881
d Opening of Suez canal (iv) 1869
Alternatives:
(A) a- (i) (B) b- (ii) (C) c- (iii) (D) d- (iv)

कॉलम- I में र्दी गई घटनाओं के िम को कॉलम- II में संबंवित िषथ से वमलाते हुए, गलत युग्म का चुनाि कीवजए:
कॉलम –I कॉलम –II
a भारत में रे लिे का पररचय (i) 1853
b रटस्को का वनगमन (ii) 1907
c भारत की पहली आविकाररक जनगणना (iii) 1881
d स्िेज नहर का उद्घाटन (iv) 1869
विकल्प:
(A) a- (i) (B) b- (ii) (C) c- (iii) (D) d- (iv)

21- Match the followings: (1)


Column-I Column-II
(a) WTO (I) Provides short – term loans to balance of payment
(b) NABARD (II) Is a multilateral trade negotiating body.
(c) IMF (III) Facilitates lending for reconstruction and development
(d) IBRD (IV) Is the apex institution for rural financing in India
Choose the correct alternative from following:
(A) a-II, b-I, c-III, d-IV (B) a-II, b-IV, c-III, d-I
(C) a-II, b-III, c-IV, d-I (D) a-II, b-IV, c-I, d-III

वनम्न को सुमेवलत कीवजए :


कॉलम –I कॉलम –II
(a) डब्लू टी ओ (I) भुगतान संतुलन की समस्या को अल्पकालीन ऋण प्रर्दान करता है
(b) नाबाडथ (II) एक बहुपक्षीय व्यापार िाताथ वनकाय है
(c) आई ऍम अफ (III) पुन: वनमाथण और विकास के वलए ऋण र्देने की सुवििा प्रर्दान करता है
(d) आई बी आर डी (IV) भारत में ग्रामीण वित्त पोषण के वलए सिोच्च संस्र्ान
वनम्नवलवखत में से सही विकल्प का चुनाि कीवजए :
(A) a-II, b-I, c-III, d-IV (B) a-II, b-IV, c-III, d-I
(D) a-II, b-III, c-IV, d-I (D) a-II, b-IV, c-I, d-III

22- Read the following Assertion (A) and Reason (R). Choose the correct alternative: (1)
Assertion: Distribution of goods among people in capitalist economy happens on the basis of purchasing power.
Reason: Market forces govern the central decisions of capitalist economy.
Alternatives:
(A) Both Assertion (A) and Reason (R) are true and Reason (R) is the correct explanation of Assertion (A).
(B) Both Assertion (A) and Reason (R) are true and Reason (R) is not the correct explanation of Assertion (A).
(C) Assertion (A) is true but Reason (R) is false.
(D) Assertion (A) is false but Reason (R) is true.

वनम्नवलवखत अवभकर्न (A) और कारण (R) पढ़ें। सही विकल्प का चुनाि कीवजए :
अवभकर्न (A): पूंजीिार्दी अर्थव्यिस्र्ा में लोगों के बीच िस्तुओं का वितरण िय शवक्त के आिार पर होता है।
कारण (R): बाज़ार की ताकतें पूंजीिार्दी अर्थव्यिस्र्ा के कें रीय वनणथयों को वनयंवत्रत करती हैं।
विकल्प:
(A) अवभकर्न (A) और कारण (R) र्दोनों सत्य हैं और कारण (R) अवभकर्न (A) का सही स्पिीकरण है।
(B) अवभकर्न (A) और कारण (R) र्दोनों सत्य हैं और कारण (R) अवभकर्न (A) का सही स्पिीकरण नहीं है।
(C) अवभकर्न (A) सत्य है लेककन कारण (R) गलत है।
(D) अवभकर्न (A) गलत है लेककन कारण (R) सत्य है।

23- Read the following statements – Assertion (A) and Reason(R). Choose one of the correct alternatives (1) given below:
Assertion (A): The Aim of Great Leap Forward (GLF) campaign initiated in China was to transform agrarian economy into
a modern economy through rapid industrialisation.
Reason (R): Under Great Leap Forward campaign, students and professionals were sent to work and learn from country
side.
Alternatives:
(A) Both Assertion (A) and Reason(R) are true and Reason (R) is the correct explanation of Assertion (A)
(B) Both Assertion (A) and Reason (R) are true and Reason(R) is not the correct explanation of Assertion (A)
(C) Assertion (A) is true but Reason (R) is false.
(D) Assertion (A) is false but Reason (R) is true

वनम्नवलवखत अवभकर्न (A) और कारण (R) पढ़ें। सही विकल्प का चुनाि कीवजए :
अवभकर्न (A) : चीन में शुरू ककए गए ग्रेट लीप फॉरिडथ (GLF) अवभयान का उद्देश्य तेजी से औद्योगीकरण के माध्यम से कृ वष अर्थव्यिस्र्ा
को आिुवनक अर्थव्यिस्र्ा में बर्दलना र्ा।
कारण (R): ग्रेट लीप फॉरिडथ अवभयान के तहत, छात्रों और पेशेिरों को र्देश से काम करने और सीखने के वलए भेजा गया र्ा।
विकल्प:
(A) अवभकर्न (A) और कारण (R) र्दोनों सत्य हैं और कारण (R) अवभकर्न (A) का सही स्पिीकरण है।
(B) अवभकर्न (A) और कारण (R) र्दोनों सत्य हैं और कारण (R) अवभकर्न (A) का सही स्पिीकरण नहीं है।
(C) अवभकर्न (A) सत्य है लेककन कारण (R) गलत है।
(D) अवभकर्न (A) गलत है लेककन कारण (R) सत्य है।

24- Which of the following statements is true about unemployment? (1)


(A) Seasonal unemployment occurs when there are more people working in a job than is required
(B) Open unemployment occurs when there are more people working in a job than is required
(C) Disguised unemployment occurs when there are more people working in a job than is required
(D) Educated unemployment occurs when there are more people working in a job than is required
वनम्नवलवखत में से कौन सा कर्न बेरोजगारी के बारे में सही है?
(A) मौसमी बेरोजगारी तब होती है जब नौकरी में आिश्यकता से अविक लोग काम करते हैं
(B) खुली बेरोजगारी तब होती है जब नौकरी में जरूरत से ज्यार्दा लोग काम करते हैं
(C) प्रच्छन्न बेरोजगारी तब होती है जब नौकरी में आिश्यकता से अविक लोग काम करते हैं
(D) वशवक्षत बेरोजगारी तब होती है जब नौकरी में आिश्यकता से अविक लोग काम करते हैं
OR (अर्िा)
If new computers are being installed in a company and some employees are fired from the Job due to lack of computer
knowledge, then what kinds of unemployment will it be called _______
(Choose the correct alternative)
(A) Disguised unemployment (B) Structural unemployment
(C) Hidden unemployment (D) Frictional unemployment

यकर्द ककसी कं पनी में नए कं प्यूटर लगाए जा रहे हैं और कु छ कमथचाररयों को कं प्यूटर ज्ञान की कमी के कारण नौकरी से वनकाल कर्दया जाता
है, तो इसे ककस प्रकार की बेरोजगारी ______________ कहा जाएगा |
(सही विकल्प का चुनाि कीवजए)
(A) प्रच्छन्न बेरोजगारी (B) संरचनात्मक बेरोजगारी
(C) वछपी हुई बेरोजगारी (D) घषथणात्मक बेरोजगारी

25- Read the following statements carefully. (1)


Statement -1: On-the-job trainings help to bridge a gap between theoretical concepts and practical experiences.
Statement- 2: On-the-job trainings update the employees, with the latest changes in their work field.
In the light of the given statements, choose the correct alternative:
Alternatives:
(A) Statement 1 is true and statement 2 is false (B) Statement 1 is false and statement 2 is true
(C) Both statements 1 and 2 are true (D) Both statements 1 and 2 are false

वनम्नवलवखत कर्नों को ध्यानपूिथक पकिए।


कर्न-1: नौकरी पर प्रवशक्षण सैिांवतक अििारणाओं और व्यािहाररक अनुभिों के बीच अंतर को कम करने में मर्दर्द करता है।
कर्न- 2: नौकरी पर प्रवशक्षण कमथचाररयों को उनके कायथ क्षेत्र में निीनतम पररितथनों के सार् अद्यतन करता है।
कर्दए गए कर्नों के सन्द्र्दभथ में सही विकल्प चुवनए :
विकल्प:
(A) कर्न 1 सत्य है और कर्न 2 गलत है (B) कर्न 1 गलत है और कर्न 2 सत्य है
(C) र्दोनों कर्न 1 और 2 सत्य हैं (D) र्दोनों कर्न 1 और 2 गलत हैं

26- Read the following statements carefully. (1)


Statement- 1: Both India and Pakistan initiated their economic reforms without any external pressures.
Statement- 2: Pakistan has successfully implemented the SEZ policy and reaped its benefits using the Export Promotion
policy.
In the light of the given statements, choose the correct alternative:
Alternatives:
(A) Statement 1 is true and statement 2 is false (B) Statement 1 is false and statement 2 is true
(C) Both statements 1 and 2 are true (D) Both statements 1 and 2 are false

वनम्नवलवखत कर्नों को ध्यानपूिथक पकढ़ए ।


कर्न- 1: भारत और पाककस्तान र्दोनों ने वबना ककसी बाहरी र्दबाि के अपने आर्र्थक सुिार शुरू ककए।
कर्न- 2: पाककस्तान ने विशेष आर्र्थक क्षेत्र (SEZ) की नीवत को सफलतापूिथक लागू ककया है और वनयाथत संििथन नीवत का उपयोग करके
इसका लाभ उठाया है।
कर्दए गए कर्नों के आलोक में सही विकल्प चुवनए :
विकल्प:
(A) कर्न 1 सत्य है और कर्न 2 गलत है (B) कर्न 1 गलत है और कर्न 2 सत्य है
(C) र्दोनों कर्न 1 और 2 सत्य हैं (D) र्दोनों कर्न 1 और 2 गलत हैं

27- Under _________ in China, farmers and industrial units were required to buy and sell fixed quantities (1)
of inputs and outputs on the basis of prices fixed by the government and the rest were purchased and sold at market
prices.
(Choose the correct alternative to fill up the blank)
(A) Commune System (B) Great Leap Forward
(C) Dual Pricing (D) Great Proletarian Cultural Revolution
चीन में _________ के तहत, ककसानों और औद्योवगक इकाइयों को सरकार द्वारा वनिाथररत कीमतों के आिार पर वनविि और उत्पार्द की
वनवित मात्रा खरीर्दने और बेचने की आिश्यकता होती र्ी और बाकी को बाजार कीमतों पर खरीर्दा और बेचा जाता र्ा।
(ररक्त स्र्ान की पूर्तथ के वलए सही विकल्प का चुनाि कीवजए)
(A) कम्यून प्रणाली (B) ग्रेट लीप फॉरिडथ
(C) र्दोहरी मूल्य वनिाथरण (D) महान सिथहारा सांस्कृ वतक िांवत

28- (A) “During the colonial period, a number of socio- economic indicators were in a (3)
dilapidated state.” List any three such indicators that led to the worsening of India’s demographic profile.
"औपवनिेवशक काल के र्दौरान, कई सामावजक-आर्र्थक संकेतक जीणथ-शीणथ अिस्र्ा में र्े।" ककन्द्हीं तीन ऐसे संकेतकों की सूची बनाइए
वजनके कारण भारत की जनसांवख्यकीय रुपरे खा खराब हुई।
OR (अर्िा)
(B) “The pre-independent India’s occupational structure experienced growing regional variation.” (3)
Justify the above statement with valid explanation.
"स्ितंत्रता-पूिथ भारत की व्यािसावयक संरचना में बढ़ती क्षेत्रीय विवििता का अनुभि हुआ।" उपरोक्त कर्न को िैि स्पिीकरण
सवहत उवचत ठहराइए |

29- “Akshi was initially working as an office clerk in a firm. In the pursuit to attain position and (3) income, she attended a
few on contributed positively to her skills and expertise.” Explain the impact of Akshi’s decision on human capital formation.
“अक्षी शुरुआत में एक फमथ में ऑकफस क्लकथ के रूप में काम कर रही र्ी। पर्द और आय प्राप्त करने की चाह में, उसने अपने कौशल
और विशेषज्ञता में सकारात्मक योगर्दान र्देने के वलए कु छ कायथिमों में भाग वलया”। मानि पूंजी वनमाथण पर अक्षी के वनणथय के प्रभाि
की व्याख्या कीवजए ।

30- Enumerate four primary policy endeavors undertaken by the Chinese government subsequent to (4) attaining
independence, aimed at attaining economic advancement and industrialization. Illustrate the consequences of these
approaches on its economic progress and evolution into a significant global economic force.
स्ितंत्रता प्रावप्त के बार्द चीनी सरकार द्वारा आर्र्थक उन्नवत और औद्योगीकरण प्राप्त करने के उद्देश्य से ककए गए चार प्रार्वमक नीवतगत
प्रयासों की गणना कीवजए । इसकी आर्र्थक प्रगवत और एक महत्िपूणथ िैवश्वक आर्र्थक शवक्त के रूप में विकास पर इन र्दृविकोणों के पररणामों
का िणथन कीवजए ।

31- Analyse the recent trends in sectoral distribution of workforce in India. (4)
भारत में कायथबल के क्षेत्रीय वितरण में हाल के रुझानों का विश्लेषण कीवजए ।
Trends in Employment pattern (sector wise) 1993- 2012 (in percentage)
रोजगार के स्िरूप में रुझान (क्षेत्रिार) 1993- 2012 (प्रवतशत में)
Sector 1993-94 1999-2000 2011-2012
क्षेत्र
Primary 64 60.4 48.9
प्रार्वमक
Secondary 16 15.8 24.3
वद्वतीयक
Service 20 23.8 26.8
सेिा
32- (A) Explain the advantages and limitations of organic farming. (2+2)
जैविक खेती के फायर्दे और सीमाओं का िणथन कीवजए |
OR (अर्िा)
(B) What do you mean by rural development? Bring out the key issues in rural development. (2+2)

ग्रामीण विकास से आप क्या समझते हैं ? ग्रामीण विकास के प्रमुख मुद्दों को उजागर कीवजए ।
33- Read the following text carefully and answer the given questions on the basis of the same and common understanding:
The Green Revolution in India began in the mid-1960s marking a transition from traditional agriculture in India to high-
yielding varieties of seeds and the associated modern agricultural techniques. The need for introduction of Green
Revolution in India arose due to a shortage of food-grains in the post-independent period.
The government in the post-independent India wanted to ensure self-dependence in terms of food-grain production. Such
efforts coincided with the development of high-yielding varieties of seeds of wheat developed by Dr. Norman Borlung and
his associates in Mexico. These seeds also necessitated changes in farming techniques such as the addition of fertilizers,
pesticides and better irrigation facilities. High yielding varieties of seeds were first introduced in India in the states of
Punjab, Haryana and parts of western Uttar Pradesh.
In the early period of the green revolution in India, the focus was to acclimatise the new system with the more resource-
intensive agricultural methods. The argument for introducing the new crop varieties was to increase agricultural
production in terms of higher crop yields. The seeds introduced during the early period of the green revolution in Punjab
were not high-yielding by themselves. These high yields were possible due to the seeds being highly responsive to certain
inputs such as irrigation water and fertilizers.
The green revolution in India, thus, necessitated a resource-intensive process whereby, those who could make significant
capital investments could benefit, whereas, those others became more marginalized in regions affected by practices of
the green revolution in India. On one hand, the results derived from the green revolution helped farmers to increase their
yield and income and on the other hand, it helped the government to procure and preserve more food grains through
agencies like Food Corporation of India. These food grain reserves were helpful in creation of buffer stocks in India, which
helped in the situations of adversities.
(A) Why was Green revolution implemented and how did it benefit the farmers? (3)
(B) Justify the following statement with valid explanation: (3)
‘Green revolution enabled the government to procure sufficient food grains to build its stocks that could be used during
time of shortage’.

वनम्नवलवखत पद्यांश को ध्यानपूिथक पकढ़ए और कर्दए गए प्रश्नों का उत्तर पद्यांश एिं सामान्द्य समझ के आिार पर र्दीवजए :

भारत में हररत िांवत 1960 के र्दशक के मध्य में शुरू हुई, जो भारत में पारं पररक कृ वष से उच्च उपज र्देने िाली ककस्मों के बीजों और
संबंवित आिुवनक कृ वष तकनीकों में संिमण का प्रतीक र्ी। स्ितंत्रता के बार्द खाद्यान्न की कमी के कारण भारत में हररत िांवत लाने की
आिश्यकता उत्पन्न हुई।
स्ितंत्र भारत में सरकार खाद्यान्न उत्पार्दन के मामले में आत्मवनभथरता सुवनवित करना चाहती र्ी। इस तरह के प्रयास मेवक्सको में डॉ.
नॉमथन बोरलुंग और उनके सहयोवगयों द्वारा विकवसत गेहं के बीजों की उच्च उपज र्देने िाली ककस्मों के विकास के सार् मेल खाते हैं। इन
बीजों के कारण खेती की तकनीकों में बर्दलाि की भी आिश्यकता पड़ी, जैसे कक उिथरक, कीटनाशक और बेहतर बसंचाई सुवििाएँ। भारत में
बीजों की अविक उपज र्देने िाली ककस्मों को सबसे पहले पंजाब, हररयाणा और पविमी उत्तर प्रर्देश के कु छ वहस्सों में पेश ककया गया र्ा।
भारत में हररत िांवत के शुरुआती र्दौर में, अविक संसािन-गहन कृ वष विवियों के सार् नई प्रणाली को अनुकूवलत करने पर ध्यान कें करत
ककया गया र्ा। नई फसल ककस्मों को पेश करने का तकथ उच्च फसल पैर्दािार के संर्दभथ में कृ वष उत्पार्दन को बढ़ाना र्ा। पंजाब में हररत िां वत
के शुरुआती र्दौर में लाए गए बीज अपने आप में अविक उपज र्देने िाले नहीं र्े। बसंचाई के पानी और उिथरकों जैसे कु छ आर्दानों के प्रवत
बीजों की अत्यविक प्रवतकिया के कारण ये उच्च पैर्दािार संभि हुई।
इस प्रकार, भारत में हररत िांवत के वलए एक संसािन-गहन प्रकिया की आिश्यकता पड़ी, वजससे जो लोग महत्िपूणथ पूंजी वनिेश कर
सकते र्े, उन्द्हें लाभ हो सके , जबकक, िे अन्द्य लोग भारत में हररत िांवत की प्रर्ाओं से प्रभावित क्षेत्रों में अविक हावशए पर चले गए। एक
ओर, हररत िांवत से प्राप्त पररणामों से ककसानों को अपनी उपज और आय बढ़ाने में मर्दर्द वमली और र्दूसरी ओर, सरकार को भारतीय खाद्य
वनगम जैसी एजेंवसयों के माध्यम से अविक खाद्यान्न खरीर्दने और संरवक्षत करने में मर्दर्द वमली। ये खाद्यान्न भंडार भारत में बफर स्टॉक के
वनमाथण में सहायक र्े, वजससे विपरीत पररवस्र्वतयों में मर्दर्द वमलती र्ी।
(A) हररत िांवत क्यों लागू की गई और इससे ककसानों को क्या लाभ हुआ?
(B) वनम्नवलवखत कर्न को िैि स्पिीकरण के सार् उवचत ठहराइए:
'हररत िांवत ने सरकार को अपने भंडार बनाने के वलए पयाथप्त खाद्यान्न खरीर्दने में सक्षम बनाया वजसका उपयोग कमी के समय ककया जा
सकता र्ा।'
34- (A) Give a thorough analysis of four primary obstacles encountered by rural banking establishments (3)
when fostering financial inclusion and driving economic growth within a developing nation.
जब एक विकासशील राष्ट्र के भीतर वित्तीय समािेशन को बढ़ािा कर्दया जाता है और आर्र्थक विकास को आगे बढ़ाया जाता है
तब ग्रामीण बैंककं ग प्रवतष्ठानों के सामने आने िाली चार प्रार्वमक बािाओं का गहन विश्लेषण र्दीवजए |
(B) Why is agricultural diversification essential for sustainable livelihoods? (3)
स्र्ायी आजीविका के वलए कृ वष विवििीकरण क्यों आिश्यक है?
OR (अर्िा)
(i) How do you evaluate Start-ups in India as a solution to the problem of unemployment? Write two (3) observations.
बेरोजगारी की समस्या के समािान के रूप में आप भारत में स्टाटथ-अप का मूल्यांकन ककस प्रकार से करते हैं? र्दो प्रेक्षण वलवखए।
(ii) "The gap between the growth of GDP and Employment is widening." State the trend which (3) highlights this
phenomenon.
" सकल घरे लू उत्पार्द (GDP) और रोजगार के विकास के बीच की खाई चौड़ी हो रही है।" उस प्रिृवत्त का उल्लेख कीवजए जो इस
पररघटना को उजागर करती है।

****************************
Kendriya Vidyalaya Sangthan, Jaipur Region
SET-8
के न्द्रीय विद्यालय संगठन, जयपुर सम्भाग
Practice Paper
अभ्यास पत्र
Subject: Economics
विषय: अर्थशास्त्र
Class: XII Time: 3 Hours Max. Marks: 80
कक्षा: XII समय: 3 घंटे अविकतम अंक: 80

General instructions:
सामान्द्य वनर्देश:
(1) This question paper contains two sections:
इस प्रश्न पत्र के र्दो भाग है :-
Section–A Macro Economics
भाग अ- समवि अर्थशास्त्र
Section–B Indian Economy
भाग ब- भारतीय अर्थव्यिस्र्ा
All the questions in both the sections are compulsory. Marks for questions are indicated against each question.
र्दोनों खंडों में सभी प्रश्न अवनिायथ हैं। प्रत्येक प्रश्न के अंक उसके सामने कर्दए गए हैं।
(2) Question number 1-10 and 18-27 are very short-answer questions carrying 1 mark each. They are required to be
answered in one word or one sentence each.
प्रश्न संख्या 1–10 एिं 18–27 अवत लघु उत्तरीय प्रश्न / बहुविकल्पीय प्रश्न हैं प्रत्येक के 1 अंक हैं | उनका उत्तर एक शब्र्द या िाक्य में
कर्दया जाना चावहए ।
(3) Question number 11-12 and 28-29 are short-answer questions caring 3 marks each. Answers to them should not
normally exceed 60-80 words each.
प्रश्न संख्या 11-12 एिं 28-29 लघु उत्तरीय प्रश्न हैं वजनमें प्रत्येक के 3 अंक हैं। उनके उत्तर सामान्द्यतः 60-80 शब्र्द से अविक नहीं होने
चावहए।
(4) Question number 13-15 and 30-32 are also short-answer questions carrying 4 marks each. Answers to them should
not normally exceed 80-100 words each.
प्रश्न संख्या 13–15 एिं 30–32 भी लघु उत्तरीय प्रश्न हैं, वजनमें से प्रत्येक के 4 अंक हैं। उनके उत्तर सामान्द्यतः 80-100 शब्र्द से अविक
नहीं होना चावहए।
(5) Question number 16-17 and 33-34 are long answer questions carrying 6 marks each. Answers to them should not
normally exceed 100-150 words each.
प्रश्न संख्या 16–17 एिं 33–34 र्दीघथ उत्तरीय प्रश्न हैं वजनमें से प्रत्येक में 6 अंक हैं। उनके उत्तर सामान्द्यतः 100-150 शब्र्दों से अविक
नहीं होने चावहए।
(6) Answer should be brief and to the point and the word limit be adhered to as far as possible.
उत्तर संवक्षप्त तर्ा तथ्यात्मक होने चावहए एिं शब्र्द सीमा का यर्ासंभि पालन ककया जाना चावहए।
(7) An additional 15 minutes has been allotted to read the question paper.
इस प्रश्न पत्र को पढ़ने हेतु पंरह (15) वमनट का अवतररक्त समय आबंरटत ककया गया है |
Q.NO. Section-A (Introductory Macro Economics) Marks
प्र. सं. खण्ड-क (पररचयात्मक समवि अर्थशास्त्र) अंक
1 In the consumption function, C= 400+0.6y, the value of autonomous saving will be 1
________________.
उपभोग फलन में, C = 400 + 0.6y, स्िायत्त बचत का मूल्य होगा ____________ |
(a) 400 (b)(-)400 (c) 0.6 (d) 0.4
2 The difference between GDP at market price and GDP at factor cost is:- 1
(a) Net indirect taxes (b) Net factor income from abroad
(c) Depreciation (d) None of these
बाजार मूल्य पर सकल घरे लू उत्पार्द और कारक लागत पर सकल घरे लू उत्पार्द के मध्य अंतर है:-
(a) शुि अप्रत्यक्ष कर (b) विर्देश से शुि कारक आय
(c) मूल्य ह्रास (d) इनमें से कोई नहीं
3 Read the following statements carefully: 1
Statement 1: Slope of Saving function is indicated by Marginal Propensity to
Consume (MPC).
Statement 2: The value of Marginal Propensity to Save (MPS) can be greater than
one.
In the light of the given statements, choose the correct alternative from the
following:
Alternatives:
(a) Statement1is true and statement 2 is false.
(b) Statement 1 is false and statement 2 is true.
(c) Both statements 1and 2 are false.
(d) Both statements 1 and 2 are true.
वनम्नवलवखत कर्नों को ध्यानपूिथक पकढ़ए :
कर्न 1: बचत फलन की िाल को उपभोग की सीमांत प्रिृवत्त (MPC) द्वारा र्दशाथया है ।
कर्न 2: बचत की सीमांत प्रिृवत्त (MPS) का मान एक से अविक हो सकता है ।
कर्दए गए कर्नों के सन्द्र्दभथ में वनम्नवलवखत में से सही विकल्प का चुनाि कीवजए :
विकल्प :
(a) कर्न 1 सत्य है और कर्न 2 गलत है ।
(b) कर्न 1 गलत है और कर्न 2 सत्य है ।
(c) कर्न 1 और 2 र्दोनों गलत हैं ।
(d) कर्न 1 और 2 र्दोनों सत्य हैं ।
4 Identify the correct pair from column I and II. (Choose the correct alternative) 1
वनम्नवलवखत कॉलम I और II से सही युग्म की पहचान कीवजए : (सही विकल्प का चयन कीवजए |)
Column I Column II
कॉलम I कॉलम II
(A)Export of software to France 1. Debit side of current account
फ्ांस को सॉफ्टिेयर का वनयाथत चालू खाते का डेवबट पक्ष
(B) Import of Machinery from China 2. Capital Account of Balance of
चीन से मशीनरी का आयात Payments
भुगतान संतुलन का पूंजी खाता
(C) Remittances to relative staying 3. Debit side of Current Account of
abroad Balance of Payments
विर्देश में रहने िाले ररश्तेर्दारों को प्रेषण भुगतान संतुलन के चालू खाते का डेवबट पक्ष
(D) Investment by Apple phones firm 4.Credit side of Current Account of
in India. Balance of Payments
भारत में ऐप्पल फोन फमथ द्वारा वनिेश भुगतान संतुलन के चालू खाते का िे वडट पक्ष
(a) A -1 (b) B- 2 (c) C– 3 (d) D- 4
5 Money can be used to transfer purchasing power from present to future. This 1
specific function of money is called _________________.
(a) Store of value (b) Measure of value (c)
Standard of deferred payment (d) Medium of exchange.
िन का उपयोग ियशवक्त को ितथमान से भविष्य में स्र्ानांतररत करने के वलए ककया जा सकता है।
मुरा के इस विवशि कायथ को ________________ कहा जाता है |
(a) मूल्य का भंडार (b) मूल्य का माप
(c) स्र्वगत भुगतान का मानक (d) विवनमय का माध्यम
6 _____________________ in government expenditure component of fiscal policy 1
can be used to correct __________________ demand.
(a) Decrease, Deficient (b) Decrease, Excess
(c) Increase, Deficient (d) Both (b) and (c)
राजकोषीय नीवत के घटक सरकारी व्यय में ___________ करके ___________ माँग को
ठीक ककया जा सकता है।
(a) कमी, न्द्यून (b) कमी, अत्यविक
(c) िृवि, कमी (d) (b) और (c) र्दोनों
7 Read the following statements carefully: 1
Statement 1: Appreciation of foreign currency induces Foreign Direct Investment
(FDI) from rest of the world.
Statement 2: Appreciation of foreign currency implies Depreciation of domestic
currency.
In the light of the given statements, choose the correct alternative from the
following:
Alternatives:
(a) Statement 1 is true and statement 2 is false.
(b) Statement 1 is false and statement 2 is true.
(c) Both statements 1 and 2 are false.
(d) Both statements 1 and 2 are true
वनम्नवलवखत कर्नों को ध्यानपूिथक पकढ़ए :
कर्न 1: विर्देशी मुरा की सराहना शेष विश्व से विर्देशों से प्रत्यक्ष वनिेश (FDI) को प्रेररत करती
है।
कर्न 2: विर्देशी मुरा की सराहना का तात्पयथ घरे लू मुरा के मूल्यह्रास से है।
कर्दए गए कर्नों के आलोक में वनम्नवलवखत में से सही विकल्प का चुनाि कीवजए :
विकल्प :
(a) कर्न 1 सत्य है और कर्न 2 गलत है।
(b) कर्न 1 गलत है और कर्न 2 सत्य है।
(c) कर्न 1 और 2 र्दोनों गलत हैं ।
(d) कर्न 1 और 2 र्दोनों सत्य हैं |
8 Find the missing figures and choose the correct alternative: 1
लुप्त आकृ वतयाँ ज्ञात कीवजए और सही विकल्प का चुनाि कीवजए :
Round Deposits Loans (60%) Reserve Ratio (40%)
चि जमा ऋण आरवक्षत अनुपात
I 4000 2400 1600
II 2400 (i) 960
… … …
… … …
Total (ii) (iii) (iv)
Alternatives:
विकल्प :
(a) (i)-980, (ii)-1000, (iii)-4000, (iv)-5000
(b) (i)-1400, (ii)-9000, (iii)-3000, (iv)-6000
(c) (i)-1440, (ii)-10000, (iii)-6000, (iv)-4000
(d) (i)-840, (ii)-3000, (iii)-2000, (iv)-1000
9 What is the level of consumer spending whose disposable income (Y) is ₹ 1200, 1
given that consumption function is C = 100 + 0.75Y.
उपभोक्ता व्यय का स्तर क्या है वजसकी प्रयोज्य आय (Y) ₹ 1200 है, यह र्देखते हुए कक उपभोग
फलन C = 100 + 0.75Y है।
(a) 400 (b) 600 (c) 1200 (d) 1000
10 Balance of Payment keeps track of country’s transactions with the rest of the 1
world. There is category called “Invisible Items” which do not involve physical
goods.
Which of following is example of “Invisible Items” of BOP?
(a) People working in other country sending money home to help their families.
(b) Buying machines from another country to improve how we make products
here.
(c) Foreign investors using money to build a factory in our country.
(d) Providing computer software to a company in another country.
भुगतान संतुलन शेष विश्व के सार् र्देश के लेन र्देन पर नज़र रखता है। "अर्दृश्य िस्तुए"ँ नामक एक
िेणी है वजसमें भौवतक िस्तुएँ शावमल नहीं हैं।
वनम्नवलवखत में से कौनसा भुगतान संतुलन की "अर्दृश्य िस्तुओं" का उर्दाहरण है?
(a) र्दूसरे र्देश में काम करने िाले लोग अपने पररिार की मर्दर्द के वलए पैसे घर भेज रहे हैं।
(b) हम यहां उत्पार्द कै से बनाते हैं, इसे बेहतर बनाने के वलए र्दूसरे र्देश से मशीनें खरीर्दना।
(c) विर्देशी वनिेशक हमारे र्देश में कारखाना बनाने के वलए िन का उपयोग कर रहे हैं।
(d) र्दूसरे र्देश की ककसी कं पनी को कं प्यूटर सॉफ्टिेयर उपलब्ि कराना।
11 Explain the structure of current account of Balance of payments with valid 3
examples.
भुगतान संतुलन के चालू खाते की संरचना को िैि उर्दाहरणों सवहत समझाइए।
OR / अर्िा
How the Depreciation of Domestic currency affect Exports and Imports of Domestic
country?
घरे लू मुरा का अिमूल्यन घरे लू र्देश के वनयाथत और आयात को ककस प्रकार से प्रभावित करता
है?
12 From the following data, calculate Net National Product at Market Price (NNPMP) 3
by Income method
वनम्नवलवखत आंकड़ों से, आय विवि द्वारा बाजार मूल्य पर शुि राष्ट्रीय उत्पार्द (NNPMP) की
गणना कीवजए |
Items Rs. (In Crores)
मर्दे रूपए (करोड़ों में )
(i) Mixed income of self-employed persons 400
स्ि-रोज़गार व्यवक्तयों की वमवित आय
(ii) Compensation of employees 500
कमथचाररयों को मुआिजा
(iii) Private final consumption expenditure 900
वनजी अंवतम उपभोग व्यय
(iv) Net factor income from abroad (-)20
विर्देश से शुि कारक आय
(v) Consumption of fixed capital 100
वस्र्र पूंजी की खपत
(vi) Profits 350
लाभ
(vii) Rent 100
ककराया
(viii) Interest 150
ब्याज
(ix) Net indirect taxes 100
शुि अप्रत्यक्ष कर
13 How is Multiplier related to Marginal Propensity to Consume (MPC)? Explain. 4
If in an economy the change in Investment is ₹ 1000 and Marginal Propensity to
Save (MPS) is 0.4 then what will be change in Income?
गुणाक ककस प्रकार से उपभोग की सीमांत प्रिृवत्त (MPC) से संबंवित है? व्याख्या कीवजए |
यकर्द ककसी अर्थव्यिस्र्ा में वनिेश में पररितथन ₹ 1000 है और बचत की सीमांत प्रिृवत्त (MPS)
0.4 है तो आय में क्या पररितथन होगा?
14 In short run equilibrium of Income and Employment with AD-AS approach, what 4
will happen if AD>AS and AD<AS in the economy?
AD-AS र्दृविकोण के सार् आय और रोजगार के अल्पािवि संतुलन में, यकर्द अर्थव्यिस्र्ा में
AD>AS और AD<AS हो तो क्या होगा?
OR / अर्िा
Explain the concept of Deflationary gap with causes and consequences.
अपस्फीवत अंतराल की अििारणा को कारणों और पररणामों सवहत समझाइए।
15 Explain how Repo Rate and Reverse Repo Rate help in condition of Excess Demand 4
in Economy?
िणथन कीवजए कक रे पो र्दर और ररिसथ रे पो र्दर अर्थव्यिस्र्ा में अवतररक्त मांग की वस्र्वत में ककस
प्रकार से मर्दर्द करते हैं ?
16 From the following calculate: 2+2+2=6
वनम्नवलवखत से गणना कीवजए:
(a) Revenue Deficit
राजस्ि घाटा
(b) Fiscal Deficit and give two implications of Fiscal Deficit.
राजकोषीय घाटा और राजकोषीय घाटे के र्दो वनवहतार्थ बताइए |
S. No. Particulars Amount in Cr.
ि. सं. वििरण रकम करोड़ों में
1 Tax revenue 1000
कर राजस्ि
2 Revenue expenditure 3500
राजस्ि व्यय
3 Non-tax revenue 2000
गैर कर राजस्ि
4 Recovery of loans 145
ऋण की िसूली
5 Capital expenditure 500
पूंजीगत व्यय
6 Disinvestment 150
विवनिेश
7 Interest payments 1050
ब्याज भुगतान
OR / अर्िा
A. Explain the objective of “Redistribution of Income and wealth” of Government
Budget.
सरकारी बजट के "आय और िन के पुनर्िथतरण" के उद्देश्य को स्पि कीवजए ।
B. What is included in Capital expenditure and Revenue expenditure?
पूंजीगत व्यय और राजस्ि व्यय में क्या शावमल है?
(i) Interest payments
ब्याज भुगतान
(ii) Subsidies
सवब्सडी
(iii) Expenditure on purchasing machines
मशीनें खरीर्दने पर व्यय

17 A. State which is included in National Income and Why? 3


बताइए राष्ट्रीय आय में कौन-से सवम्मवलत है और क्यों?
(i) Expenditure by government on providing free education.
वनःशुल्क वशक्षा प्रर्दान करने पर सरकार द्वारा व्यय |
(ii) Payment of income tax by a producing unit.
उत्पार्दक इकाई द्वारा आयकर का भुगतान |
(iii) Rent free house by employer to employee.
वनयोक्ता द्वारा कमथचारी को ककराए मुक्त मकान र्देना।
B. What is the concept behind Nominal GDP and Real GDP? 3
सांकेवतक सकल घरे लू उत्पार्द और िास्तविक सकल घरे लू उत्पार्द के पीछे क्या अििारणा है?

SECTION–B (Indian Economy)


खण्ड-ख (भारतीय अर्थव्यिस्र्ा)
18 Marketed surplus refers to _______________________. 1
(a) Sale of consumer goods in the domestic market.
(b) Portion of agricultural produce sold in the market by farmers.
(c) Portion of industrial produce sold in the market.
(d) A certain proportion of both industrial and domestic produce sold in the
market.
विपवणत अविशेष का तात्पयथ ______________ से है।
(a) घरे लू बाजार में उपभोक्ता िस्तुओं की वबिी।
(b) ककसानों द्वारा बाजार में बेची गई कृ वष उपज का वहस्सा।
(c) बाज़ार में बेची जाने िाली औद्योवगक उपज का वहस्सा।
(d) बाजार में बेचे जाने िाले औद्योवगक और घरे लू उत्पार्द र्दोनों का एक वनवित अनुपात।
19 Read the following Assertion (A) and Reason (R). 1
Choose the correct alternative:
Assertion (A): Great Proletarian Cultural Revolution was launched to address
economic recession.
Reason (R): Great Leap Forward failed to accelerate the pace of development.
Alternatives:
(a) Both Assertion and reason are true and R is the correct explanation of A.
(b) Both Assertion and reason are true and R is not the correct explanation of A.
(c) Assertion is true and Reason is false.
(d) Assertion is false and Reason is true
वनम्नवलवखत अवभकर्न (A) और कारण (R) को ध्यानपूिथक पकढ़ए |
वनम्नवलवखत विकल्पों में से एक सही विकल्प का चुनाि कीवजए |
अवभकर्न (A): आर्र्थक मंर्दी से वनपटने के वलए महान सिथहारा सांस्कृ वतक िांवत शुरू की गई र्ी।
कारण(R): ग्रेट लीप फॉरिडथ विकास की गवत को तेज करने में विफल रहा।
विकल्प:
(a) अवभकर्न (A) और कारण (R) र्दोनों सत्य हैं और कारण (R) अवभकर्न (A) का सही
स्पिीकरण है।
(b) अवभकर्न (A) और कारण (R) र्दोनों सत्य हैं और कारण (R) अवभकर्न (A) का सही
स्पिीकरण नहीं है।
(c) अवभकर्न (A) सत्य है लेककन कारण (R) गलत है।
(d) अवभकर्न (A) गलत है लेककन कारण (R) सत्य है।
20 The Economic reforms under the New Economic Policy of 1991 were made so that 1
________________________.
(a) Market forces would drive the economy towards growth and development
(b) The government would drive the economy towards growth and development.
(c) The public would drive the economy towards growth and development.
(d) None of the above
1991 की नई आर्र्थक नीवत के अंतगथत आर्र्थक सुिार इस प्रकार ककए गए कक _____________
(a) बाजार की ताकतें अर्थव्यिस्र्ा को िृवि और विकास की ओर ले जाएंगी |
(b) सरकार अर्थव्यिस्र्ा को िृवि और विकास की ओर ले जाएगी।
(c) जनता अर्थव्यिस्र्ा को िृवि और विकास की ओर ले जाएगी।
(d) उपरोक्त में से कोई नहीं
21 When extraction is above the rate of regeneration of the resource and the wastage 1
are outside the absorptive capacity of environment, then environment fails to
perform its one of the vital function. What is that function?
(a) It supplies resources. (b) It assimilates waste.
(c) It sustains life. (d) It provide aesthetic service.
जब वनष्कषथण, संसािन के पुनजथनन की र्दर से ऊपर होता है और अपव्यय पयाथिरण की अिशोषण
क्षमता से बाहर होता है, तो पयाथिरण अपना एक महत्िपूणथ कायथ करने में विफल रहता है। िह
कायथ क्या है?
(a) यह संसािनों की आपूर्तथ करता है। (b) यह अपवशि को आत्मसात करता है।
(c) यह जीिन को कायम रखता है। (d) यह सौंर्दयथ सेिा प्रर्दान करता है।
22 Read the following statements: Assertion (A) and Reason (R). 1
Choose the correct alternative from those given below.
Assertion (A): The Navratnas were granted financial and operational autonomy in
the working of the companies.
Reason (R): The government decided to give special treatment to some profit
making PSU.
Alternatives:
(a) Both Assertion and reason are true and R is the correct explanation of A.
(b) Both Assertion and reason are true and R is not the correct explanation of A.
(c) Assertion is true and Reason is false.
(d) Assertion is false and Reason is true
वनम्नवलवखत अवभकर्न (A) और कारण (R) को ध्यानपूिथक पकढ़ए |
वनम्नवलवखत विकल्पों में से एक सही विकल्प का चुनाि कीवजए |
अवभकर्न (A): निरत्नों को कं पवनयों के कामकाज में वित्तीय और पररचालन स्िायत्तता प्रर्दान की
गई र्ी।
कारण (R): सरकार ने कु छ लाभ कमाने िाले सािथजावनक क्षेत्र के उपिमों (PSU) को विशेष
उपचार र्देने का वनणथय वलया।
विकल्प:
(a) अवभकर्न (A) और कारण (R) र्दोनों सत्य हैं और कारण (R) अवभकर्न (A) का सही
स्पिीकरण है।
(b) अवभकर्न (A) और कारण (R) र्दोनों सत्य हैं और कारण (R) अवभकर्न (A) का सही
स्पिीकरण नहीं है।
(c) अवभकर्न (A) सत्य है लेककन कारण (R) गलत है।
(d) अवभकर्न (A) गलत है लेककन कारण (R) सत्य है।
23 State the correct combinations between following columns 1
वनम्नवलवखत कॉलमों के मध्य से सही युग्म की पहचान कीवजए :
Column I Column II
कॉलम I कॉलम II
(A) 1850 (i) Introduction of railways by British
अंग्रेजों द्वारा भारत में रे लिे का पररचय
(B) 1920 (ii) Establishment of TISCO
रटस्को का वनगमन
(C) 1907 (iii) First railway bridge linking Bombay to Thane
बॉम्बे से ठाणे को जोड़ने िाला रे लिे का प्रर्म विज
(D) 1921 (iv) The year of Great divide
महान विभाजन का िषथ
Alternatives:
विकल्प :
(a) A- (i) and D-(iv) (b) B-(ii) and D-(iv)
(c) C-(iii) and A- (i) (d) C-(iii) and D-(iv)
24 Which of the following provides correct explanation of rural development? 1
(a) Economic and legal upliftment of rural area.
(b) Social and technical upliftment of rural areas.
(c) Social and legal upliftment of rural areas.
(d) Social and economic upliftment of rural area.
वनम्नवलवखत में से कौन-सा ग्रामीण विकास की सही व्याख्या प्रर्दान करता है?
(a) ग्रामीण क्षेत्र का आर्र्थक एिं कानूनी उत्र्ान।
(b) ग्रामीण क्षेत्रों का सामावजक एिं तकनीकी उत्र्ान।
(c) ग्रामीण क्षेत्रों का सामावजक एिं कानूनी उत्र्ान।
(d) ग्रामीण क्षेत्र का सामावजक एिं आर्र्थक उत्र्ान।
25 In terms of sectoral contribution to GDP, the economy of China is relying more on 1
___________.
सकल घरे लू उत्पार्द में क्षेत्रीय योगर्दान के मामले में चीन की अर्थव्यिस्र्ा ___________ पर
अविक वनभथर है |
(a) Primary sector (b) Secondary sector
प्राइमरी क्षेत्र वद्वतीयक क्षेत्र
(c)Tertiary sector (d) None of these
तृतीय क्षेत्र इनमें से कोई नहीं
26 Read the following statements: Assertion (A) and Reason (R). 1
Choose the correct alternative from those given below
Assertion (A): High growth alone is not sufficient to reduce unemployment.
Without the active participation of the poor, successful implementation of any
program is not possible.
Reason (R): Over the years there have been reduction in the gender wage parity
and in the female labour force participation rate.
Alternatives:
(a) Both Assertion and reason are true and R is the correct explanation of A.
(b) Both Assertion and reason are true and R is not the correct explanation of A.
(c) Assertion is true and Reason is false.
(d) Assertion is false and Reason is true
वनम्नवलवखत अवभकर्न (A) और कारण (R) को ध्यानपूिथक पकढ़ए |
वनम्नवलवखत विकल्पों में से एक सही विकल्प का चुनाि कीवजए |
अवभकर्न (A): अके ले उच्च विकास बेरोजगारी को कम करने के वलए पयाथप्त नहीं है। गरीबों की
सकिय भागीर्दारी के वबना ककसी भी कायथिम का सफल कियान्द्ियन संभि नहीं है।
कारण (R): वपछले कु छ िषों में बलंग िेतन समानता और मवहला िम बल भागीर्दारी र्दर में
कमी आई है।
विकल्प:
(a) अवभकर्न (A) और कारण (R) र्दोनों सत्य हैं और कारण (R) अवभकर्न (A) का सही
स्पिीकरण है।
(b) अवभकर्न (A) और कारण (R) र्दोनों सत्य हैं और कारण (R) अवभकर्न (A) का सही
स्पिीकरण नहीं है।
(c) अवभकर्न (A) सत्य है लेककन कारण (R) गलत है।
(d) अवभकर्न (A) गलत है लेककन कारण (R) सत्य है।
27 Identify the correctly matched pair in column A and column B from the following: 1
वनम्नवलवखत में से कॉलम A और कॉलम B में सही सुमेवलत जोड़ों की पहचान कीवजए :
Column A Column B
कॉलम A कॉलम B
1.Planning commission (A) Apex body to planning
योजना आयोग योजना बनाने के वलए शीषथ वनकाय
2.Characteristic of economic planning (B) Maximum utilization of resources
आर्र्थक वनयोजन की विशेषता संसािनों का अविकतम उपयोग
3.Objectiveofeconomic planning (C) Economic instability
आर्र्थक वनयोजन का उद्देश्य आर्र्थक अवस्र्रता
4.National development council (D) NITI Aayog
राष्ट्रीय विकास पररषर्द राष्ट्रीय विकास पररषर्द
Alternatives:
विकल्प:
(a) 1- (A) (b) 2 – (B) (c) 3 – (C) (d) 4 – (D)
28 Mention key features of India’s Foreign trade during eve of Independence with 3
examples to show that there was Drain of Wealth from India and in which
directions was drain of wealth?
स्ितंत्रता की पूिथ संध्या के र्दौरान भारत के विर्देशी व्यापार की प्रमुख विशेषताओं का उर्दाहरण
सवहत उल्लेख कीवजए वजससे पता चल सके कक भारत से िन का वनकास हुआ र्ा और ककन
कर्दशाओं में िन का वनकास हुआ र्ा?
OR / अर्िा
How has the shift from inward looking strategy to outward looking strategy
impacted India’s exports in the post NEP era?
निीन आर्र्थक नीवत युग के बार्द अंतमुथखी रणनीवत से बहुमुथखी रणनीवत में बर्दलाि ने भारत के
वनयाथत को ककस प्रकार से प्रभावित ककया ?
29 Explain any two sources of Human Capital formation in context of Economic 3
Development. Explain how investment in Human capital will affect country’s
Economic Growth.
आर्र्थक विकास के संर्दभथ में मानि पूंजी वनमाथण के ककन्द्हीं र्दो स्रोतों की व्याख्या कीवजए । िणथन
कीवजए कक मानि पूंजी में वनिेश र्देश की आर्र्थक िृवि को ककस प्रकार से प्रभावित करे गा |
30 Compare and analyse the given data of India and China with valid reasons 4
प्रर्दान ककए गए आंकड़ों को िैि कारणों के सार् तुलना और विश्लेषण कीवजए |
Country Annual growth rate of population (2015) Gender ratio (per
राष्ट्र जनसंख्या की िार्षथक िृवि र्दर (2015) thousand males)
बलंगानुपात (प्रवत हजार पुरुष)
India 1.2% 929
भारत
China 0.5% 941
चीन
India and China have many similarities in their development strategies. Elaborate
two points.
भारत और चीन की विकास रणनीवतयों में कई समानताएं हैं। र्दो बबंर्दओं ु का विस्तरण कीवजए |
31 A. Critically evaluate two challenges faced by rural banking institutions in 2+2=4
increasing financial responsibility.
वित्तीय वजम्मेर्दारी बढ़ाने में ग्रामीण बैंककं ग संस्र्ानों के सामने आने िाली र्दो चुनौवतयों का
आलोचनात्मक मूल्यांकन कीवजए |
B. What is the role of Organic farming in improving conditions of rural areas and
what is its relevance for environment?
ग्रामीण क्षेत्रों की वस्र्वत सुिारने में जैविक खेती की क्या भूवमका है और पयाथिरण के वलए
इसकी प्रासंवगकता क्या है?
OR / अर्िा
Explain four key measures initiated by government in developing country to
enhance efficiency and effectiveness of agricultural marketing system.
विकासशील र्देशों में कृ वष विपणन प्रणाली की र्दक्षता और प्रभािशीलता बढ़ाने के वलए सरकार
द्वारा अपनाए गए चार प्रमुख उपायों की व्याख्या कीवजए ।
32 A. “In India women participation rate is lower in urban areas than in rural areas”. 2+2=4
Explain with valid arguments.
"भारत में शहरी क्षेत्रों में मवहलाओं की भागीर्दारी र्दर ग्रामीण क्षेत्रों की तुलना में कम है"।
उवचत तकों के सार् समझाइए।
B. Define worker population ratio with its significance.
िवमक जनसंख्या अनुपात को उसके महत्ि सवहत पररभावषत कीवजए ।
33 A. What was the main features of Industrial Policy Resolution of 1956? 3+3=6
1956 की औद्योवगक नीवत संकल्प की मुख्य विशेषताएं क्या र्ीं?
B. Give description of Infrastructure Development during British rule and analyse
their impact on transportation and commercialization.
विरटश शासन के र्दौरान बुवनयार्दी िांचे के विकास का वििरण र्दीवजए और पररिहन और
व्यािसायीकरण पर उनके प्रभाि का विश्लेषण कीवजए |
OR / अर्िा
A. What were the financial sector reforms in India after Liberalisation?
उर्दारीकरण के बार्द भारत में वित्तीय क्षेत्र में कौन-से सुिार हुए?
B. Explain the need and type of Land reforms implemented in the agriculture
sector.
कृ वष क्षेत्र में लागू भूवम सुिारों की आिश्यकता एिं प्रकार की व्याख्या कीवजए ।
34 Read the following text carefully and answer the given questions on the basis of
the same and common understanding:
Sustainable development is a development that meets the need of the present,
without compromising the ability of the future generation to meet their own
needs. Sustainable Development is synonymous in the minds of many with the
colour green and for good reason. 20 years ago at the first Earth Summit in Rio de
Janerio, leaders set out what today is conventional wisdom human progress-both
social and economic cannot be divorced from environmental protection. It is as
much about health, education and job, as it is about ecosystem. It is about ever
widening inclusion and movement away from decisions that Erode democratic
space and do not at rest social inequality, intolerance and violence.
The Indian government has accelerated the pace of green growth as India's facing
the grave reality of depleting natural resources, limited supply of water, minerals
and fossil fuels. In a bid to counter the climatic threat, India is committed to
achieve net zero by 2070, released a low carbon development strategy and
introduced the concept of LIFE- (Lifestyle for Environment) to protect responsible
consumption.
The green growth action includes measures like steady de-carbonization of Indian
industries, reduce dependency on fossil fuels imports and establish technology and
market leadership in this sunrise sector.
To encourage the optimal use of wetlands, enhance biodiversity, carbon stock, Eco-
tourism opportunities and income generation for local communities, the Amrit
Dharohar scheme will be implemented over 3 years.

वनम्नवलवखत पद्यांश को ध्यानपूिथक पकढ़ए और कर्दए गए प्रश्नों का उत्तर पद्यांश एिं सामान्द्य
समझ के आिार पर र्दीवजए :
सतत िारणीय विकास िह विकास है जो भविष्य की पीढ़ी की अपनी जरूरतों को पूरा करने की
क्षमता से समझौता ककए वबना ितथमान की जरूरतों को पूरा करता है। सतत िारणीय विकास कई
लोगों के मन में अच्छे कारण से भी हरे रं ग का पयाथय है । 20 साल पहले ररयो डी जेनेररयो में पहले
पृथ्िी वशखर सम्मेलन में नेताओं ने बताया र्ा कक आज मानि प्रगवत का पारं पररक ज्ञान क्या है –
सामावजक और आर्र्थक र्दोनों को पयाथिरण संरक्षण से अलग नहीं ककया जा सकता है। यह वजतना
स्िास्थ्य, वशक्षा और नौकरी के बारे में है, उतना ही पाररवस्र्वतकी तंत्र के बारे में भी है । यह
वनरं तर व्यापक समािेशन और उन वनणथयों से र्दूर जाने के बारे में है जो लोकतांवत्रक स्र्ान को नि
करते हैं और सामावजक असमानता, असवहष्णुता और बहंसा को शांत नहीं करते हैं।
भारत सरकार ने हररत विकास की गवत को तेज कर कर्दया है क्योंकक भारत घटते प्राकृ वतक
संसािनों, पानी, खवनज और जीिाश्म ईंिन की सीवमत आपूर्तथ की गंभीर िास्तविकता का
सामना कर रहा है। जलिायु संबंिी खतरे का मुकाबला करने के वलए, भारत 2070 कम काबथन
विकासरण नीवत जारी कर शुि शून्द्य हावसल करने के वलए प्रवतबि है, और वजम्मेर्दार खपत की
रक्षा के वलए LIFE- (पयाथिरण के वलए जीिन शैली) की अििारणा पेश की है।
हररत विकास कारथ िाई में भारतीय उद्योगों का वस्र्रडी-काबोनाइजेशन, जीिाश्म ईंिन के
आयात पर वनभथरता कम करना और इस उभरते क्षेत्र में प्रौद्योवगकी और बाजार नेतृत्ि स्र्ावपत
करना जैसे उपाय शावमल हैं।
आरथभूवम के इितम उपयोग को प्रोत्सावहत करने, जैि विवििता, काबथनस्टॉक, इको-पयथटन के
अिसरों और स्र्ानीय समुर्दायों के वलए आयसृजन को बढ़ाने के वलए, अमृत िरोहर योजना तीन
िषों में लागू की जाएगी।
(i) Define Sustainable development. 2
सतत िारणीय विकास को पररभावषत कीवजए |
(ii) Elaborate any two reasons behind objective of Green Growth being set up by 2
Indian Government.
भारत सरकार द्वारा स्र्ावपत ककए जा रहे हररत विकास के उद्देश्य के पीछे ककन्द्हीं र्दो कारणों को
स्पि कीवजए |
(iii) Write about any two strategies of Sustainable development. 2
सतत िारणीय विकास की ककन्द्हीं र्दो रणनीवतयों के बारे में वलवखए।
Kendriya Vidyalaya Sangthan, Jaipur Region SET-9
के न्द्रीय विद्यालय संगठन, जयपुर सम्भाग
Subject: Economics
विषय: अर्थशास्त्र
Practice Paper
अभ्यास पत्र
Class: XII Time: 3 Hours Max. Marks: 80
कक्षा: XII समय : 3 घंटे अविकतम अंक: 80

General instructions:
सामान्द्य वनर्देश:
(1) This question paper contains two sections:
इस प्रश्न पत्र के र्दो भाग है :-
Section–A Macro Economics
भाग अ- समवि अर्थशास्त्र
Section–B Indian Economy
भाग ब- भारतीय अर्थव्यिस्र्ा
All the questions in both the sections are compulsory. Marks for questions are indicated against each question.
र्दोनों खंडों में सभी प्रश्न अवनिायथ हैं। प्रत्येक प्रश्न के अंक उसके सामने कर्दए गए हैं।
(2) Question number 1-10 and 18-27 are very short-answer questions carrying 1 mark each. They are required to be
answered in one word or one sentence each.
प्रश्न संख्या 1–10 एिं 18–27 अवत लघु उत्तरीय प्रश्न / बहुविकल्पीय प्रश्न हैं प्रत्येक के 1 अंक हैं | उनका उत्तर एक शब्र्द या िाक्य में
कर्दया जाना चावहए ।
(3) Question number 11-12 and 28-29 are short-answer questions caring 3 marks each. Answers to them should not
normally exceed 60-80 words each.
प्रश्न संख्या 11-12 एिं 28-29 लघु उत्तरीय प्रश्न हैं वजनमें प्रत्येक के 3 अंक हैं। उनके उत्तर सामान्द्यतः 60-80 शब्र्द से अविक नहीं होने
चावहए।
(4) Question number 13-15 and 30-32 are also short-answer questions carrying 4 marks each. Answers to them should
not normally exceed 80-100 words each.
प्रश्न संख्या 13–15 एिं 30–32 भी लघु उत्तरीय प्रश्न हैं, वजनमें से प्रत्येक के 4 अंक हैं। उनके उत्तर सामान्द्यतः 80-100 शब्र्द से अविक
नहीं होना चावहए।
(5) Question number 16-17 and 33-34 are long answer questions carrying 6 marks each. Answers to them should not
normally exceed 100-150 words each.
प्रश्न संख्या 16–17 एिं 33–34 र्दीघथ उत्तरीय प्रश्न हैं वजनमें से प्रत्येक में 6 अंक हैं। उनके उत्तर सामान्द्यतः 100-150 शब्र्दों से अविक
नहीं होने चावहए।
(6) Answer should be brief and to the point and the word limit be adhered to as far as possible.
उत्तर संवक्षप्त तर्ा तथ्यात्मक होने चावहए एिं शब्र्द सीमा का यर्ासंभि पालन ककया जाना चावहए।
(7) An additional 15 minutes has been allotted to read the question paper.
इस प्रश्न पत्र को पढ़ने हेतु पंरह (15) वमनट का अवतररक्त समय आबंरटत ककया गया है |
Q.NO. SECTION-A (MACRO ECONOMICS) MARKS
प्र. सं. भाग-अ (समवि अर्थशास्त्र) अंक
1 Read the following statements carefully: 1

Statement 1: If the machine is bought by a household, then it is a final good.


Statement 2: If the machine is bought by a firm for re-sale, then it is a final good.
In the light of the given statements, choose the correct alternative from the following:
Alternatives:
(a) Statement 1 is true and Statement 2 is false.
(b) Statement 1 is false and Statement 2 is true.
(c) Both Statement 1 and Statement 2 is true.
(d) Both Statement 1 and Statement 2 is false.
वनम्नवलवखत कर्नों को ध्यानपूिथक पकढ़ए :
कर्न 1: यकर्द एक पररिार द्वारा मशीन खरीर्दी जाती है, तो यह अंवतम िस्तु है।
कर्न 2: यकर्द एक फमथ द्वारा पुनर्िथिय के वलए मशीन खरीर्दी जाती है, तो यह एक अंवतम िस्तु है।
कर्दए गए कर्नों के सन्द्र्दभथ में, वनम्नवलवखत में से सही विकल्प का चयन कीवजए:
विकल्प:
(a) कर्न 1 सत्य है और कर्न 2 गलत है |
(b) कर्न 1 गलत है और कर्न 2 सत्य है |
(c) कर्न 1 और कर्न 2 र्दोनों सत्य हैं |
(d) कर्न 1 और कर्न 2 र्दोनों गलत हैं |
2 Read the following statement -Assertion (A) and Reason (R). 1
Choose one of the correct alternatives given below:
Assertion (A): Deficient demand gives rise to a deflationary gap.
Reason (R): Deficient demand causes a fall in the price level.
Alternatives:
(a) Both Assertion (A) and Reason (R) are true and Reason (R) is the correct
explanation of Assertion (A).
(b) Both Assertion (A) and Reason (R) are true and Reason (R) is not the correct
explanation of Assertion (A).
(c) Assertion (A) is true but Reason (R) is false.
(d) Assertion (A) is false but Reason (R) is true.
वनम्नवलवखत अवभकर्न (A) और कारण (R) को ध्यानपूिथक पकढ़ए |
वनम्नवलवखत विकल्पों में से एक सही विकल्प का चुनाि कीवजए |
अवभकर्न (A): मांग की कमी एक अपस्फीवत अंतराल को जन्द्म र्देती है।
कारण (R) : न्द्यून मांग के कारण कीमत स्तर में वगरािट आती है।
विकल्प:
(a) र्दोनों अवभकर्न (A) और कारण (R) सत्य हैं और कारण (R) अवभकर्न (A) का सही स्पिीकरण है।
(b) र्दोनों अवभकर्न (A) और कारण (R) सत्य हैं और कारण (R) अवभकर्न (A) की सही व्याख्या नहीं है।
(c) अवभकर्न (A) सच है लेककन कारण (R) गलत है।
(d) अवभकर्न (A) गलत है लेककन कारण (R) सही है।
3 “Savers suffer as inflation rises to 10%” which function of money is most 1
involved in the statement-
(a) Medium of exchange (b) Unit of account
(c) Store of value (d) Transfer of value
“मुरास्फीवत 10% तक बढ़ने से बचतकताथ पीवड़त होते हैं" कर्न में मुरा का कौन सा कायथ सबसे अविक शावमल
है-
(a) विवनमय का माध्यम (b) खाते की इकाई
(c) मूल्य का भंडारण (d) मूल्य का हस्तांतरण
4 Read the following statements carefully: 1
Statement 1: Purchase of a firm by Reliance Group in USA is example of Foreign
Direct Investment.
Statement 2: Purchase of shares of a foreign company by the Reliance Group is
example of Portfolio Investment.
In the light of the given statements, choose the correct alternative from the following:
Alternatives:
(a) Statement 1 is true and Statement 2 is false.
(b) Statement 1 is false and Statement 2 is true.
(c) Both Statement 1 and Statement 2 is true.
(d) Both Statement 1 and Statement 2 is false.
वनम्नवलवखत कर्नों को ध्यानपूिथक पकढ़ए :
कर्न 1: संयुक्त राज्य अमेररका में ररलायंस समूह द्वारा एक फमथ की खरीर्द प्रत्यक्ष विर्देशी वनिेश का उर्दाहरण
है |
कर्न 2: ररलायंस समूह द्वारा ककसी विर्देशी कं पनी के शेयरों की खरीर्द संविभाग वनिेश का उर्दाहरण है |
कर्दए गए कर्नों के सन्द्र्दभथ में, वनम्नवलवखत में से सही विकल्प का चयन कीवजए:
विकल्प:
(a) कर्न 1 सत्य है और कर्न 2 गलत है |
(b) कर्न 1 गलत है और कर्न 2 सत्य है |
(c) कर्न 1 और कर्न 2 र्दोनों सत्य हैं |
(d) कर्न 1 और कर्न 2 र्दोनों गलत हैं |
5 What is the value of multiplier, when S = -100 + 0.4Y 1
गुणक का मान क्या होगा, जब S = -100 + 0.4Y
(a) 1.5 (b) 1.05 (c) 2.5 (d) 2.05
OR / अर्िा
If investment is ₹ 20 crore and C = 5 + 0.9Y, the equilibrium level of Income is:
यकर्द वनिेश ₹ 20 करोड़ है और C = 5 + 0.9Y है, तो आय का संतुलन स्तर है :
(a) ₹ 100 crore (b) ₹ 350 crore (c) ₹ 150 crore (d) ₹ 250 crore
6 Read the following statements carefully: 1
Statement 1: Receipts of unilateral transfers are put on the credit side.
Statement 2: Import of services is put on the credit side.
In the light of the given statements, choose the correct alternative from the following:
Alternatives:
(a) Statement 1 is true and Statement 2 is false.
(b) Statement 1 is false and Statement 2 is true.
(c) Both Statement 1 and Statement 2 is true.
(d) Both Statement 1 and Statement 2 is false.
वनम्नवलवखत कर्नों को ध्यानपूिथक पकढ़ए :
कर्न 1: एक तरफा हस्तांतरण की प्रावप्तयां उिार पक्ष में रखी जाती हैं।
कर्न 2: सेिाओं का आयात उिार पक्ष पर रखा गया है।
कर्दए गए कर्नों के सन्द्र्दभथ में, वनम्नवलवखत में से सही विकल्प का चयन कीवजए:
विकल्प:
(a) कर्न 1 सत्य है और कर्न 2 गलत है |
(b) कर्न 1 गलत है और कर्न 2 सत्य है |
(c) कर्न 1 और कर्न 2 र्दोनों सत्य हैं |
(d) कर्न 1 और कर्न 2 र्दोनों गलत हैं |
7 _______________ determination the value of a fixed exchanges rate. 1
(a) Speculation (b) Foreign direct investment
(c) Export and import (d) Government intervention
एक वनवित विवनमय र्दर का मूल्य _____________ वनिाथररत करता है |
(a) सट्टेबाजी (b) प्रत्यक्ष विर्देशी वनिेश
(c) वनयाथत और आयात (d) सरकारी हस्तक्षेप
OR / अर्िा
If Foreign Exchange Rate changes from $1 = ₹50 to $1 = ₹40, the money is
(a) Depreciated (b) Appreciated (c) Devaluated (d) None of these
यकर्द विर्देशी विवनमय र्दर $1 = ₹50 से $1 = ₹40 में बर्दलती है, तो रुपए में हुआ है :
(a) मूल्यह्रास (b) अवभमूल्यन (c) अिमूल्यन (d) इनमे से कोई नहीं
8 Read the following statement -Assertion (A) and Reason (R). 1
Choose one of the correct alternatives given below:
Assertion (A): When income increases, APS rises but at less rate than MPS.
Reason (R): The MPS is high in the case of the rich people.
Alternatives:
(a) Both Assertion (A) and Reason (R) are true and Reason (R) is the correct
explanation of Assertion (A).
(b) Both Assertion (A) and Reason (R) are true and Reason (R) is not the correct
explanation of Assertion (A).
(c) Assertion (A) is true but Reason (R) is false.
(d) Assertion (A) is false but Reason (R) is true.
वनम्नवलवखत अवभकर्न (A) और कारण (R) को ध्यानपूिथक पकढ़ए |
वनम्नवलवखत विकल्पों में से एक सही विकल्प का चुनाि कीवजए |
अवभकर्न (A) : जब आय में िृवि होती है तो APS बढ़ता तो है लेककन MPS से कम र्दर पर।
कारण (R) : िनी लोगों के मामले में MPS अविक होता है।
विकल्प:
(a) र्दोनों अवभकर्न (A) और कारण (R) सत्य हैं और कारण (R) अवभकर्न (A) का सही स्पिीकरण है।
(b) र्दोनों अवभकर्न (A) और कारण (R) सत्य हैं और कारण (R) अवभकर्न (A) की सही व्याख्या नहीं है।
(c) अवभकर्न (A) सच है लेककन कारण (R) गलत है।
(d) अवभकर्न (A) गलत है लेककन कारण (R) सही है।
9 During period of excess demand RBI take these following steps – 1
1. Increase Bank Rate 2. Decrease Repo Rate
3. Increase SLR 4. Sale of Securities
Alternatives:
(a) 1, 2 and 3 are correct (b) 2, 3 and 4 are correct
(c) 1, 3 and 4 are correct (d) 1, 2 and 4 are correct
अवतररक्त मांग की अिवि के र्दौरान भारतीय ररज़िथ बैंक (RBI) वनम्नवलवखत कर्दम उठाता है –
1. बैंक र्दर बढ़ाना 2. रे पो र्दर घटाना
3. एसएलआर बढ़ाना 4. प्रवतभूवतयों की वबिी
विकल्प:
(a) 1, 2 और 3 सही हैं (b) 2, 3 और 4 सही हैं
(c) 1, 3 और 4 सही हैं (d) 1, 2 और 4 सही हैं
OR / अर्िा
Which instruments of Fiscal Policy includes in the following –
1. Public Welfare 2. Public Debt
3. Taxes 4. Saving
Alternatives:
(a) 1,2 and 3 are correct (b) 2,3 and 4 are correct
(c) 1,3 and 4 are correct (d) 1,2 and 4 are correct
वनम्नवलवखत में से ककन उपकरणों को राजकोषीय नीवत में शावमल ककया जाता हैं –
1. लोक कल्याण 2. लोक ऋण 3. कर 4. बचत
विकल्प:
(a) 1,2 और 3 सही हैं (b) 2,3 और 4 सही हैं
(c) 1,3 और 4 सही हैं (d) 1,2 और 4 सही हैं
10 Which one of the following is not a current account transaction? 1
(a) Exports (b) Insurance
(c) Dividend (d) External commercial borrowings
वनम्नवलवखत में से कौन एक चालू खाता लेनर्देन नहीं है?
(a) वनयाथत (b) बीमा (c) लाभांश (d) बाहरी िावणवज्यक उिार
11 Distinguish between domestic product and national product. When can 3
domestic product be more than the national product?
घरे लू उत्पार्द और राष्ट्रीय उत्पार्द के मध्य अंतर स्पि कीवजए। घरे लू उत्पार्द राष्ट्रीय उत्पार्द से अविक कब हो
सकता है?
12 State the components of capital account of Balance of Payments. 3
भुगतान संतुलन के पूंजी खाते के घटकों का उल्लेख कीवजए।
OR / अर्िा
Distinguish between Balance of Trade and balance on current account of Balance of
Payments.
व्यापार संतुलन और भुगतान संतुलन के चालू खाते पर शेष के मध्य अन्द्तर स्पि कीवजए |
13 “Mohan deposit his money in bank and bank grant loan to Sohan”. Do you consider a 4
commercial bank ‘creator of money’ in the economy’? Explain it.
"मोहन ने अपना पैसा बैंक में जमा ककया और बैंक ने सोहन को ऋण कर्दया" क्या आप एक िावणवज्यक बैंक को
अर्थव्यिस्र्ा में 'मुरा का वनमाथता' मानते हैं? इसकी व्याख्या कीवजए |
14 C = 50 + 0.5Y is the consumption function; where C is consumption expenditure and Y 2+2=4
is national income and investment expenditure is 72000 in an economy. Calculate:
(i) Equilibrium level of national income.
(ii) Consumption expenditure at equilibrium level of national income.
C = 50 + 0.5Y उपभोग फलन है; जहाँ C उपभोग व्यय है और Y राष्ट्रीय आय है और एक अर्थव्यिस्र्ा में
वनिेश व्यय 72000 है। गणना कीवजए :
(i) राष्ट्रीय आय का संतुलन स्तर।
(ii) राष्ट्रीय आय के संतुलन स्तर पर उपभोग व्यय |
15 “Central bank plays an important role for an economy” Explain. 4
"कें रीय बैंक अर्थव्यिस्र्ा के वलए एक महत्िपूणथ भूवमका वनभाता है" व्याख्या कीवजए |
OR / अर्िा
What is a barter system? What are its drawbacks?
िस्तु विवनमय प्रणाली क्या है? इसकी कवमयां क्या हैं?
16 Calculate National Income 6
राष्ट्रीय आय की गणना कीवजए
Items ₹ in crores
मर्द (₹ करोड़ में)
i) Compensation of employees 2,000
कमथचाररयों का मुआिजा
ii) Profit 800
लाभ
iii) Rent 300
ककराया
iv) Interest 250
ब्याज
v) Mixed income of self employed 7000
स्िरोजगार की वमवित आय
vi) Net current transfers to abroad 200
विर्देशों में शुि ितथमान स्र्ानान्द्तरण
vii) Net Exports (-)100
शुि वनयाथत
viii) Net indirect taxes 1,500
शुि अप्रत्यक्ष कर
ix) Net Factor income to abroad 60
विर्देशों में वनिल कारक आय
x) Consumption of fixed capital 120
अचल पूंजी की खपत
17 Explain the meaning of the following: 2+2+2=6
(i) Revenue Deficit (ii) Fiscal Deficit (iii) Primary Deficit
वनम्नवलवखत का अर्थ स्पि कीवजए :
(i) राजस्ि घाटा (ii) राजकोषीय घाटा (iii) प्रार्वमक घाटा
OR / अर्िा
(i) Find budget deficit from the following data: 3+3=6
Items (₹ in Crore)
1. Revenue receipts 40,000
2. Revenue expenditure 30,000
3. Capital receipts 30,000
4. Capital expenditure 50,000
वनम्नवलवखत आँकड़ों से बजट घाटा ज्ञात कीवजए:
मर्द (₹ करोड़ में)
1. राजस्ि प्रावप्तयां 40,000
2. राजस्ि व्यय 30,000
3. पूंजीगत प्रावप्तयां 30,000
4. पूंजीगत व्यय 50,000
(ii) Can there be a fiscal deficit in a government budget without a revenue deficit?
क्या वबना राजस्ि घाटे के सरकारी बजट में राजकोषीय घाटा हो सकता है?
SECTION – B (Indian Economy)
खण्ड: ख (भारतीय अर्थव्यिस्र्ा)
18 Generation of large ________ surplus led to the _______of the Indian wealth. 1
Fill in the with correct alternatives:
(a) Output, High (b) Import, Drain
(c) Export, Drain (d) Capital, High
बड़े _________ अविशेष की उत्पवत्त से भारतीय िन का _________ हुआ।
सही विकल्पों से पूर्तथ कीवजए :
(a) उत्पार्द, उच्च (b) आयात, वनष्कासन
(c) वनयाथत, वनष्कासन (d) पूंजी, उच्च
19 The purpose of land ceiling was: 1
(a) To reduce the concentration of land ownership in a few hands
(b) To increase the concentration of land ownership in a few hands
(c) To increase the concentration of income in many hands
(d) To reduce the concentration of income in a few hands
भूवम चकबंर्दी का उद्देश्य र्ा:
(a) कु छ हार्ों में भूवम स्िावमत्ि की एकाग्रता को कम करने के वलए
(b) कु छ हार्ों में भूवम स्िावमत्ि की एकाग्रता बढ़ाने के वलए
(c) कई हार्ों में आय की एकाग्रता बढ़ाने के वलए
(d) कु छ हार्ों में आय की एकाग्रता को कम करने के वलए
OR / अर्िा
The portion of agricultural produce which is sold in the market by the farmers is called:
(a) Consumer Surplus (b) Marketed Surplus
(c) Export Surplus (d) Producer Surplus
कृ वष उपज का िह भाग जो ककसानों द्वारा बाजार में बेचा जाता है, कहलाता है :
(a) उपभोक्ता अविशेष (b) विपणन अविशेष
(c) वनयाथत अविशेष (d) उत्पार्दक अविशेष
20 Arrange the following events in Chronologic order and choose the correct answer from 1
the given alternatives:
(i) Establishment of people’s Republic of China
(ii) Creation of Pakistan
(iii) First Five Year Plan of India
(iv) First Five Year Plan of China
Alternatives
(a) (i), (iv), (ii), (iii) (b) (iii), (ii), (i), (iv)
(c) (ii), (i), (iii), (iv) (d) (iv), (iii), (ii), (i)
वनम्नवलवखत घटनाओं को कालानुिवमक िम में व्यिवस्र्त कीवजए और कर्दए गए विकल्पों में से सही उत्तर का
चुनाि कीवजए :
(i) पीपुल्स ररपवब्लक ऑफ चीन की स्र्ापना
(ii) पाककस्तान का वनमाथण
(iii) भारत की पहली पंचिषीय योजना
(iv) चीन की पहली पंचिषीय योजना
विकल्प
(a) (i), (iv), (ii), (iii) (b) (iii), (ii), (i), (iv)
(c) (ii), (i), (iii), (iv) (d) (iv), (iii), (ii), (i)
OR / अर्िा
Arrange the following events of China in Chronologic order and choose the correct
answer from the given alternatives:
(i) Great Proletarian Cultural Revolution
(ii) Great Leap Forward Campaign
(iii) Introduction of economic reforms
(iv) First Five Year Plan
Alternatives:
(a) (ii), (iv), (iii), (i) (b) (iv), (ii), (i), (iii)
(c) (ii), (iv), (i), (iii) (d) (iv), (i), (ii), (iii)
चीन की वनम्नवलवखत घटनाओं को कालानुिवमक िम में व्यिवस्र्त कीवजए और कर्दए गए विकल्पों में से सही
उत्तर चुवनए :
(i) महान सिथहारा सांस्कृ वतक िांवत
(ii) ग्रेट लीप फॉरिडथ अवभयान
(iii) आर्र्थक सुिारों का पररचय
(iv) प्रर्म पंचिषीय योजना
विकल्प:
(a) (ii), (iv), (iii), (i) (b) (iv), (ii), (i), (iii)
(c) (ii), (iv), (i), (iii) (d) (iv), (i), (ii), (iii)
21 Read the following statements carefully: 1
Statement 1: Self-Help Groups (SHGs) are an example of a microcredit organization
Statement 2: Rapid expansion of the banking system had no effect on rural farm and
employment.
In the light of given statements, choose the correct alternatives from the following:
Alternatives:
(a) Statement 1 is true and Statement 2 is false
(b) Statement 1 is false and Statement 2 is true
(c) Both Statement 1 and Statement 2 is true
(d) Both Statement 1 and Statement 2 is false
वनम्नवलवखत कर्नों को ध्यानपूिथक पकढ़ए :
कर्न 1: स्ियं सहायता समूह (SHG) एक सूक्ष्म साख संगठन का एक उर्दाहरण हैं
कर्न 2: बैंककं ग प्रणाली में तेजी से विस्तार का ग्रामीण खेत और रोजगार पर कोई प्रभाि नहीं पड़ा।
कर्दए गए कर्नों के सन्द्र्दभथ में, वनम्नवलवखत में से सही विकल्प का चयन कीवजए:
विकल्प:
(a) कर्न 1 सत्य है और कर्न 2 गलत है |
(b) कर्न 1 गलत है और कर्न 2 सत्य है |
(c) कर्न 1 और कर्न 2 र्दोनों सत्य हैं |
(d) कर्न 1 और कर्न 2 र्दोनों गलत हैं |
22 Read the following statement -Assertion (A) and Reason (R). 1
Choose one of the correct alternatives given below:
Assertion (A): India’s agriculture sectors registered strong performance despite the
Covid-19 pandemic.
Reason (R): The sector has got renewed thrust due to various measures on credit,
market reforms and food processing under Atma-Nirbhar Bharat announcements.
Alternatives:
(a) Both Assertion (A) and Reason (R) are true and Reason (R) is the correct
explanation of Assertion (A).
(b) Both Assertion (A) and Reason (R) are true and Reason (R) is not the correct
explanation of Assertion (A).
(c) Assertion (A) is true but Reason (R) is false.
(d) Assertion (A) is false but Reason (R) is true.
वनम्नवलवखत अवभकर्न (A) और कारण (R) को ध्यानपूिथक पकढ़ए |
वनम्नवलवखत विकल्पों में से एक सही विकल्प का चुनाि कीवजए |
अवभकर्न (A) : भारत के कृ वष क्षेत्रों ने कोविड -19 महामारी के बािजूर्द मजबूत प्रर्दशथन र्दजथ ककया |
कारण (R) : आत्म-वनभाथर भारत घोषणाओं के तहत ऋण, बाजार सुिार और खाद्य प्रसंस्करण पर विवभन्न
उपायों के कारण इस क्षेत्र को नए वसरे से बल वमला है।
विकल्प:
(a) र्दोनों अवभकर्न (A) और कारण (R) सत्य हैं और कारण (R) अवभकर्न (A) का सही स्पिीकरण है।
(b) र्दोनों अवभकर्न (A) और कारण (R) सत्य हैं और कारण (R) अवभकर्न (A) की सही व्याख्या नहीं है।
(c) अवभकर्न (A) सच है लेककन कारण (R) गलत है।
(d) अवभकर्न (A) गलत है लेककन कारण (R) सही है।
23 Diversification is related to: 1
(a) Change in cropping pattern
(b) Shift of workforce from agriculture to other allied activities
(c) Shift of workforce from agriculture to non-agriculture sector
(d) All of the above
विवििीकरण संबंवित है:
(a) फसल प्रारूप में पररितथन
(b) कृ वष से अन्द्य संबि गवतविवियों के वलए कायथबल का स्र्ानांतरण
(c) कृ वष से गैर-कृ वष क्षेत्र में कायथबल का स्र्ानांतरण
(d) उपरोक्त सभी
24 UNCED stands for ____________________________________. 1
(a) United National Conference on Ecological and Deforestation
(b) United National Conference on Environment and Development
(c) United Nations Conference on Environment Development
(d) United Nations Conference on Environment and Development
UNCED का अर्थ ______________________________________ है |
(a) पाररवस्र्वतक और िनों की कटाई पर संयुक्त राष्ट्रीय सम्मेलन |
(b) पयाथिरण और विकास पर संयुक्त राष्ट्रीय सम्मेलन |
(c) पयाथिरण विकास पर संयुक्त राष्ट्र सम्मेलन |
(d) पयाथिरण और विकास पर संयुक्त राष्ट्र सम्मेलन |
25 Read the following statement -Assertion (A) and Reason (R).
Choose one of the correct alternatives given below:
Assertion (A): In the past few decades, primary sector has created maximum jobs in
India.
Reason (R): Agriculture is not a major source of employment in urban areas.
Alternatives: 1
(a) Both Assertion (A) and Reason (R) are true and Reason (R) is the correct
explanation of Assertion (A).
(b) Both Assertion (A) and Reason (R) are true and Reason (R) is not the correct
explanation of Assertion (A).
(c) Assertion (A) is true but Reason (R) is false.
(d) Assertion (A) is false but Reason (R) is true.
वनम्नवलवखत अवभकर्न (A) और कारण (R) को ध्यानपूिथक पकढ़ए |
वनम्नवलवखत विकल्पों में से एक सही विकल्प का चुनाि कीवजए |
अवभकर्न (A) : वपछले कु छ र्दशकों में प्रार्वमक क्षेत्र ने भारत में सिाथविक रोजगार सृवजत ककए हैं।
कारण (R) : शहरी क्षेत्रों में कृ वष रोजगार का प्रमुख स्रोत नहीं है।
विकल्प:
(a) र्दोनों अवभकर्न (A) और कारण (R) सत्य हैं और कारण (R) अवभकर्न (A) का सही स्पिीकरण है।
(b) र्दोनों अवभकर्न (A) और कारण (R) सत्य हैं और कारण (R) अवभकर्न (A) की सही व्याख्या नहीं है।
(c) अवभकर्न (A) सच है लेककन कारण (R) गलत है।
(d) अवभकर्न (A) गलत है लेककन कारण (R) सही है।
26 From the set of the events given in column I and corresponding facts given in Column II 1
Column-I Column-II
i GLF Campaign A 1958
ii Economic reforms in India B 1978
iii Economic reforms in China C 1991
choose the correct alternatives:
(a) i-A, ii-B, iii-C (b) i-C, ii-A, iii-B
(c) i-A, ii-C, iii-B (d) i-C, ii-B, iii-A
स्तम्भ I में र्दी गई घटनाओं का युग्म स्तम्भ II में कर्दए गए संगत तथ्यों से कीवजए
स्तम्भ-I स्तम्भ-II
i जीएलएफ अवभयान A 1958
ii भारत में आर्र्थक सुिार B 1978
iii चीन में आर्र्थक सुिार C 1991
सही विकल्प का चुनाि कीवजए :
(a) i-A, ii-B, iii-C (b) i-C, ii-A, iii-B
(c) i-A, ii-C, iii-B (d) i-C, ii-B, iii-A
27 Read the following statements carefully: 1
Statement 1: Structural reforms were forced upon China and India by international
agencies.
Statement 2: Pakistan and India introduced structural reforms on their own initiative.
In the light of the given statements, choose the correct alternative from the following:
Alternatives:
(a) Statement 1 is true and Statement 2 is false
(b) Statement 1 is false and Statement 2 is true
(c) Both Statement 1 and Statement 2 is true
(d) Both Statement 1 and Statement 2 is false
वनम्नवलवखत कर्नों को ध्यानपूिथक पकढ़ए :
कर्न 1: अंतराथष्ट्रीय एजेंवसयों द्वारा संरचनात्मक सुिारों को चीन और भारत पर र्ोपा गया।
कर्न 2: पाककस्तान और भारत ने अपनी पहल पर संरचनात्मक सुिारों की शुरुआत की।
कर्दए गए कर्नों के सन्द्र्दभथ में, वनम्नवलवखत में से सही विकल्प का चयन कीवजए:
विकल्प:
(a) कर्न 1 सत्य है और कर्न 2 गलत है |
(b) कर्न 1 गलत है और कर्न 2 सत्य है |
(c) कर्न 1 और कर्न 2 र्दोनों सत्य हैं |
(d) कर्न 1 और कर्न 2 र्दोनों गलत हैं |
28 “The real progress of India did not mean simply the growth and expansion of industrial 3
urban centers but mainly the development of the villages.” Explain this statement.
"भारत की िास्तविक प्रगवत का मतलब के िल औद्योवगक शहरी कें रों का विकास और विस्तार नहीं र्ा, बवल्क
मुख्य रूप से गांिों का विकास र्ा।" इस कर्न की व्याख्या कीवजए |
OR / अर्िा
“Need for credit is gradually getting more pronounced with the modernization of rural
economic activities.” Explain this statement with the help of sources of Rural Credit.
"ग्रामीण आर्र्थक गवतविवियों के आिुवनकीकरण के सार् ऋण की आिश्यकता िीरे -िीरे अविक स्पि हो रही
है।" ग्रामीण साख के स्रोतों की सहायता से इस कर्न की व्याख्या कीवजए।
29 Keeping in view your locality, describe any three strategies of sustainable development. 3

अपने इलाके को ध्यान में रखते हुए सतत विकास की ककन्द्हीं तीन रणनीवतयों का िणथन कीवजए ।
30 Compare and analyses the following information related to Demographic Indicators :- 4
Demographic Indicators, 2000-01
Annual Growth
Estimated
Rate of Density Sex Fertility
Country Population Urbanisation
Population (per sq. km) Ratio Rate
(in millions)
(1990-2003)
India 1103.6 1.7 358 933 3.0 27.8
China 1303.7 1.0 138 937 1.8 36.1
Pakistan 162.4 2.5 193 922 5.1 33.4
जनसांवख्यकीय संकेतकों से संबंवित वनम्नवलवखत जानकारी की तुलना और विश्लेषण कीवजए :
जनसांवख्यकीय संकेतक, 2000-01
र्देश अनुमावनत जनसंख्या की घनत्ि बलंगानुपात प्रजनन शहरीकरण
जनसंख्या िार्षथक िृवि र्दर (प्रवत िगथ र्दर
(लाखों में) (1990-2003) कक.मी)
भारत 1103.6 1.7 358 933 3.0 27.8
चीन 1303.7 1.0 138 937 1.8 36.1
पाककस्तान 162.4 2.5 193 922 5.1 33.4
31 Explain the need and type of land reforms implemented in the agriculture sector. 4
कृ वष क्षेत्र में लागू ककए गए भूवम सुिारों की आिश्यकता और प्रकार की व्याख्या कीवजए |
OR / अर्िा
Why was it necessary for a developing country like India to follow self-reliance as a
planning objective?
योजना के उद्देश्य के रूप में भारत जैसे विकासशील र्देश के वलए आत्मवनभथरता का पालन करना क्यों आिश्यक
र्ा?
32 “Process of improvement in knowledge, skill, ability and physical capacity of the people 4
is the best way to increase GDP of the country.” Explain this statement.
लोगों के ज्ञान, कौशल, क्षमता और शारीररक क्षमता में सुिार की प्रकिया र्देश के सकल घरे लू उत्पार्द को बढ़ाने
का सबसे अच्छा तरीका है।" इस कर्न की व्याख्या कीवजए |
33 What is Green Revolution? Why was it implemented and how did it benefit the farmers? 6
Explain.
हररत िांवत क्या है? इसे क्यों लागू ककया गया और इससे ककसानों को क्या लाभ हुआ? िणथन कीवजए |
OR / अर्िा
Explain in brief:- 2+2+2=6
(a) Why did India opt for planning?
(b) Explain ‘growth with equity’
(c) Explain how import substitution can protect domestic industry.
संवक्षप्त में िणथन कीवजए :-
(a) भारत ने योजना बनाने का विकल्प का चुनाि क्यों ककया ?
(b) 'समानता के सार् विकास' की व्याख्या कीवजए |
(c) स्पि कीवजए कक आयात प्रवतस्र्ापन घरे लू उद्योग की रक्षा कै से कर सकता है।
34 Read the following text carefully and answer the given questions on the basis of the 3+3=6
same and common understanding:
As in the case of agriculture, so also in manufacturing, India could not develop a sound
industrial base under the colonial rule. Even as the country’s world famous handicraft
industries declined, no corresponding modern industrial base was allowed to come up
to take pride of place so long enjoyed by the former. The primary motive of the colonial
government behind this policy of systematically de- industrialising India was two-fold.
The intention was, first, to reduce India to the status of a mere exporter of important raw
materials for the upcoming modern industries in Britain and, second, to turn India into a
sprawling market for the finished products of those industries so that their continued
expansion could be ensured to the maximum advantage of their home country —
Britain. In the unfolding economic scenario, the decline of the indigenous handicraft
industries created not only massive unemployment in India but also a new demand in
the Indian consumer market, which was now deprived of the supply of locally made
goods. This demand was profitably met by the increasing imports of cheap
manufactured goods from Britain.
(A) What is the primary motive of the colonial government?
(B) What is causes behind the massive unemployment in India?

वनम्नवलवखत पद्यांश को ध्यान से पकढ़ए और सामान्द्य समझ के आिार पर कर्दए गए प्रश्नों के उत्तर र्दीवजए :
जैसे कृ वष के मामले में, िैसे ही विवनमाथण क्षेत्र में भी, भारत औपवनिेवशक शासन के तहत एक मजबूत
औद्योवगक आिार विकवसत नहीं कर सका। यहां तक कक र्देश के विश्व प्रवसि हस्तवशल्प उद्योगों में वगरािट
आई, ककसी भी आिुवनक औद्योवगक आिार को उस स्र्ान पर गिथ करने के वलए आने की अनुमवत नहीं र्दी गई
र्ी वजसे पूिथ में इतने लंबे समय तक आनंर्द वलया गया र्ा |औपवनिेवशक सरकार का प्रार्वमक उद्देश्य भारत को
व्यिवस्र्त रूप से औद्योगीकरण से मुक्त करने की इस नीवत के पीछे इरार्दा र्दो गुना र्ा। इरार्दा, सबसे पहले,
भारत को विटेन में आने िाले आिुवनक उद्योगों के वलए महत्िपूणथ कच्चे माल के मात्र वनयाथतक की वस्र्वत में कम
करना र्ा। और र्दूसरा, भारत को उन उद्योगों के तैयार उत्पार्दों के वलए एक विशाल बाजार में बर्दलने के वलए
ताकक उनका वनरं तर विस्तार हो सके और अपने गृह र्देश - विटेन के अविकतम लाभ के वलए सुवनवित ककया।
उभरते आर्र्थक पररर्दृश्य में, स्िर्देशी हस्तवशल्प उद्योगों के पतन ने न के िल भारत में बड़े पैमाने पर बेरोजगारी
पैर्दा की, बवल्क भारतीय उपभोक्ता बाजार में एक नई मांग भी पैर्दा की, जो अब स्र्ानीय रूप से वनर्मथत
िस्तुओं की आपूर्तथ से िंवचत र्ी। इस मांग को विटेन से सस्ते वनर्मथत माल के बढ़ते आयात से लाभप्रर्द रूप से
पूरा ककया गया।

(A) औपवनिेवशक सरकार का प्रार्वमक उद्देश्य क्या है?


(B) भारत में भारी बेरोजगारी के पीछे क्या कारण हैं?
Kendriya Vidyalaya Sangthan, Jaipur Region SET-10
Subject: Economics
Practice Paper
Class: XII Time: 3:00 Hours Maximum Marks: 80 Marks
General Instructions:
(1) This question paper contains two sections:
Section–A (Introductory Macro Economics) Section–B (Indian Economic Development)
(2) All questions in both the sections are compulsory. However, there is internal choice in some
questions. Marks for questions are indicated against each question.
(3) Question number 1 –10 and 18 – 27 are very short-answer / multiple choice questions
carrying 1 mark each. They are required to be answered in one word or one sentence each.
(4) Question number 11 – 12 and 28 – 29 are short-answer questions carrying 3 marks each.
Answers to them should normally not exceed 60-80 words each.
(5) Question number 13 – 15 and 30 – 32 are also short-answer questions carrying 4 marks
each. Answers to them should normally not exceed 80-100 words each.
(6) Question number 16 – 17 and 33 – 34 are long-answer questions carrying 6 marks each.
Answers to them should normally not exceed 100-150 words each.
(7) Answers should be brief and to the point and the above word limits should be adhered to as far
as possible.
(8) An additional 15 minutes has been allotted to read the question paper.

Q. NO. Section-A (Introductory Macro Economics) Marks


1 Under the Balance of Payments structure of a nation, the two main categories 1
of accounts for the classification of the transactions are ______ and _______.
(i) current account (iii) capital account
(ii) unilateral transfer account (iv) loan account

Identify the correct alternatives from the following:


(a) (iii) and (iv) (c) (i) and (iii)
(b) (iv) and (i) (d) (i) and (ii)
2 Which out of the following items is not included in the money supply of a 1
country?
(a) Demand deposits (c) Time deposits
(b) Coins and Currency (d) All of these

3 ‘Gifts and remittances to abroad’ are recorded on the: 1


(a) Debit side of Current Account (c) Credit side of Capital Account
(b) Credit side of Current Account (d) Debit side of Capital Account

4 Read the following statements carefully: 1


Statement 1: Accommodating items of trade are undertaken in order to
maintain the balance in the BoP account.
Statement 2: Accommodating items are net consequences of autonomous
transactions that are undertaken to correct disequilibrium in autonomous
items of BoP.
In the light of the given statements, choose the correct alternative from the
following:
(a) Statement 1 is false and statement 2 is true.
(b) Statement 1 is true and statement 2 is false.
(c) Both statements 1 and 2 are false.
(d) Both statements 1 and 2 are true.
5 Aggregate demand can be increased by: 1
(a) increasing cash reserve ratio
(b) increasing bank rate
(c) selling government securities by Reserve Bank of India
(d) None of these
6 The following information is given for an imaginary country: 1
Sl. No Transactions Rs. in Crore
(i) Imports of goods 400
(ii) Export of goods 340
(iii) Shipping 3
(iv) Travel, tourism, etc. 5
(v) Interest, dividends, profits 50
(vi) Unilateral transfers 40

The value of Balance of Trade would be :


(a) (-) 40 crores (b) 60 crores (c) 40 crores (d) (-) 60 crores

7 Read the following statements carefully: 1


Statement 1: In India, SLR and CRR are fixed by the commercial banks
themselves.
Statement 2: Secondary deposits of a commercial bank are always less than
its primary deposits.
In the light of the given statements, choose the correct alternative from the
following:
(a) Statement 1 is false and statement 2 is true.
(b) Statement 1 is true and statement 2 is false.
(c) Both statements 1 and 2 are false.
(d) Both statements 1 and 2 are true.
8 Read the following statements-Assertion (A) and Reason (R). Choose one of 1
the correct alternatives given below:
Assertion (A): Import of goods and services reflects demand of foreign
currency.
Reason (R): Import of goods and services show inflow of foreign exchange.
Alternatives:
(a) Both Assertion (A) and Reason (R) are true and Reason (R) is not the
correct explanation of Assertion (A).
(b) Both Assertion (A) and Reason (R) are true and Reason (R) is the correct
explanation of Assertion (A).
(c) Assertion (A) is false but Reason (R) is true
(d) Assertion (A) is true but Reason (R) is false
9 Choose the correct alternatives to be filled in given blanks A, B and C. 1

(a) A - Export of goods, B - Gifts, remittances and grants, C - Net non-factor


income
(b) A - Gifts, remittances and grants, B - Export of goods, C - Net non-factor
income
(c) A - Net non-factor income, B - Export of goods, C - Gifts, remittances and
grants
(d) A - Export of goods, B - Net non-factor income, C - Gifts, remittances and
grants
10 Suppose in a hypothetical economy, the income rises from 5,000 crores to 1
6,000 crores. As a result, the consumption expenditure rises from 4,000
crores to 4,600 crores. Marginal propensity to consume in such a case would
be_______.
(a) 0.8 (b) 0.6 (c) 0.4 (d) 0.2

11 It is stated that “amalgamation of many central and state taxes into a 3


single tax called ‘Goods and Services tax (GST)’ would facilitate a
common national market.” Do you agree with the statement? Justify your
answer.
12 State whether the following statements are true or false. Support your 3
answer with reason.
(a) Taxation is an effective tool to reduce the inequalities of income.
(b) Revenue deficit increases when the government fails to recover loans
forwarded to different nations.
13 On the basis of given figure, answer the questions that follow: 4
(a) Interpret the shown trends of GST collections.
(b) When was GST implemented in India?
14 "In an economy, ex-ante Aggregate Demand is more than ex-ante 4
Aggregate Supply." Elaborate the possible impact of the same, on the level of
output, income and employment.
OR
“With an objective to combat deflation, the Reserve Bank of India may
lower the lending interest rate to commercial banks.”
Discuss the rationale behind the step taken by the Reserve Bank of India.
15 Explain the role of the Reserve Bank of India as the “lender of last resort”. 4
16 (a) Calculate Net Domestic Product at Factor Cost from the given data: 4
Sl.No Items Rs. in Crore.
(i) Net factor income to abroad 30
(ii) Sales 2000
(iii) Subsidies 20
(iv) Consumption of fixed capital 50
(v) Closing stock 100
(vi) Opening stock 200
(vii) Intermediate costs 1000
(viii) Indirect Tax 150

(b) Transfer payments should not be included in the estimation of National 2


Income? Do you agree? Give reason in support of your answer.
17 (a) Giving valid reasons, explain how the following would be treated while 3
estimating domestic income?
(i) Payment made by American tourists for goods purchased in India.
(ii) Tomatoes grown by Ms. Puja in her kitchen garden.
(b) Given Nominal income, how can we find Real income? 3
SECTION–B (Indian Economic Development)
18 In______, the Great Proletarian Cultural Revolution was introduced in China. 1
(a) 1985 (b) 1965 (c) 1995 (d) 1975
19 Read the following statements carefully: 1
Statement 1: SAARC stands for South Asian Association for Regional
Cooperation.
Statement 2: ASEAN stands for Association of South-East Asian Nations.
In the light of the given statements, choose the correct alternative from the
following:
(a) Statement 1 is false and statement 2 is true.
(b) Statement 1 is true and statement 2 is false.
(c) Both statements 1 and 2 are false.
(d) Both statements 1 and 2 are true.
20 Arrange the following events of China in chronological order and choose the 1
correct alternative:
(i) Special Economic Zones
(ii) Great Proletarian Cultural Revolution
(iii)Great Leap Forward Campaign
(iv) Economic reforms were initiated
Choose the correct alternative:
(a) (iii), (ii), (iv), (i) (c) (iv), (ii), (i), (iii)
(b) (iv), (i), (ii), (iii) (d) (ii), (iv), (iii), (i)
21 Study the following picture and answer the given question: 1

Rural households take up sheep-rearing as an entrepreneurial activity. Such


kind of activities may be envisaged under ______________ diversification
activity. (Choose the correct alternative to fill up the blank)
(a) Animal husbandry (c) Horticulture
(b) Fisheries (d) Poultry
22 Read the following statements-Assertion (A) and Reason (R). Choose one of 1
the correct alternatives given below:
Assertion (A): Rural development in India is of key significance.
Reason (R): Nearly two-third of population in India is dependent on
agriculture.
Alternatives:
(a) Both Assertion (A) and Reason (R) are true and Reason (R) is not the
correct explanation of Assertion (A).
(b) Both Assertion (A) and Reason (R) are true and Reason (R) is the correct
explanation of Assertion (A).
(c) Assertion (A) is false but Reason (R) is true
(d) Assertion (A) is true but Reason (R) is false
23 From the set of events / systems given in Column I and corresponding relevant 1
fact given in column II, choose the correct pair of statement:
Column I Column II
A. The growth rate of the agriculture sector decelerated (i) 1995
to about 3% per annum.
B. A major change occurred when India adopted social (ii) after 1989
banking.
C. A thrift and credit society was started in Kerala. (iii) 1995
D. By March end more than seven lakh SHG’s been credit (iv) 2010
linked.
Alternatives:
(a) C - (iii) (c) D - (iv)
(b) A - (i) (d) B - (ii)
24 States where land reforms were successful on account of they being 1
committed to the policy of ‘land to the tiller’ are:
(a) Punjab and Kerala (c) Andhra Pradesh and West Bengal
(b) Kerala and West Bengal (d) Kerala and Andhra Pradesh

25 A person got two job offers, one from Mumbai while other from Chennai. 1
Before deciding which one to opt for, he goes through the cost and benefit
analysis. Which source of human capital formation he takes into account?
(a) Expenditure on Training (b) Expenditure on Education
(c) Expenditure on Migration (d) Expenditure on Health

26 Read the following statements—Assertion (A) and Reason (R). 1


Choose one of the correct alternatives given below:
Assertion (A): The growth rate of population of Pakistan is lowest as
compared to India and China in 2017-18.
Reason (R): China adopted a one child policy to control population in the
year 1979.
Alternatives:
(a) Both Assertion (A) and Reason (R) are true and Reason (R) is not the
correct explanation of Assertion (A).
(b) Both Assertion (A) and Reason (R) are true and Reason (R) is the correct
explanation of Assertion (A).
(c) Assertion (A) is false but Reason (R) is true
(d) Assertion (A) is true but Reason (R) is false
27 Which of the following is not true for small scale industries? 1
(a) SSI is capital intensive and therefore, increases productivity.
(b) SSI is labor intensive and therefore, employment oriented.
(c) SSI shows locational flexibility and is therefore equality oriented.
(d) SSI needs small investment and is therefore equity oriented.
28 Interpret the given picture on account of the World Trade Organisation (WTO). 3
29 “Subsidies were needed to encourage farmers.” Discuss the importance of 3
subsidies in relevance to the Indian farmers.
OR
“Small scale and cottage industries are important in the development of
the Indian economy.” Elaborate.
30 Answer the following questions on the basis of the following data: 4
Country People Below Poverty Line (%) Undernourished Children (%)
India 37 39
China 32 9
Pakistan 44 45

(a) Comment upon the People Below Poverty line among the three countries.
(b) Comment upon the percentage of Undernourished Children among China
and Pakistan.
31 Elaborate any four positive contributions made by Britishers in India. 4
OR
Throw light on the ‘economic drain’ during the colonial period.
32 Identify the ‘need for credit depicted’ in the given image. What are the 4
sources of the credit for rural farmers?

33 Elucidate the function of Pollution Control Board in India to address the 6


environmental concerns in India.
OR
(a) Give any three causes of backwardness of Indian agriculture. 3
(b) Give any three challenges facing the Indian education system. 3
34 Read the following text carefully and answer the given questions on the
basis of the same and common understanding:
Blessed with a varying climate and soil conditions, India has adopted
growing of diverse horticultural crops such as fruits, vegetables, tuber crops,
flowers, medicinal and aromatic plants, spices and plantation crops. These
crops play a vital role in providing food and nutrition, besides addressing
employment concerns. The Horticulture sector contributes nearly one-third
of the value of agriculture output and six per cent of Gross Domestic Product
of India. India has emerged as a world leader in producing a variety of fruits
like mangoes, bananas, coconuts, cashew nuts and a number of spices and is
the second largest producer of fruits and vegetables. Economic condition of
many farmers engaged in horticulture has improved and it has become a
means of improving livelihood for many unprivileged classes. Flower
harvesting, nursery maintenance, hybrid seed production and tissue culture,
propagation of fruits and flowers and food processing are highly
remunerative employment options for women in rural areas.
(a) Differentiate how the term ‘Golden Revolution’ is different from ‘Green 3
Revolution’?
(b) How horticulture as an alternative source of employment improves the 3
economic condition of farmers?
Kendriya Vidyalaya Sangthan, Jaipur Region SET-11
Subject: Economics
Practice Paper
Class: XII Time: 3:00 Hours Maximum Marks: 80 Marks
General Instructions:
(1) This question paper contains two sections:
Section–A (Introductory Macro Economics) Section–B (Indian Economic Development)
(2) All questions in both the sections are compulsory. However, there is internal choice in some
questions. Marks for questions are indicated against each question.
(3) Question number 1 –10 and 18 – 27 are very short-answer / multiple choice questions carrying
1 mark each. They are required to be answered in one word or one sentence each.
(4) Question number 11 – 12 and 28 – 29 are short-answer questions carrying 3 marks each.
Answers to them should normally not exceed 60-80 words each.
(5) Question number 13 – 15 and 30 – 32 are also short-answer questions carrying 4 marks each.
Answers to them should normally not exceed 80-100 words each.
(6) Question number 16 – 17 and 33 – 34 are long-answer questions carrying 6 marks each.
Answers to them should normally not exceed 100-150 words each.
(7) Answers should be brief and to the point and the above word limits should be adhered to as
far as possible.
(8) An additional 15 minutes has been allotted to read the question paper.

Q. NO. Section-A (Introductory Macro Economics) Marks


1 At the income level of Rs. 5000 crores, total savings are Rs.1000 crores. 1
Calculate average propensity to consume (APC):
(a) 0.1 (c) 0.8
(b) 0.2 (d) None of the above
2 The difference between GDP at market price and GDP at factor cost is:- 1
(a) Net indirect taxes (c) Depreciation
(b) Net factor income from abroad (d) None of these
3 Unilateral transfers are included in- 1
(a) Capital Account of BOP (c) Foreign Account of BOP
(b) Current Account of BOP (d) None of these
OR
Define flexible exchange rate.
4 Full employment implies absence of _________________. 1
(a) Unemployment (c) Involuntary unemployment
(b) Voluntary unemployment (d) None of these
5 Which of the following is a qualitative instrument of credit control? 1
(a) Bank rate (c) Open market operation
(b) Repo rate (d) Margin requirements
6 If consumption function in an economy is given as: C= 40 + 0.6Y, then MPS is: 1
(a) 1 (c) 0.4
(b) 0.6 (d) None of these
7 Find the true statement from the following- 1
(a) There is an inverse relation between foreign exchange rate and supply for
foreign exchange.
(b) An American purchases the goods and services from India creates demand
of foreign exchange.
(c) An Indian firm to purchase the goods and services from America creates
demand of foreign exchange.
(d) All statements are true.
8 Which one of these is a revenue expenditure: 1
(a) Purchase of shares (c) Subsidies
(b) Loans advanced (d) Expenditure on acquisition of land
9 Define money supply. 1
10 Define Capital receipts. 1
11 How will the following factor income be included in domestic factor income of 3
India? Give reasons for your answer.
1. Compensation of employees to the residents of Japan working in Indian
embassy in Japan.
2. Rent received by an Indian resident from Russian embassy in India.
3. Profits earned by a branch of state bank of India in England.
12 Explain the steps in deriving saving curve from a consumption curve. Use 3
diagram.
13 Write two differences between- 4
1. Current Account of BOP and capital account of BOP
2. Balance of Trade and Balance of Payment
14 Explain in brief the function of central bank- 4
1. Banker’s Bank
2. Issuing Authority of currency and notes
15 Giving reasons, categorize the following into revenue expenditure & capital 4
expenditure.
1. Subsidies
2. Grants given to state governments
3. Repayment of loans
4. Salary Payment
OR
Tax rates on higher income group have been increased. Explain the economic
value it reflects.
16 From the following data, calculate Net National Product at Market Price 6
(NNPMP) by -
(i) Income method (ii) Expenditure method
Items Rs. (In Crores)
(i) Mixed income of self- employed persons 400
(ii) Compensation of employees 500
(iii) Private final consumption expenditure 900
(iv) Net factor income from abroad (-) 20
(v) Net indirect taxes 100
(vi) Consumption of fixed capital 100
(vii) Net domestic capital formation 280
(viii) Net exports (-) 30
(ix) Profits 350
(x) Rent 100
(xi) Interest 150
(xii) Government final consumption expenditure 450
17 Explain the concept of deflationary gap (deficient demand). How it can be 6
controlled by government spending and taxation policy? Use diagram.
OR
In an economy S = -100 + 0.6Y is the saving function, where S is saving and Y is
national income. If investment expenditure is 1100. Calculate
1. Equilibrium level of national income.
2. Consumption expenditure at equilibrium level of national income.
SECTION–B (Indian Economic Development)
18 What do you mean by Great Leap Forward 1
19 In which year was India’s first Five year plan launched? 1
20 What percent of GDP was recommended by Education Commission (1964-66) 1
to be spent on education sector?
(a) 6% (c) 3%
(b) 5% (d) 4%
21 What is not a function of an environment? 1
(a) It supplies resources
(b) It assimilates waste
(c) It sustains life by providing genetic and bio diversity
(d) It is cause of global warming.
22 Reforms in _________________ were introduced in 1978. (India/China/Pakistan) 1
23 When was the first population data collected through a census in British India 1
(a)1901 (c)1881
(b)1871 (d)1891
24 KUTUMBSHREE is a women oriented community based poverty reduction 1
programme being implemented in which state?
(a) Rajasthan (c) Kerala
(b) Tamilnadu (d) Andhra Pradesh
25 Which of the institutional source of credit? 1
(a) Landlords (c) Money lenders
(b) Bank (d) Village traders
26 Reforms have not been able to benefit agriculture due to which of the following 1
reason
(a) Fall in public investment in agriculture infrastructure
(b) Removal of fertiliser subsidy
(c) Removal of minimum support price
(d) All of the above
27 Read the following statement – Assertion (A) and Reason (R). Choose one of 1
the correct alternative given below:
Assertion (A): Higher employment among women in rural areas (compared
with urban area) only points to widespread rural poverty in India.
Reason (R): Poverty forces women to avoid education and find opportunities
of employment in the rural areas.
Alternatives:-
(a) Both Assertion (A) and Reason (R) are true and Reason (R) is the correct
explanation of Assertion (A)
(b) Both Assertion (A) and Reason (R) are true and Reason (R) is not the
correct explanation of Assertion (A)
(c) Assertion (A) is true and Reason (R) are false.
(d) Assertion (A) is false and Reason (R) are true.
OR
Read the following statement – Assertion (A) and Reason (R). Choose one of
the correct alternative given below:
Assertion (A): Higher employment among women in rural areas (compared to
urban areas) suggests higher rural wage rate in India.
Reason (R): The bulk of female workers in rural areas are engaged in low wage
and less productive job just to make a living for their famlies.
Alternatives:-
(a) Both Assertion (A) and Reason (R) are true and Reason (R) is the correct
explanation of Assertion (A)
(b) Both Assertion (A) and Reason (R) are true and Reason (R) is not the
correct explanation of Assertion (A)
(c) Assertion (A) is true and Reason (R) are false.
(d) Assertion (A) is false and Reason (R) are true.
28 Rita is a housewife. Besides taking care of household chores, she works in the 3
cloth shop which is owned and operated by her husband. Can she be
considered a worker? Why?
29 What is a green revolution? Why was it implemented and how did it benefit the 3
farmers?
30 State the meaning of import substitution. Explain how import substitution can 4
protect the domestic industries.
31 Read carefully the given paragraph and write the answer of following
questions:-
Statistical Office (Previously it was known as National Sample Survey
Organisation) defines unemployment as a situation in which all those who,
owing to lack of work, are not working but either seek work through
employment exchanges, intermediaries, friends or relatives or by making
applications to prospective employers or express their willingness or
availability for work under the prevailing condition of work and
remunerations. Suppose a farmer has four acres of land and he actually needs
only two workers and himself to carry out various operations on his farm in a
year, but if he employs five workers and his family members such as his wife
and children, this situation is known as disguised unemployment. One study
conducted in the late 1950s showed about one-third of agriculture workers in
India as disguisedly unemployed.
(a) What is unemployment as per NSSO? 2
(b) Explain disguised unemployment. 2
32 Though public sector is very essential for industries, many public sector 4
undertaking incur huge losses and are a drain on the economy’s resources.
Discuss usefulness of public sector undertaking in the light of this fact.
33 What do you mean by rural development? Bring out the key issues in rural 6
development.
34 Mention the salient demographic indicators of China, Pakistan and India. 6
OR
What similar developmental strategies have India and Pakistan followed for
their respective developmental path? Any four points.
Kendriya Vidyalaya Sangthan, Jaipur Region
Subject: Economics SET-12
Practice Paper
Class: XII Time: 3:00 Hours Maximum Marks: 80 Marks
General Instructions:
(1) This question paper contains two sections:
Section–A (Introductory Macro Economics) Section–B (Indian Economic Development)
(2) All questions in both the sections are compulsory. However, there is internal choice in some
questions. Marks for questions are indicated against each question.
(3) Question number 1 –10 and 18 – 27 are very short-answer / multiple choice questions
carrying 1 mark each. They are required to be answered in one word or one sentence each.
(4) Question number 11 – 12 and 28 – 29 are short-answer questions carrying 3 marks each.
Answers to them should normally not exceed 60-80 words each.
(5) Question number 13 – 15 and 30 – 32 are also short-answer questions carrying 4 marks each.
Answers to them should normally not exceed 80-100 words each.
(6) Question number 16 – 17 and 33 – 34 are long-answer questions carrying 6 marks each.
Answers to them should normally not exceed 100-150 words each.
(7) Question number 16 – 17 and 33 – 34 are long-answer questions carrying 6 marks each.
Answers to them should normally not exceed 100-150 words each.
(8) An additional 15 minutes has been allotted to read the question paper.

Q. NO. Section-A (Introductory Macro Economics) Marks


1 Read the following statements carefully: 1
Statement 1: Slope of Saving function is indicated by MPC.
Statement 2: The value of marginal propensity to save can be greater than one.
In the light of the given statements, choose the correct alternative from the
following:
Alternatives:
(a) Statement 1 is true and statement 2 is false
(b) Statement 1 is false and statement 2 is true
(c) Both statements 1 and 2 are false
(d) Both statements 1 and 2 are true
2 GDP is not an appropriate indicator of welfare because of ________________. 1
(a) Externalities.
(b) Composition of GDP and distribution of GDP.
(c) Non-monetary transactions.
(d) All of the above.
3 The value of __________ can be less than, equals to, or more than one. 1
(Choose the correct alternative to fill up the blank)
(a) Marginal Propensity to Consume (c) Average Propensity to Save
(b) Average Propensity to Consume (d) None of the above

4 Under the balance of payments structure of a nation, the two main categories of 1
accounts for the classification of the transactions are __________ and __________.
(Fill up the blank with correct alternative)
(i) Current account (iii) Capital account
(ii) Unilateral transfer account (iv) Loan account

Alternatives:
(a) i and ii (c) iii and iv
(b) i and iii (d) iv and i

5 Money can be used to transfer purchasing power from present to future. Which 1
specific function of money is this called?
(a) store of value (c) measure of value
(b) standard of deferred payments (d) medium of exchange

6 Find the missing figures and choose the correct alternative: 1


Round Deposits Loans (60%) Reserve Ratio (40%)
I 2000 1200 800
II 1200 ….(i)..... 480
… … …
… … …
Total …(ii).... …(iii).... …(iv)....

Alternatives:
(a) 980,1000,4000,5000 (c) 660,4000,1000,5000
(b) 720,5000,3000,2000 (d) 640,3000,5000,1000

7 Graphically, Aggregate Supply function can be obtained by vertically adding the 1


__________________ and __________________ function.
(Choose the correct alternative to fill up the blanks)
Alternatives:
(a) consumption, saving (c) investment, saving
(b) consumption, investment (d) aggregate supply, consumption

8 Identify the correct reason(s) that may affect the supply of foreign exchange in an 1
economy.
I. Imports of visible
II. Exports of invisibles
III. Remittances by residents working abroad
IV. Purchase of assets abroad
Alternatives:
(a) I and II (c) III and IV
(b) II and III (d) I and IV
9 If C = 100 + 0.75 Y, then the corresponding saving function will be expressed as: 1
(Choose the correct alternative)
Alternatives:
(a) S = 100 + 0.25Y (c) S = -100 + 0.25Y
(b) S = -100 + 0.75Y (d) S = 75 + 0.25Y

10 Read the following chart carefully and identify the visible trade: 1

(a) Trade in services (c) Trade in goods


(b) Transfer payments (d) None of these

11 Giving reasons, state whether the following statements are true or false: 1½+1½=3
(i) Current account of Balance of Payment account records only export and import of
goods and services.
(ii) Foreign investments are recorded in the capital account of Balance of Payments.
12 On the basis of the given data, estimate the value of Domestic Income: 3
S. No. Items Amount (₹ in crores)
i Rent and royalties 1300
ii Net Indirect Taxes 200
iii Wages and salaries (in cash and kind) 1700

iv Corporate tax 400


v Depreciation 400
vi Retained earnings 300
vii Dividends 400
viii Net factor income from Abroad (-) 120
ix Mixed income of self employed 1400
x Change in stock (-) 200
OR
State the meaning of the following:
(i) National income (ii) Intermediate goods (iii) Capital loss

13 Income rises from ₹50,000 to ₹60,000, consumption increases from ₹ 40,000 to ₹ 4


48,000. In this situation, what will be the value of Marginal Propensity to consume
(MPC)?
14 (A) "In an economy, ex-ante Aggregate Demand is less than ex-ante Aggregate 4
Supply." Elaborate the possible impact of the same, on the level of output, income
and employment.
OR
(B) “With an objective to correct the deflationary gap, the Reserve Bank of India
may reduce the Reverse Repo Rate.” Discuss the rationale behind the step taken
by the Reserve Bank of India.
15 Elaborate the ‘Banker to the Government’ function performed by the Reserve Bank 4
of India.
16 (A) On the basis of the given information, calculate the value of:
(i) Fiscal deficit 3
(ii) Primary deficit 2
S. No. Content Amount (₹ in crore)
1. Revenue expenditure 100
2. Capital receipts 40
3. Net borrowings 38
4. Net interest payments 27
5. Tax revenue 50
6. Non-tax revenue 15

(B) State any two features of private goods. 1


OR
(C) Explain the ‘reduction of income inequalities among the people’ objective of the 3
Government Budget.
(D) ‘Under the Pradhan Mantri MUDRA Yojana (PMMY), the government 3
provides loans to the non-corporate, non-farm small/micro enterprises.’
Identify and discuss the nature of the government expenditure indicated in the
given statement.
17(a) Giving valid reasons, explain how the following would be treated while estimating 3
National income?
(i) Profits earned by foreign Banks in India.
(ii) Bonus given to railway employees.
(b) "Subsidies to the producers should be treated as transfer payments.” Defend 3
or Refute the given statement with valid reason.
SECTION–B (Indian Economic Development)
18 Identify the correctly matched pair in column A and column B from the following: 1

Column A Column B
1.Planning commission (A) apex body to planning
2. Characteristic of economic planning (B) maximum utilization of resources
3. Objective of economic planning (C) economic instability
4. National development council (D) NITI Aayog

Alternatives:
(a) 1 - (A) (b) 2 - (B) (c) 3 - (C) (d) 4 - (D)

19 In terms of sectoral contribution to GDP, the economy of China is relying more on: 1

(a) Primary sector (c) Tertiary sector


(b) Secondary sector (d) None of these

20 Fixation of maximum land ceiling is one type of _______________ 1


(a) Technical reform (c) structural reform
(b) institutional reform (d) Market reform

21 Study the following picture and answer the given question: 1

People in coastal areas take up the activity shown in the picture as a diversification
activity for sustainable employment. identify the activity:
(a) Poultry (c) Fisheries
(b) Sericulture (d) Farming
22 Read the following Assertion (A) and Reason (R). 1
Choose the correct alternative.
Assertion (A): Modernisation creates unemployment.
Reason (R): Modernisation as a planning objective implies use of advanced
technology that replaces human resources in the same fields.
Alternatives:
(a) Both Assertion (A) and Reason (R) are true and Reason (R) is the correct
explanation of Assertion (A).
(b) Both Assertion (A) and Reason (R) are true, but Reason (R) is not the correct
explanation of Assertion (A).
(c) Assertion (A) is true, but Reason (R) is false.
(d) Assertion (A) is false, but Reason (R) is true.
23 India is not a member of __________________ of the following regional / global 1
economic grouping.
(a) SAARC (c) G-7
(b) BRICS (d) G-20
24 National Bank for Agricultural and Rural Development (NABARD) was set up in 1
1982 as a/the ____________________ body to coordinate the activities of all institutions
involved in the rural financing system.
(a) Micro Credit (c) private credit
(b) Apex (d) Cooperative

25 The nature of Pakistan’s economy is _____________________. 1


(a) Capitalist (c) Socialist
(b) Mixed (d) Communist

26 Read the following statements carefully: 1


Statement 1: Women empowerment is necessary for improving the employment
condition of India.
Statement 2: Almost 50% of the population are women.
In light of the given statements, choose the correct alternative from the following:
(a) Statement 1 is true and Statement 2 is false.
(b) Statement 1 is false and Statement 2 is true.
(c) Both Statements 1 and 2 are true.
(d) Both Statements 1 and 2 are false
27 From the set of the events given in column I and corresponding facts given in 1
Column II, choose the correct pair of statements:
Column I Column II
A India I Economic reforms in 1997
B Pakistan II Economic reforms in 1991
C China III Economic reforms in 1978
D India IV GST rolled out in 1999
(a) A - I (b) B - II (c) C - III (d) D - IV

28 What was the stage of the secondary sector (Industry) on the eve of 3
independence?
OR
Discuss briefly any three salient features of India’s pre-independence occupational
structure.
29 Explain how ‘Investment in Human Capital’ contributes to growth of an economy. 3

30 The phenomenon of male child-preference is common in many developing 4


countries including India, China and Pakistan. Why do people practice
discrimination between male and female children?
31 How is unemployment an economic as well as a social problem? 4
OR
State and elaborate whether the following statements are true or false, with valid
arguments:
(i) Diversification is essential in rural employment generation.
(ii) There are a lot of hindrances in the mechanism of agricultural marketing.
32 4

Identify the situation depicted in the given image. Suggest the impact of the
indicated situation, on the Indian economy.
33(A) “The Green Revolution has made India self-reliant with respect to the food 3
grains.” Justify the statement, giving reasons in support of your answer.
(B) “GST is a game changing reform for the Indian Economy.” Justify. 3
OR
(C) “In India, after 1947 industrial reforms were introduced on a large scale.” In 3
the light of the given statement, discuss any one such industrial reform.
(D) “The Navaratna policy of the government helped in improving the 3
performance of public sector undertakings in India.” Do you agree with the
given statement? Give valid reasons in support of your answer.
34 Read the following text carefully:
Global warming is a gradual increase in the average temperature of the earth’s
lower atmosphere as a result of the increase in greenhouse gases since the
Industrial Revolution. Much of the recent observed and projected global warming
is human-induced. It is caused by man-made increases in carbon dioxide and other
greenhouse gases through the burning of fossil fuels and deforestation. Adding
carbon dioxide, methane and such other gases (that have the potential to absorb
heat) to the atmosphere with no other changes will make our planet’s surface
warmer. The atmospheric concentrations of carbon dioxide and CH4 have
increased by 31 per cent and 149 per cent respectively above pre-industrial levels
since 1750. During the past century, the atmospheric temperature has risen by
1.1°F (0.6°C) and sea level has risen several inches. Some of the longer-term
results of global warming are melting of polar ice with a resulting rise in sea level
and coastal flooding; disruption of drinking water supplies dependent on snow
melts; extinction of species as ecological niches disappear; more frequent tropical
storms; and an increased incidence of tropical diseases. Among factors that may be
contributing to global warming are the burning of coal and petroleum products
(sources of carbon dioxide, methane, nitrous oxide, ozone); deforestation, which
increases the amount of carbon dioxide in the atmosphere; methane gas released
in animal waste; and increased cattle production, which contributes to
deforestation, methane production, and use of fossil fuels. A UN Conference on
Climate Change, held in Kyoto, Japan, in 1997, resulted in an international
agreement to fight global warming which called for reductions in emissions of
greenhouse gases by industrialized nations.
Source: www.wikipedia.org
On the basis of the given text and common understanding, answer the following
questions:
(i) Define Global Warming. 2
(ii) Briefly elaborate two causes and consequences of Global Warming. 4
Kendriya Vidyalaya Sangthan, Jaipur Region
Subject: Economics SET-13
Practice Paper
Class: XII Time: 3:00 Hours Maximum Marks: 80 Marks
General Instructions:
(1) This question paper contains two sections:
Section–A (Introductory Macro Economics) Section–B (Indian Economic Development)
(2) All questions in both the sections are compulsory. However, there is internal choice in some
questions. Marks for questions are indicated against each question.
(3) Question number 1 –10 and 18 – 27 are very short-answer / multiple choice questions
carrying 1 mark each. They are required to be answered in one word or one sentence each.
(4) Question number 11 – 12 and 28 – 29 are short-answer questions carrying 3 marks each.
Answers to them should normally not exceed 60-80 words each.
(5) Question number 13 – 15 and 30 – 32 are also short-answer questions carrying 4 marks
each. Answers to them should normally not exceed 80-100 words each.
(6) Question number 16 – 17 and 33 – 34 are long-answer questions carrying 6 marks each.
Answers to them should normally not exceed 100-150 words each.
(7) Question number 16 – 17 and 33 – 34 are long-answer questions carrying 6 marks each.
Answers to them should normally not exceed 100-150 words each.
(8) An additional 15 minutes has been allotted to read the question paper.

Q. NO. Section-A (Introductory Macro Economics) Marks


1 “CRR cut will improve bottom line”, says Karnataka Bank CEO Private 1
Mahabaleshwara MS on Friday said the Reserve bank of India (RBI) slashing the
Cash Reserve Ratio (CRR) by 100 basis points (BPS) from 4% to 3% will improve
the bottom line of banks. Identify the quantitative function of Central bank among
the given options.
(a) Repo rate (c) Cash reserve ratio
(b) Margin requirement (d) (a) and (c) Both
2 If the value of Average Propensity to Save (APS) is 0.2 and National Income is 1
₹4,000 crore, then consumption will be ______________________.
(a) ₹ 4,000 crores (c) ₹ 3,800 crores
(b) ₹ 3,200 crores (d) ₹ 2,600 crores
3 Balance of Payments account records: 1
(a) A country’s inflow & outflow of foreign exchange during a fiscal year.
(b) International transactions in goods, services and assets during a fiscal year.
(c) Transactions affecting foreign exchange assets & liabilities during a fiscal year.
(d) Transactions relating to exports & import of goods & services, and transfers
during a fiscal year.
OR
From the following given sets of statements in column I and II, chose the correct
pair of statements.
Column I Column II

A Exports of goods and services (i) Excess of Exports of goods over the
imports of goods
B Trade Surplus (ii) An element of invisible items
C Current Transfers to rest of the (iii) Recorded as a positive item in the
world BOP account
D Portfolio Investments (iv) Foreign Institutional investors
Alternatives:
(a) A-(i) (b) B-(ii) (c) C-(iii) (d) D-(iv)
4 On the basis of following schedule, what is the value of MPS? 1

Income (in ₹ crores) 0 50 100 150 200

Saving (in ₹ crores) -20 -10 0 10 20


OR
In case of an underemployment equilibrium, which of the following alternative is not
true?
(a) Aggregate demand is equal to Aggregate supply.
(b) There exist excess production capacity in the economy.
(c) Resources are not fully and efficiently utilized.
(d) Resources are fully and efficiently utilized.
5 Read the following statement – Assertion (A) and Reason (R). 1
Choose one of the correct alternatives given below:
Assertion (A): Demand Deposits are considered as a convenient mode of
payment for execution of even the high value transactions.
Reason (R): Demand Deposits are non-withdrawable in nature and cannot be
withdrawn against issue of cheques and other similar instruments of payment.
Alternatives:
(a) Both Assertion (A) and Reason (R) are true and Reason (R) is the correct explanation
of Assertion (A).
(b) Both Assertion (A) and Reason (R) are true and Reason (R) is not the correct
explanation of Assertion (A).
(c) Assertion (A) is true but Reason (R) is false.
(d) Assertion (A) is false but Reason (R) is true.
6 Read the following statement – Assertion (A) and Reason (R). 1
Choose one of the correct alternatives given below:
Assertion (A): Trade of invisible items between two nations is a part of capital
account of Balance of Payment.
Reason (R): transactions that affect the asset liability status of a country in relation to
the rest of the world are known as capital account transaction.
Alternatives:
(a) Both Assertion (A) and Reason (R) are true and Reason (R) is the correct explanation
of Assertion (A).
(b) Both Assertion (A) and Reason (R) are true and Reason (R) is not the correct
explanation of Assertion (A).
(c) Assertion (A) is true but Reason (R) is false.
(d) Assertion (A) is false but Reason (R) is true.
7 Read the following statements carefully and choose the correct alternatives given 1
below:
Statement 1: Subsidies do not add any burden on the financial health of a nation.
Statement 2: Complete removal of subsidies may violate the aim of equitable
distribution of income.
Alternatives:
(a) Both the statements are true.
(b) Both the statements are false.
(c) Statement 1 is true and Statement 2 is false.
(d) Statement 2 is true and Statement 1 is false.
8 Two friends Riya and Surendra were discussing about aggregate demand. “The 1
components of aggregate demand in an open economy are household
consumption expenditure, private investment expenditure and
government expenditure.”
Identify the one of the components that is missing in the above statement.
(a) Household investment expenditure (c) Factor payments
(b) Net exports (d) Transfer payments
9 Read the following statements carefully and choose the correct alternatives 1
given below:
Statement 1: Demonetization was the step taken by the government of India in order
to tackle the problems of corruption, black money, terrorism and circulation of fake
currency in the Indian economy.
Statement 2: demonetization has ensured improved tax compliance in India over a
period of time.
Alternatives:
(a) Both the statements are true.
(b) Both the statements are false.
(c) Statement 1 is true and Statement 2 is false.
(d) Statement 2 is true and Statement 1 is false.
10 Inventory is a ______________ concept whereas the change in inventory is a 1
______________________ concept
(a) Stock, Flow (c) Stock, Stock
(b) Flow, Stock (d) Flow, Flow
OR
If in an economy the value of Net factor Income from Abroad is ₹ 200 crores and the
value of Factor Income to Abroad is ₹ 40 crores, Identify the value of factor Income
from Abroad.
11 “GDP only measures goods and services produced in an economy and ignores those 3
transactions that do not come under monetary terms”. In view of the above
statement, discuss the limitations of GDP as an index of welfare of a country.
12 The ratio of MPC and MPS is 4:1. The consumption at zero level of income is ₹ 40 3
(A) crores.
(i) Frame a consumption equation
(ii) Find the value of investment multiplier
(iii) Find the break-even level of income
OR
(B) What is investment multiplier? If change in investment of ₹ 100 crores is required to
bring a change in income by ₹ 1000 crores. Calculate MPC and MPS.
13 “Loans are children of deposits and deposits are children of loan” Explain. 4
OR
“The Central Bank acts as a banker, adviser and agent to the government.”
Explain.
14 A country with trade deficit cannot have current account surplus in its balance of 4
payments. Do you agree with the given statement? Give reasons.
15 Define inflationary gap. Show inflationary gap with a well labelled diagram. Give one 4
monetary measure to control it.
16 (A) “Government uses the fiscal policy to stabilize economic fluctuations in an 3
economy”. Discuss.
(B) “Government across nations are worried about the term fiscal deficit”. Do you 3
think fiscal deficit is necessarily inflationary in nature? Support your answer with
reasons.
17 (A) Calculate Gross Domestic Product at factor cost and Factor Income to Abroad 6

Particular ₹ (in crores)


(i) Compensation of employees 800
(ii) Profits 200
(iii) Dividends 50
(iv) GNP at MP 1400
(v) Rent 150
(vi) Interest 100
(vii) Gross Domestic Capital Formation 300
(viii) Net fixed Capital Formation 200
(ix) Change in stocks 50
(x) Factor income from abroad 60
(xi) NIT 120
OR
(B) Elaborate the objective of “Allocation of resources” in the government budget. 3
(C) Discuss briefly how the government budget can be used as an effective tool in the 3
process of employment generation.
SECTION–B (Indian Economic Development)
18 Match the situations given in Column I with their respective implications given in 1
Column II :
Column I Column II

A Migration (i) Reduced per capita economic growth


B Low level of academic (ii) Imbalance between demand and
standards supply of human resource
C Population: High growth rate (iii) Brain drain
D Lack of proper manpower (iv) Mismatch between required skill and
planning academic standards
(a) A-(ii), B-(iii), C-(iv), D-(i) (c) A-(i), B-(ii), C-(iii), D-(iv)
(b) A-(iii), B-(iv), C-(i), D-(ii) (d) A-(ii), B-(iv), C-(i), D-(iii)
19 Just as a country can turn physical resources like land into physical capital like 1
factories, similarly, it can also turn human resources like nurses, farmers, teachers,
students into human capital like engineers and doctors. Which of these justifies the
need for human capital?
(a) It leads to efficient use of resources
(b) It leads to economic growth
(c) It helps in creating a society free of discrimination
(d) It allows individual freedom
OR
Which of the following is working in the informal sector?
(a) A private school teacher in a leading school in a big city.
(b) The owner of a textile shop employing 9 workers.
(c) Driver of a bus company which has more than 10 buses with 20 drivers, conductors
& other workers.
(d) Civil engineer working in a construction company which has 10 workers.
20 Arrange the following events of China and Pakistan in chronological order and 1
choose the correct alternative:
(i) Great Leap Forward (iii) Introduction of reforms in China
(ii) Nationalization of capital goods in Pakistan (iv) Pakistan’s First five year plan
Alternatives:
(a) i, ii, iii, iv (c) iv, i, ii, iii
(b) ii, iii, iv, i (d) iv, iii, ii, i
OR
Which of the following countries has the highest infant mortality rate per 1000 live
births?
(a) India (c) Pakistan
(b) China (d) None of these
21 _________________ adopted “One Child Policy" as a measure to control population. 1
(a) India (c) Pakistan
(b) China (d) Russia
22 Read the following statement – Assertion (A) and Reason (R). 1
Choose one of the correct alternatives given below:
Assertion (A): Basic education and basic health are important in themselves,
irrespective of their contribution to labour productivity.
Reason (R): human welfare should be increased through investment in education
and health even if such investments do not result in higher labour productivity.
Alternatives:
(a) Both Assertion (A) and Reason (R) are true and Reason (R) is the correct explanation
of Assertion (A).
(b) Both Assertion (A) and Reason (R) are true and Reason (R) is not the correct
explanation of Assertion (A).
(c) Assertion (A) is true but Reason (R) is false.
(d) Assertion (A) is false but Reason (R) is true.
23 Read the following statement – Assertion (A) and Reason (R). 1
Choose one of the correct alternatives given below:
Assertion (A): NABARD do not provide direct loan to farmers.
Reason (R): National Bank for Agriculture and Rural Development is an apex
regulatory body for overall regulation and licensing of regional rural banks and
apex cooperative banks in India.
Alternatives:
(a) Both Assertion (A) and Reason (R) are true and Reason (R) is the correct explanation
of Assertion (A).
(b) Both Assertion (A) and Reason (R) are true and Reason (R) is not the correct
explanation of Assertion (A).
(c) Assertion (A) is true but Reason (R) is false.
(d) Assertion (A) is false but Reason (R) is true.
24 __________________ is not the reason for the stagnation in the agriculture sector? 1
(a) Use of fertilizers (c) Low levels of technology
(b) Lack of irrigation facilities (d) Various systems of land settlement
OR
_____________ have been removed to increase the competitive position of Indian goods
in the international markets.
(a) Import licensing (c) Quantitative restrictions
(b) Export duties (d) Tariffs
25 In the year____________________ the Indian Government made education free & 1
compulsory for all children between 6 to 14 years.
(a) 2001 (c) 2003
(b) 2009 (d) 2007
26 Read the following statements carefully and choose the correct alternatives given 1
below:
Statement 1: India announced the New Economic Policy in 1991 consisting of wide
range of economic reforms.
Statement 2: World Bank and International Monetary Fund granted ₴ 7 billion to
manage the Economic crisis in late 1990, with conditions.
Alternatives:
(a) Both the statements are true.
(b) Both the statements are false.
(c) Statement 1 is true and Statement 2 is false.
(d) Statement 2 is true and Statement 1 is false.
27 ________________________ is regarded as architect of Indian Planning, responsible for 1
formulating Five Years Plans.
(a) Prashant Chandra Mahalanobis (c) Dadbhai Naaroji
(b) J.C. Kumarappa (d) Amiya kumar Bagchi
28 The given picture shows the main causes of India’s agricultural stagnation during the 3
colonial period, explain the cause shown in the picture

29 Give two instances of : 3


(i) Overuse of environmental resources
(ii) Misuse of environmental resources
OR
Explain the function of the environment.
30 “The rural areas in India suffer from lack of adequate financing facilities”. State
the reason for the requirement and sources available to meet out this problem.
OR
“Agricultural marketing has come a long way with the intervention of the
government”. Discuss
31 With reference to the table below, list points stating a comparison of the
demographic features of India, China and Pakistan.
Country India China Pakistan
Est. population (in millions) 1311 1371 188
Annual Growth of population 2015 1.2 0.5 2.1
Density (per sq.km) 441 146 245
Sex Ratio 2015 929 941 947
Fertility Rate 2015 2.3 1.6 3.7
Urbanization 2015 33 56 39
32 Do you think that in the last 50-60 years, employment generated in the country is
commensurate with the growth of GDP in India? How?
33 Discuss in brief the goals of five year plans 6
OR
“Though public sector is very essential for industries, many public sector
undertakings incur huge losses and are a drain on the country’s resources”.
Discuss the usefulness of public sector undertakings in the light of this fact.
34 Read the following text carefully and answer questions given below:
NEW DELHI: India and Pakistan – two of the biggest south Asian nations- started
their economic journey around the same time after gaining independence. Not many
know that there was a point in the 1960s when Pakistan’s per capita GDP used to be
higher than that of India’s. However, over the years, India has not only surpassed
Pakistan’s per capita GDP but taken a commanding lead on almost every economic
front. Pakistan today is facing global criticism for failing to rein in homegrown terror
groups even as it grapples with an ailing economy and mounting global debt. India,
on the other hand, has become a global front runner which recently surpassed the UK
to become the fifth largest economy in the world. The stark economic contrast
between the two neighbors shows that Pakistan’s unstable government, conflicting
power centers, convert support to terror groups and involvement in global terror
strikes such as the 26/11 Mumbai attacks, have bled its economy. From a higher per
capita GDP of ₴83.33 in 1960. Pakistan’s today lags much behind India which
witnessed multi-fold rise its per capita GDP over the years. From 2007, India’s per
capita GDP has consistently stayed higher than that of Pakistan’s.
India is today the fastest growing trillion-dollar economy in the world and the fifth
largest overall, according to data compiled by IMF’s World Economic Outlook. It
jumped from 9th spot in 2010 to the 5th spot in a span of just 9 years. The GDP of
India’s is almost 10 times that of Pakistan, which is placed at the 45th position.
India’s rise has been even more dramatic across the past couple of years. Since 2008,
India’s GDP has risen almost 140 per cent as against an increase of about 63 per cent
for Pakistan.
(i) “India and Pakistan started their economic journey at the same time but now India is 3
far ahead of Pakistan’ in the light of above statement, give any two reasons.
(ii) Compare and analyses India and Pakistan economy in the term of GDP growth. 3
Kendriya Vidyalaya Sangthan, Jaipur Region
Subject: Economics SET-14
Practice Paper
Class: XII Time: 3:00 Hours Maximum Marks: 80 Marks
General Instructions:
(1) This question paper contains two sections:
Section–A (Introductory Macro Economics) Section–B (Indian Economic Development)
(2) All questions in both the sections are compulsory. However, there is internal choice in some
questions. Marks for questions are indicated against each question.
(3) Question number 1 –10 and 18 – 27 are very short-answer / multiple choice questions
carrying 1 mark each. They are required to be answered in one word or one sentence each.
(4) Question number 11 – 12 and 28 – 29 are short-answer questions carrying 3 marks each.
Answers to them should normally not exceed 60-80 words each.
(5) Question number 13 – 15 and 30 – 32 are also short-answer questions carrying 4 marks
each. Answers to them should normally not exceed 80-100 words each.
(6) Question number 16 – 17 and 33 – 34 are long-answer questions carrying 6 marks each.
Answers to them should normally not exceed 100-150 words each.
(7) Question number 16 – 17 and 33 – 34 are long-answer questions carrying 6 marks each.
Answers to them should normally not exceed 100-150 words each.
(8) An additional 15 minutes has been allotted to read the question paper.

Q. NO. Section-A (Introductory Macro Economics) Marks


1 Read the following statements carefully: 1
Statement 1: Ex-ante savings are the planned savings or expected savings.
Statement 2: When people who are willing to work at the giving wage rate do not
get work is called involuntary unemployment.
In light of the given statements, choose the correct alternative from the following:

(a) Statement 1 is true and Statement 2 is false.


(b) Statement 1 is false and Statement 2 is true.
(c) Both Statements 1 and 2 are true.
(d) Both Statements 1 and 2 are false.
2 Nominal GNP is same as: 1
(a) GNP at constant prices (c) GNP at current prices
(b) Real GNP (d) GNP less Net factor income from abroad
3 Average Propensity to Consume can never be __________________________.
1
(Choose the correct alternative).
(a) positive (c) more than one
(b) zero (d) less than one
4 As per the Reserve Bank of India (RBI) press report, dated 29th December, 2022: 1
"Net external commercial borrowings to India recorded an outflow of US$ 0.4
billion in the second quarter (2022-23)".
Outflow of foreign exchange is recorded on which side of BOP account?
(a) current, credit (c) capital, credit
(b) current, debit (d) capital, debit
5 ‘Money removes the problem of double coincidence of wants.’ 1
On the basis of the given statement, identify the function performed by money:
(a) Medium of exchange (c) Unit of account
(b) Store of Value (d) Means of standard of deferred payments
6 Find the missing figures and choose the correct alternative: 1
The total deposits are ______________ if LRR is 25% and initial deposits are Rs.
100000?
a) 250000 c) 400000
b) 500000 d) 800000
7 Read the following statements carefully: 1
Statement 1: Aggregate demand is the sum total of consumption and investment
expenditures.
Statement 2: Total consumption consists of an autonomous component and an
induced component.
Choose the correct option based on the above statements.
(a) Statement 1 is true and statement 2 is false
(b) Statement 1 is false and statement 2 is true
(c) Both statements 1 and 2 are true
(d) Both statements 1 and 2 are false
8 Choose the correctly matched pair from the following – 1

Column A Column B
A. Sum of credit is equals to 1. Accommodating items
B. Unilateral transactions 2. Transfer receipts
C. Autonomous items 3. Sum of debit
D. Change in foreign exchange reserves with 4. Transactions above the line
government
(a) A-1 (b) B-2 (c) C-3 (d) D-4
9 The total consumption and investment curves are given below: 1
Identify which of the following represents “Autonomous Consumption”.
a) OR b) RC c) RY d) RI
10 “Interest on the deposit from a foreign bank is recorded in the current 1
account”. Choose the correct reason.
(a) It is visible good (c) Income from abroad
(b) It is invisible service (d) It is a transfer receipt
11 State whether the following statements are true or false. Give reasons for your 3
answer.
(i) Difference between value of exports and imports of goods and services are
called Balance of Trade.
(ii) External assistance is not recorded in Balance of Payments account.
12 Do you agree with the statement, ‘Machine purchased is always a final goods’. 3
Give reason for your answer.
OR
Calculate net value added at market price of a firm:

Sr. No. Items Amount

(i) Sale 300

(ii) Change in stock (-) 10

(iii) Depreciation 20

(iv) Net in direct taxes 30

(v) Purchase of machinery 100

(vi) Purchase of intermediate product 150

13 If in an economy Saving function is given by S = (-) 50 + 0.2 Y and Y = ₹ 2000 4


crores; consumption expenditure for the economy would be ₹ 1,650 crores and
the autonomous investment is ₹ 50 crores and the marginal propensity to consume
is 0.8. True or False?
Justify your answer with proper calculations.
14 ‘An economy is operating at the under-employment level of income’. 4
What is meant by the given statement?
Discuss one fiscal measure and one monetary measure to tackle the situation.
15 As per the following news published in The Economic Times on 26 December 4
th

2021: ‘Reserve Bank of India has sold government securities worth ₹ 8,710
crore in the secondary market, over the last four weeks, to drain out
excessive liquidity’. Identify the likely cause and the consequences behind, this
type of action plan of the Reserve Bank.
16 (A) State which budget expenditure does not result in creation of assets or reduction 3
of liability. Give examples also.
(B) What indicates zero primary deficit? 2
(C) Why is tax treated as revenue receipt? 1
OR
(A) Define the terms “direct” and “indirect” taxes and provide two examples of each. 3
(B) From the following data about a government budget, find out: 3
(i) Revenue deficit (ii) Fiscal deficit (iii) Primary deficit
Sr. No. Particulars ₹ in Arab
(i) Capital receipts net of borrowings 95
(ii) Revenue expenditure 100
(iii) Interest payments 10
(iv) Revenue receipts 80
(v) Capital expenditure 110
17 (a) Giving reason, explain whether the following are included in domestic product 3
of India.
1. Profits earned by a branch of foreign bank in India
2. Payment of salaries to its staff by embassy located in New Delhi
3. Interest received by an Indian resident from its abroad firms.
(b) What is double counting in the economy? How can it be avoided? 3
SECTION–B (Indian Economic Development)
18 Consider the following statement with respect to Indian agriculture and mention 1
the correct combination.
(I) Green Revolution led to the introduction of new technology
(II) Green Revolution started the usage of HYV seeds
(III) Green Revolution resulted in the improvement in oilseeds.
(a) I and II (c) I only
(b) II and III (d) None of the above
19 Which of the following statements about fertility rates is correct? 1
(a) The fertility rate is very low in India and very high in Pakistan
(b) The fertility rate is very low in India and very high in China
(c) The fertility rate is very low in China and very high in Pakistan
(d) The fertility rate is very low in China and very high in Bangladesh
20 Modernization means:- 1
1) Increase the production of goods and services by adopt goods and services by
adopt new technology.
2) Avoiding imports of those goods which could be produced in India itself.
3) Recognition the women should have the same right as men.
Options:
(a) only 1 (c) Only 3
(b) Both 1 and 3 (d) All 1,2,3
21 Study the following picture and answer the given question: 1
_____________ is a village based financial intermediary committee usually composed
of 10-20 local women or men. (Fill in the blank with correct alternative)
(a) RRB (c) NABARD
(b) Self-help group (d) Banking system

22 Read the following statements: Assertion (A) and Reason (R). 1


Choose the correct alternative from those given below:
Assertion (A): The navratnas were granted financial and operational autonomy in
the working of the companies.
Reason (R): The government decided to give special treatment to some profit
making PSU.
Alternatives:
(a) Both Assertion (A) and Reason (R) are true and Reason (R) is the correct
explanation of Assertion (A).
(b) Both Assertion (A) and Reason (R) are true, but Reason (R) is not the correct
explanation of Assertion (A).
(c) Assertion (A) is true, but Reason (R) is false.
(d) Assertion (A) is false, but Reason (R) is true.
23 Which of the following statements about human development is incorrect? 1
(a) The life expectancy at birth of a country is the true indicator of its human
development levels.
(b)The literacy rate of a country is the true indicator of its human development
levels.
C) The density of population of a country is the true indicator of its human
development levels.
(d) The per capita gross domestic product (GDP) of a country is the true indicator
of its human development levels.
24 Read the following statements: Assertion (A) and Reason (R). 1
Choose the correct alternative from those given below.
Assertion (A): Rural development includes only agricultural development.
Reason (R): Rural development aims at improving the economic and social
conditions of well-being.
Alternatives:
(a) Both Assertion (A) and Reason (R) are true and Reason (R) is the correct
explanation of Assertion (A).
(b) Both Assertion (A) and Reason (R) are true, but Reason (R) is not the correct
explanation of Assertion (A).
(c) Assertion (A) is true, but Reason (R) is false.
(d) Assertion (A) is false, but Reason (R) is true.
25 Read the following statement carefully: 1
Statement 1- China has the lowest density among India, Pakistan and China.
Statement 2- Only 64% of people of India live in urban areas.
In light of the given statements, choose the correct alternative from the following:
(a) Both are correct.
(b) Both are incorrect.
(c) Statement 1 is correct and statement 2 is incorrect.
(d) Statement 1 is incorrect and statement 2 is correct.
26 Read the following statements carefully: 1
Statement 1: Casualisation refers to a situation when the percentage of casually
hired workers in the total workforce tends to rise over time.
Statement 2: Informalisation refers to a situation when people tend to find
employment more in the informal sector of the economy, and less in the formal
sector of the economy.
In light of the given statements, choose the correct alternative from the following:
(a) Statement 1 is true and Statement 2 is false.
(b) Statement 1 is false and Statement 2 is true.
(c) Both Statements 1 and 2 are true.
(d) Both Statements 1 and 2 are false
27 From the set of the events given in column I and corresponding facts given in 1
Column II, choose the correct pair of statements:
Column I Column II

A. Great proletarian cultural revolution 1. 1966-67

B. Great leap forward 2. 0.557


C. Decline in Pakistan’s growth rate 3. 1958

D. Value of HDI in Pakistan 4. 2015-17

(a) A-1 (b) B-2 (c) C-3 (D) D-4


28 (A) State the reasons for low agricultural productivity during the colonial period. 3
OR
(B) What were the aims of economic policies pursued by the colonial government in
India?
29 “Ravya’s father admitted her in an engineering college to attain higher 3
education. This contributed positively to her skills and expertise.” Explain the
impact of Ravya’s decision on human capital formation.
30 What are the reasons for the low population growth in China? 4
31 (A) State and elaborate whether the following statements are true or false, with valid 4
arguments:
(i) Agricultural diversification is essential for sustainable livelihoods.
(ii) Moneylender and traders exploits small farmers.
OR
(B) Meena is a housewife. Besides taking care of household chores, she works in the
cloth shop which is owned and operated by her husband. Can she be considered as
a worker? Why?
32 Identify the situation depicted in the given image. Suggest the impact of the 4
indicated situation, on the Indian economy.

33 (A) “The goal of equitable distribution of land was fully served by the abolition 3
of intermediaries, in post-independence India”. Justify the statement, giving
reasons in support of your answer.
‘India’s Green Revolution is an example of how the productivity of scarce 3
land resources can be increased with improved production technology’.
Justify the statement, giving reasons in support of your answer.
OR
(C) ‘Though public sector is very essential for industries, many public sector 3
undertakings incur huge losses and are a drain on the economy’s
resources’. Discuss the usefulness of public sector undertakings in the light of
this fact.
(D) ‘Modernisation as a planning objective create contradiction in the light of 3
employment generation.’ Do you agree with the given statement? Give valid
reasons in support of your answer.
34 Read the text carefully:
An important lesson that the COVID-19 pandemic has taught the policymakers in
India is to provide greater impetus to sectors that make better allocation of
resources and reduce income inequalities. COVID-19 has also taught a lesson that
in crisis the population returns to rely on the farm sector. India has a large arable
land, but the farm sector has its own structural problems. However, directly or
indirectly, 50 percent of the households still depend on the farm sector. Greater
support to MSMEs, higher public expenditure on health and education and making
the labour force a formal employee in the economy are some of the milestones that
the nation has to achieve.One of the imminent reforms to be done in the country is
labour reforms. Labour laws are outmoded in India, and some of these date back to
the last century. India’s complex labour laws have been blamed for keeping
manufacturing businesses small and hindering job creation. Industry hires labour
informally because of complex laws and that is responsible for low wages.
Answer the given questions on the basis of common understanding:
(i) “It is necessary to create employment in the formal sector rather than in the 3
informal sector.’’ Defend or refute the given statement with valid argument.
(ii) Hired labour comes in _______________________ (Informal organisation / formal 1
organisation)
(iii) What do you mean by MSMEs? 2
Kendriya Vidyalaya Sangthan, Jaipur Region
Subject: Economics SET-15
Practice Paper
Class: XII Time: 3:00 Hours Maximum Marks: 80Marks
General Instructions:
(1) This question paper contains two sections:
Section–A (Introductory Macro Economics) Section–B (Indian Economic Development)
(2) All questions in both the sections are compulsory. However, there is internal choice in some
questions. Marks for questions are indicated against each question.
(3) Question number 1 –10 and 18 – 27 are very short-answer / multiple choice questions carrying
1 mark each. They are required to be answered in one word or one sentence each.
(4) Question number 11 – 12 and 28 – 29 are short-answer questions carrying 3 marks each.
Answers to them should normally not exceed 60-80 words each.
(5) Question number 13 – 15 and 30 – 32 are also short-answer questions carrying 4 marks each.
Answers to them should normally not exceed 80-100 words each.
(6) Question number 16 – 17 and 33 – 34 are long-answer questions carrying 6 marks each.
Answers to them should normally not exceed 100-150 words each.
(7) Question number 16 – 17 and 33 – 34 are long-answer questions carrying 6 marks each.
Answers to them should normally not exceed 100-150 words each.
(8) An additional 15 minutes has been allotted to read the question paper.

Q. NO. Section-A (Introductory Macro Economics) Marks


Q. 1 Define a Capital goods. 1
Q. 2 Money is: 1
(a) Anything which does not requires double coincidence of wants.
(b) Anything which is commonly accepted as a medium of exchange.
(c) Anything which is commonly accepted as commodity money.
(d) None of these
Q. 3 What are demand deposits? 1
Q. 4 AD curve: 1
(a) is a diagrammatic presentation of AD schedule.
(b) Shows desired expenditure corresponding to different levels of income in the
economy.
(c) Indicates ex-post demand in the economy.
(d) Both (a) and (b)
Q. 5 In the consumption function, C = 400 + .6y, the value of autonomous saving will be: 1
(a) 400 (b) (-) 400 (c) 0.6 (d) 0.4
Q. 6 The focus of government budget is: 1
(a) to maximize fiscal deficit (c) to maximize expenditure
(b) to minimize fiscal deficit (d) to maximize revenue
Q. 7 The programmers and policies of government presented in the budget are known 1
as__________. (fiscal policy / monetary policy)
Q. 8 State whether the given statement is true or false: 1
‘Managed Floating Exchange Rate is decided by market forces but is not
controlled by central bank’.
Q. 9 Government expenditure on Mid-Day-Meal scheme running in government schools 1
is a type of ___________ Expenditure in government budget.
(Fill up the blank with correct answer)
Q. 10 If MPC = .5, value of investment multiplier is: 1
(a) 2 (b) 3 (c) 4 (d) 5
Q. 11 Give one example of ‘externality’ which reduces welfare of the people. 3
Q. 12 Complete the following table: 3
Consumption expenditure Saving Income MPC
100 50 150 -
175 75 - -
250 100 - -
325 125 - -
OR
What is the ‘inflationary gap’? Explain the role of open market operations in
removing this gap.
Q. 13 Explain the ‘banker’s bank’ function of central bank. 4
Q. 14 What do you mean by balance of payments? Write the components of current 2+2
account of balance of payments.
OR
What is meant by ‘depreciation and appreciation’ of domestic currency. Give
examples.
Q. 15 Give reason whether the following will include in revenue receipt, capital receipt, 2+2
revenue expenditure and capital expenditure in a government budget.
(a) Disinvestment (c) Repayment of loan
(b) Fee and fine (d) Pension
Q. 16 Calculate from the given data: 3+3
(i) Operating surplus
(ii) Domestic Income
Particulars Rs. in crores
(i) Compensation of employee 2000
(ii) Rent and interest 800
(iii) Indirect taxes 120
(iv) Corporation tax 460
(v) Consumption of fixed capital 100
(vi) Subsidies 20
(vii) Dividends 940
(viii) Undistributed profits 300
(ix) Net factor income to abroad 150
(x) Mixed income 200
OR
(a) Distinguish between stock and flow concept of national income.
(b) Define intermediate and final goods. Give examples.
Q. 17 In an economy C= 400 + .5Y is the consumption function where C is consumption 3+3
expenditure and Y is national income. Investment expenditure is 5000. Calculate
equilibrium level of income and consumption expenditure.
SECTION–B (Indian Economic Development)
Q. 18 Name any two taxes which were subsumed in Goods and Services Tax (GST). 1
Q. 19 Planning commission in India has been replaced by NITI AAYOG (True / False) 1
Q. 20 Identify the correct sequence of alternatives given in Column II by matching them 1
with respective terms in Column I:
Column – I Column – II
A. Land Ceiling i. Increase in production of food grain using high
yielding variety seeds
B. Land reforms ii. Portion of agricultural produce sold in the market
C. Green Revolution iii. Fixing the maximum limit of land holding for an
individual
D. Marketed Surplus iv. Change in the ownership of land (land to tillers)

Choose the correct alternative:


(a) ii, iv, iii, i (c) ii, iv, i, iii
(b) iii, iv, i, ii (d) iv, i, ii, iii
Q. 21 Identify the correct sequence of alternatives given in column II by matching them 1
with respective items in column I :
Column – I Column – II
(i) Private sector (a) uses more of labour than capital
(ii) Small-scale industry (b) aimed to promote regional equality
(iii) Karve committee (c) controlled and run by private individuals
(iv) Licensing policy of the (d) 1955
government
Choose the correct alternative:
(a) ii, iv, iii, i (c) ii, iv, i, iii
(b) iii, iv, i, ii (d) iii, i, iv, ii
Q. 22 The Great leap forward campaign was launched in ________________________. (Name of 1
the country)
Q. 23 Arrange the following events of China in chronological order and choose the 1
correct alternate:
i. Great Proletarian Cultural Revolution iii. Introduction of Economic Reforms
ii. Great Leap Forward campaign iv. First five year plan
Choose the correct alternative:
(a) ii, iv, iii, i (c) ii, iv, i, iii
(b) iv, ii, i, iii (d) iv, i, ii, iii
Q. 24 Brain-drain is a major problem of human capital formation in India. (True / False) 1
Q. 25 The basis objective of the cooperative credit societies is ____________. 1
(a) To ensure timely flow of credit to the farmer
(b) To eliminate the moneylenders from the rural scene
(c) To make available credit facilities to all the region
(d) All of these
Q. 26 Work -force refers to that part of: 1
(a) Labour force which is employed (c) Population which is forced to work
(b) Population which is unemployed (d) Labour force which is unemployed
Q. 27 Which of the following is not a component of economic infrastructure? 1
(a) Health of human (c) Communication system
(b) Power plants (d) Transport system
Q. 28 Explain any three challenges of rural areas regarding rural development. 3
OR
What are the main deficiencies of rural banking system?
Q. 29 Explain the relationship between human capital and economic growth. 3
Q. 30 How does British rule in India enforced to the decay of Indian handicrafts? State 4
the possible factors.
Q. 31 What do you understand by the term ‘import substitution’? Explain the role of 4
tariffs and quotas in to achieve import substitution.
OR
Was demonetization a good policy decisions? Give your opinion.
Q. 32 Read carefully the given paragraph and write the answer of following questions:- 4
Workers who own and operate an enterprise to earn their livelihood are known as
self-employed. Thus the cement shop owner is self-employed. About 52 per cent
workforce in India belongs to this category. The construction workers are known
as casual wage laborers; they account for about 25 per cent of India’s workforce.
Such laborers are casually engaged in others’ farms and, in return, get
remuneration for the work done. Workers like the civil engineer working in the
construction company account for 23 per cent of India’s workforce. When a
worker is engaged by someone or an enterprise and paid his or her wages on a
regular basis, they are known as regular salaried employees.
Define the followings:
(a) Self- employed (c) Regular salaried employees
(b) Casual wage labours (d) Hired worker.
Q. 33 (i) Write any three differences between physical and human capital. 3+3
(ii) Write short note on global warming.
Q. 34 Compare and analysis the given data of distribution of workforce (in 2018-2019) 6
with valid reason:
Country Agriculture Industry Service
INDIA 43 25 32
CHINA 26 28 46
PAKISTHAN 41 24 35
OR
What similar developmental strategies have India and Pakistan followed for their
respective developmental path? Explain any four points.
Kendriya Vidyalaya Sangthan, Jaipur Region SET-16
Subject: Economics
Practice Paper
Class: XII Time: 3:00 Hours Maximum Marks: 80 Marks
General Instructions:
(1) This question paper contains two sections:
Section–A (Introductory Macro Economics) Section–B (Indian Economic Development)
(2) All questions in both the sections are compulsory. However, there is internal choice in some
questions. Marks for questions are indicated against each question.
(3) Question number 1 –10 and 18 – 27 are very short-answer / multiple choice questions
carrying 1 mark each. They are required to be answered in one word or one sentence each.
(4) Question number 11 – 12 and 28 – 29 are short-answer questions carrying 3 marks each.
Answers to them should normally not exceed 60-80 words each.
(5) Question number 13 – 15 and 30 – 32 are also short-answer questions carrying 4 marks
each. Answers to them should normally not exceed 80-100 words each.
(6) Question number 16 – 17 and 33 – 34 are long-answer questions carrying 6 marks each.
Answers to them should normally not exceed 100-150 words each.
(7) Question number 16 – 17 and 33 – 34 are long-answer questions carrying 6 marks each.
Answers to them should normally not exceed 100-150 words each.
(8) An additional 15 minutes has been allotted to read the question paper.

Q. NO. Section-A (Introductory Macro Economics) Marks


1 In the following questions, two statements are given. Read the statements carefully 1
and choose the correct alternative :
Statement 1: When income are zero consumption never be zero.
Statement 2:. In initial period APC will more than one.
Alternatives:
(a) Both the statements are true
(b) Both the statements are false
(c) Statement 1 is true and statement 2 is false
(d) Statement 2 is true and statement 1 is false
2. A company located in India receives a loan from a company located abroad. 1
How is this transaction recorded in India’s Balance of payments account?
(a) Credit side of current account
(b) Debit side of current account
(c) Credit side of capital account
(d) Debit side of capital account
3 Which one of following is not a function of the Reserve bank of India? 1
(a) Issue of currency
(b) Banker to the public
(c) Banker to Government
(d ) Controller of money supply
4 Read the following Assertion (A) and Reason (R). 1
Choose the correct alternative.
Assertion (A): Imports of crude oil by India will have a favourable impact on
Balance of payments of India.
Reason (R): Imports of goods are recorded on the debit side of current
account as it leads to outflow of foreign exchange.
Alternatives:
(a) Both Assertion (A) and Reason (R) are true and Reason (R) is the correct
explanation for Assertion (A).
(b) Both Assertion (A) and Reason (R) are true and Reason (R) is not the correct
explanation for Assertion (A).
(c) Assertion (A) is true but Reason (R) is false.
(d) Assertion (A) is false but Reason (R) is true.
5 Read the following Assertion (A) and Reason (R). 1
Choose the correct alternative.
Assertion (A): Cement purchase by the constructor is considered and
intermediate good.
Reason (R): The cement is used for the construction of a building to be sold to
the consumer.
Alternatives:
(a) Assertion(A) is true and Reason (R) is a correct reason of assertion (A).
(b) Assertion (A) is true but Reason (R) are is not a correct reason of Assertion
(A) .
(c) Assertion(A) a is false Reason (R) are is true
(d) both are false
OR
National income = domestic income, net factor income from abroad is negative:
true or false
6. Suppose in a hypothetical economy, the income rises from ₹ 2,000 cores to ₹ 1
4,000 cores. As a result, the consumption expenditure rises from ₹ 1,000 to ₹
2,000 cores. Marginal propensity to consume in such case would be ___________.
(a) 0.8 (c) 0.2
(b) 0.4 (d) 0.5
OR
When MPS = 1 , then K ( investment multiplier) is :
(a) 1 (c) ∞ (infinity)
(b) 0 (d) None of these
7 Which of the following institution (s) perfume the activity of credit creation? 1
(a) Commercial banks (c) Both (a) and (b)
(b) Central bank (d) None of these
8 Foreign exchange transactions which are independent of other transactions in 1
the Balance of payment accounts are called :
(a) Current transactions (c) Autonomous transactions
(b) Capital transactions (d) Accommodating transactions
OR
From the set of statements given in column I and column II, chose the correct
pair of statements:
Column I Column II
(a) Export of software to France (i) Debit side of current account
(b) Import of Machinery from China (ii) Capital Account of Balance of
Payments
(c) Remittances to relative staying (iii) Debit side of Current Account of
abroad Balance of Payments
(d) Investment by Apple phones firm (iv) Credit side of Current Account of
in India Balance of Payments
9 Read the following Assertion (A) and Reason (R). 1
Choose the correct alternative.
Assertion (A): MPC represents the slope of the consumption function.
Reason (R): MPC represents change in consumption due to a given change in
income.
Alternatives:
(a) Both assertion (a) and Reason (R) are true and Reason (R) is the correct
explanation for Assertion (A).
(b) Both Assertion (A) and Reason (R) are true but Reason (R) is not the correct
explanation for Assertion (A).
(c) Assertion (A) is true but Reason (R) is false.
(d) Assertion (A) is false but Reason (R) is true.
10 Read the following Assertion (A) and Reason (R). 1
Choose the correct alternative.
Assertion (A): APC is continuously increasing as income increases; and APS is
continuously decreasing as income increases.
Reason (R): As income increases, the proportion of income saved increases
and the proportion of income consumed decreases.
Alternatives:
(a) Both assertion (a) and Reason (R) are true and Reason (R) is the correct
explanation for Assertion (A).
(b) Both Assertion (A) and Reason (R) are true but Reason (R) is not the correct
explanation for Assertion (A).
(c) Assertion (A) is true but Reason (R) is false.
(d) Assertion (A) is false but Reason (R) is true.
11 Compute National income from the following data: 3
Sr. no Particulars Amount (₹ in crores )
(i) Private final consumption expenditure 1000
(ii) Government final consumption expenditure 550
(iii) Net imports 30
(iv) Gross domestic capital formation 250
(v) Change in stock 50
(vi) Net domestic fixed capital formation 180
(vii) Net indirect taxes 100
(viii) Net factor income from abroad (-) 40
(ix) Profits 100
12 Distinguish between Autonomous and accommodating transactions in balance 3
of payments accounts.
OR
What is foreign exchange rate? Distinguish between fixed and flexible exchange
rates.
13 In an economy 50% of the increase in income is spent on consumption. 4
Investment is increased by ₹ 1000 corers. Calculate :
(a) Total increase in income
(b) Total increase in consumption expenditure.
14 “Monetary measure offers a valid solution to the problem of inflationary 4
gap in an economy”. State and discuss any two monetary measures to justify
to given statement.
15 Discuss any one of the following functions of a central bank : 4
(a) As government bank
(b) Open market operations
OR
Assuming that initial deposit with bank ₹ 1000 crore legal reserve ratio 20%,
Explain the process of credit creation by the bank.
16 (A) “Real gross domestic product is a better indicator of economic growth 3+3
then nominal Gross domestic Product.”Do you agree with given statement?
Support your answer with the suitable numerical example.
(B) Calculate ‘Depreciation on capital asset’ From the following data:
Sl.no. Particulars Amount (₹ In crore)
(i) Capital value of the asset 1,000
(ii) Estimated life of the asset 20 years
(iii) Scrap value nil

17(A) Elaborate the objective of ‘Redistribution of income and wealth’ in the 3+3
government budget.
(B) Distinguish between revenue Expenditure and capital Expenditure of the
government, with suitable examples.
OR
Identify the following as Revenue expenditure or capital expenditure. Give
reason
(i) Salary paid to government employee
(ii) Purchase of machine from Korea.
(iii) Repayment of loan taken from IMF.
(iv) Interest paid on national debt.
(v) Expenditure on construction of dam.
(vi) Social security benefits paid to retired government employee.
SECTION–B (Indian Economic Development)
18 Major contribution to the GDP of the country on the eve of independence was 1
from:
(a) Secondary sector (c) Primary sector
(b) Tertiary sector (d) None of these
19 In the Industrial Policy Resolution of 1956, industries were classified in 1
__________________________ Categories.
(a) Two (c) four
(b) three (d) five
OR
In 1955, Karve Committee was constituted for aiming the _________________.
(a) Modernization (c ) Development of small scale industries
(b) Industrial development (d) Self-reliance
20 Introduction of Economic Reforms in China took place in the year _______________ 1
(a) 1978 (C ) 1988
(b) 1980 (d) 1991
OR
Choose the correct alternative showing chronological order of the following
event:
(i) One child policy in China
(ii) Economic reform in Pakistan
(iii) New economic policy in India
(iv) Commune system in agriculture in China
Alternatives:
(a) ii,iv,iii,i (c) iii,iv,i,ii
(b) iv,ii,i,iii (d) iv,i,ii,iii
21 National Bank for Agriculture and Rural Development (NABARD) was set up in 1
____________________.
(a) 1980 (C ) 1982
(b) 1981 (d) 1983
22 Identify the correct pair by matching the respective events of China in column I 1
with the year of occurrence in column II:
Column I Column II
(A) Introduction of economic reforms in China (i) 1949
(B) Establishment of People’s Republic of China. (ii) 1953
(C) The Commune system in China. (iii) 1978
(D) Great Leap Forward campaign initiated in China. (iv) 1958
(a) A- i (c ) C-iii
(b) B-ii (d) D-iv
23 Read the following statements carefully and choose the correct alternative: 1
Statement-I: Human capital is not sold in the market; only the services of the
human capital are sold.
Statement-II: Human capital is perfectly mobile between countries.
Alternatives:
(a) Both the statements are true.
(b) Both the statements are false.
(c) Statement-I is true, Statement-II is false.
(d) Statement-II is true, statement -I is false.
24 Read the following statements carefully and choose the correct alternative: 1
Statement-I: while India and Pakistan became independent nations in 1947,
People’s Republic of China was established in 1949.
Statement-II: while India announced its first Five Year Plan in 1951, Pakistan
announced its first five year plan, now called the Medium Term Development
Plan, in 1953. China announced its First Five Year Plan in 1956.
Alternatives:
(a) Both the statements are true.
(b) Both the statements are false.
(c) Statement-I is true, Statement-II is false.
(d) Statement-II is true, statement -I is false.
25 The use of __________________________ as fuel in public transport system may 1
reduce air pollution.
(a) LPG (c) water
(b) CNG (d) Electricity

OR
_______________________ emphasises on protecting the future generation.
(a) Edward Barbier
(b) United Nations Conference on Environment and Development (UNCED)
(c) Brundtland Commission
(d) Herman Daly
26 Read the following statements carefully and choose the correct alternative: 1
Statement-I: Work at home includes not only traditional work like weaving,
lace making or variety of handicrafts but also modern jobs like programming
work in the IT industry.
Statement-II: During Covid 19 pandemic in 2020-21, millions of workers
delivered their products and services through work from home.
Alternatives:
(a) Both the statements are true.
(b) Both the statements are false.
(c) Statement-I is true, Statement-II is false.
(d) Statement-II is true, Statement-I is false.
27 Read the following Assertion (A) and Reason (R). 1
Choose the correct alternative.
Assertion (A): The Great Leap Forward (GLF) campaign initiated in 1958 met
with many problems.
Reason (R): A severe drought caused havoc in China killing about 30 Million
people. When Russia had conflicts with China, it withdrew its professionals
who had earlier been sent to China to help in the industrialisation process.
Alternatives:
(a) Both assertion (a) and Reason (R) are true and Reason (R) is the correct
explanation for Assertion (A).
(b) Both Assertion (A) and Reason (R) are true but Reason (R) is not the correct
explanation for Assertion (A).
(c) Assertion (A) is true but Reason (R) is false.
(d) Assertion (A) is false but Reason (R) is true.
28 Are environment crises a recent phenomenon? If so, why? 3
29 Explain the role of Micro-credit programmers in meeting the credit 3
requirements of the poor.
OR
‘In recent times the Indian Economy has experienced the problem of
Casualization of the workforce. This Problem has only been aggravated
by the outbreak of COVID-19.’ Do you agree with the given statement?
Discuss any two disadvantages of casualization of the workforce in the light of
the above statement.
30 Study the following table showing distribution by workforce by Industry, 2011- 4
12 and analyse the trend of the workforce on the basis of industry and
residence.
Distribution of workforce by Industry, 2011-12
Place of residence
Industrial Category Rural Urban
Primary Sector 64.1 6.7
Secondary Sector 20.4 35.0
Tertiary Sector 15.5 58.3
Total 100.0 100.0
31 Explain how Goods and Services tax (GST) has Simplified the multiplicity of 4
taxes on goods and services.
OR
Explain types of land reforms / institutional reforms implemented in the
agriculture sector in India.
32 “Human capital formation gives birth to innovation, invention and 4
technological improvements.” Do you agree with the given statement?
Support your answer with valid arguments.
33 Critically evaluate the role of the rural credit / banking system in the process of 6
rural development in India. Also, give some suggestions to improve the
situation.
OR
Define Human Capital Formation. What is the source of human capital
formation? Explain.
34 Read the following text carefully :

Special Economic Zones (SEZs) are intended to function as zones of rapid


economic growth by using tax and business incentives to attract foreign
investment and technology. The first four special economic zones were created
in 1980 in southern coastal China and consisted of what were then the small
cities of Shenzhen, Zhubai, and Shantou in Guangdong province and Xiamen
(Amoy) in Fujian province. In these areas, local governments have been
allowed to offer tax incentives to foreign investors and to develop their own
infrastructure without the approval of the central government. Business
enterprises have made most of their own investment, production and
marketing decisions, and foreign ownership of such ventures has been
legalised. Though some of them began as little more than small towns, the new
SEZs soon attracted foreign investment and became boom towns, with rapidly
expanding light and consumer goods industries and growing population.
Encouraged by the zones success, the Chinese government in 1984 opened 14
larger and older cities along the coast to foreign trade and investment. In 1988,
Haiman Island was made a separate province and Special Economic Zone, and
in 1990 the Pudong area within the Shanghai municipality became a Special
Economic Zone with policies even more flexible than those already in force in
the original four Special Economic Zones (SEZs).
Answer the given questions on the basis of common understanding:

(a) What are Special Economic Zones (SEZs)? Name any two cities in which 3
Special Economic Zones were started in China in 1980.
(b) When were economic reforms initiated in China? What autonomy was given to 3
local government for motivating foreign investors in China under Special
Economic Zones (SEZs)?

You might also like